Anda di halaman 1dari 243

Short Notes and

Short Cases in
SURGERY
Short Notes and
Short Cases in
SURGERY

UN Panda
MD
Senior Physician
New Delhi

JAYPEE BROTHERS
MEDICAL PUBLISHERS (P) LTD
New Delhi
Published by
Jitendar P Vij
Jaypee Brothers Medical Publishers (P) Ltd
EMCA House, 23/23B Ansari Road, Daryaganj
New Delhi 110 002, India
Phones: 23272143, 23272703, 23282021, 23245672, 23245683
Fax: 011-23276490
E-mail: jpmedpub@del2.vsnl.net.in
Visit our website: http://www.jpbros.20m.com

Branches
• 202 Batavia Chambers, 8 Kumara Kruppa Road, Kumara Park East,
Bangalore 560 001, Phones: 2285971, 2382956 Tele Fax: 2281761
e-mail:jaypeebc@bgl.vsnl.net.in
• 282 IIIrd Floor, Khaleel Shirazi Estate, Fountain Plaza
Pantheon Road, Chennai 600 008, Phone: 28262665 Fax: 28262331
e-mail:jpmedpub@md3.vsnl.net.in
• 4-2-1067/1-3, Ist Floor, Balaji Building, Ramkote Cross Road,
Hyderabad 500 095, Phones: 55610020, 24758498 Fax: 24758499
e-mail:jpmedpub@rediffmail.com
• 1A Indian Mirror Street, Wellington Square
Kolkata 700 013, Phone: 22451926 Fax: 22456075
e-mail:jpbcal@cal.vsnl.net.in
• 106 Amit Industrial Estate, 61 Dr SS Rao Road, Near MGM Hospital
Parel, Mumbai 400 012 , Phones: 24124863, 24104532 Fax: 24160828
e-mail:jpmedpub@bom7.vsnl.net.in

Short Notes and Short Cases in Surgery

© 2003, UN Panda
All rights reserved. No part of this publication should be reproduced, stored in a
retrieval system, or transmitted in any form or by any means: electronic,
mechanical, photocopying, recording, or otherwise, without the prior written
permission of the author and the publishers.
This book has been published in good faith that the material provided by author
is original. Every effort is made to ensure accuracy of material, but the publisher,
printer and author will not be held responsible for any inadvertent error(s). In case
of any dispute, all legal matters to be settled under Delhi jurisdiction only.

First Edition: 2003

Publishing Director : RK Yadav

ISBN 81-8061-131-0
Typeset at: JPBMP typesetting unit
Printed at: Gopsons Papers Ltd., A-14, Sector 60, Noida
Preface

Short Notes and Short Cases in Surgery is a handbook specially


structured for medical undergraduates. It depicts the basic themes
of surgical conditions in question-answer format for quick revision,
easy assimilation and effective reproduction in examination. The
answers are crisp and concise and can be elaborated by the student
at will. Greater emphasis has been laid on commonly encountered
surgical conditions and well-accepted practices and procedures.
Students will find it handy and very useful while preparing for
examination, both theory and clinical. The short cases discussed in
detail are the ones commonly presented to students during
undergraduate examinations. So the students will have the feel of
the clinical and viva voce while going through these short cases.
I am sure, the book though small, will be of immense help to the
undergraduate medical students. All suggestions for improvement
are cordially welcome.

UN Panda
Contents

Short Notes
1. Skin and Subcutaneous Tissue ................................... 1
2. Sinus and Fistula ......................................................... 12
3. Swellings in Neck ......................................................... 16
4. Lesions of Oral Cavity ................................................ 30
5. Varicose Veins .............................................................. 33
6. Hernia ............................................................................. 39
7. Testis, Scrotum and Penis ......................................... 51
8. Anal Canal and Rectum .............................................. 59
9. Diseases of Intestines.................................................. 63
10. Diseases of Breast ........................................................ 74
11. Diseases of Prostate .................................................... 81
12. Malignancies of Urinary Tract .................................. 86

Short Cases
Cases 1 to 30 .......................................................................... 90
READER SUGGESTIONS SHEET
Please help us to improve the quality of our publications by completing and returning
this sheet to us.

Title/Author: Short Notes and Short Cases in Surgery by UN Panda

Your name and address:

Phone and Fax:

e-mail address:

How did you hear about this book? [please tick appropriate box (es)]

# Direct mail from publisher # Conference # Bookshop

# Book review # Lecturer recommendation # Friends

# Other (please specify) # Website

Type of purchase: # Direct purchase # Bookshop # Friends

Do you have any brief comments on the book?


Please return this sheet to the name and address given below.

JAYPEE BROTHERS
MEDICAL PUBLISHERS (P) LTD
EMCA House, 23/23B Ansari Road, Daryaganj
New Delhi 110 002, India
Short Notes

1
Skin and Subcutaneous Tissue

Dermoid Cyst
Dermoids can be sequestration dermoid, implantation dermoid,
tubulodermoid and teratodermoid. Sequestration dermoid is a the
congenital cyst arising from the primitive ectodermal cells burried
along the lines of embryonic fusion. Common sites are midline of
the body, especially at root of nose, and in the neck, on the scalp, at
the inner and outer angles of eyes. The cyst is lined by squamous
epithelium with hair follicles, sweat and sebaceous glands. The
content of cyst is toothpaste like material, a mixture of sebum, sweat
and desquamated debris. The overlying skin can be pinched off
from the cyst.
The cyst has a smooth surface, is floctuant non-compressible
and transillumination negative. Complications include infection
and ulceration.
Implantation dermoid is due to inward migration of surface
epithelium following trauma-surgical or otherwise. It is found in
areas subjected to repeated trauma like pulp of fingers, palm and
sole. It is common to tailors and gardeners. Often the cyst is painful
and tender. Though it is lined by squamous epithelium, hair
follicles, sweat and sebaceous glands are absent. The cyst may feel
tense and hard, often stony hard.
Tubulodermoid arises from non-obliterated ectodermal duct, eg.
thyroglossal cyst, post anal dermoid, ependymal cyst. In
teratodermoid the cystic swelling arises from totipotent cells with
ectodermal preponderance.
All dermoids need excision by marsupialization. Those occuring
on skull be differentiated from meningocele that have impulse on
coughing and bone erosion.
2 SHORT NOTES AND SHORT CASES IN SURGERY

Sebaceous Cyst
It is a retention cyst due to accumulation of sebum consequent to
obstruction of sebaceous duct. It can occur in any body part except
palm and sole where sebaceous glands are absent. It contains
yellowish-white poultice cheesy material with an unpleasant smell.
The cyst is globular with smooth surface, adherent to skin. It is
non-compressible, but floctuation may be positive. Transillu-
mination is also negative because of thick content. The discovery
of a blue punctum indicates site of blockage of sebaceous duct.
Scrotal sebaceous cyst are usually multiple and in them punctum
is usually not visible.
Cock’s peculiar tumor is infected ulcerated sebaceous cyst
mimicking squamous cell carcinoma due to granulomatous
reaction. Sebaceous horn is accumlation and drying up of leaked
sebaceous material from the cyst. Excision is the treatment of choice.

Sebaceous Adenoma
It is a benign tumor arising from sebaceous gland. When occuring
on nose it is called rhinophyma when occuring on face and frontal
region, it may be associated with intracranial lesions. (tuberous
sclerosis) causing epilepsy and mental retardation.
Exocrine glands on body are three types.
1. Holocrine—where the entire cell disintegrates to liberate its
secretion, e.g. sebaceous gland
2. Apocrine—where only the terminal part of the cell disintegrate
leaving the nucleus and basal portions intact from which the
cell regenerates (e.g. mammary gland)
3. Merocrine—where the secretion is discharged without
destruction of the cell, e.g. most sweat glands.

Lipoma
Lipoma is a benign connective tissue tumor arising from fat cell. It
can be encapsulated or diffuse (pseudolipoma). It can occur in
subcutaneous tissue any where but has predilection for nape of
neck. It can occur in skin, muscle, periosteum, mucous membrane,
within joint, meninges also retropetritoneum. It can have fibrous
tissue (filbrolipoma), blood vessels (naevolipoma), nerve tissue
(neurolipoma) naevolipoma has bluish colour, is compressible and
blanches on pressure. Neurolipoma is painful. Presence of diffuse
SKIN AND SUBCUTANEOUS TISSUE 3

neurolipoma is called Dercum’s disease or adeposis dolorosa. It is


mostly seen in women affecting trunk, buttock and thighs.
Lipomas can be sessile or pedunculated. The cut surface often
may show patchy calcification and myxomatous degeneration.
Subcutaneous lipoma is a painless, freely mobile, soft, solid
swelling often lobulated, with well-defined but slipping edge. It is
non-floctuant and transillumination is negative, Complications
include infection, calcification, malignancy (liposarcoma),
myxomatous degeneration. Retroperitonial lipoma-lipoma in
informular septa thigh and loponea on neck have predilection for
malighany. Lipomas need excision when they are symptomatic due
to compression of adjacent nervel or vessel. They are painful and
interfere with movement or muscular function or cause cord
compression (intracranial lipoma does not occur due to absence of
fat in extradural tissue of skull).

Haemangioma
Haemangioma is developmental malformation of blood vessels, a
hamartoma, than a true tumor. It commonly occurs in skin and
subcutaneous tissue but can occur in any organ of body like tongue,
lung, liver and brain. The commonly encountered types are
capillary, cavernous and plexiform.
The common from of Capillary haemangioma are salmon pink
patch, port-wine stain, straw berry angioma and spider naevus.
Salmon pink patch is present since birth, common over forehead
and face and disappears by 1 year of age port-wine stain is a diffuse
telangiectasia of face, lip, and buccal mucosa that blanches on
pressure. It persists throughout life and may be associated with
ipsilateral cerebral haemangioma, often causing epilepsy.
Strawberry angioma appear during neonatal period and enlarge
in size into one year of age to completely involute by 7-8 years.
Though it can occur on any part; head and neck is the most favoured
site. The lesion is dark or bright, red hemispherical swelling, soft
compressible non-pusatile with irrigular surface. Excision with or
without skin grafting or injection of sclerosants or application of
carbondioxide snow can be curative. Spider angioma is a telangie
ctasia of chronic liver disease. It appears as a central red spot with
radiating fine blood vessels like a spider.
4 SHORT NOTES AND SHORT CASES IN SURGERY

Cavernous haemangioma contains multiple venous channels and


grows in size with increasing age. It is bluish in colour, compressible
with a spongy feel. It commonly involves face, lip and tongue.
Plexiform haemangioma is the least common form and is a form of
congenital a-v fistula where aterial blood flows through dilated
venous channels. It feels like a bag of earthworms, is pulsatile
compressible, often with a systolic thrill and audible bruit.
Haemangioma are not only asthetically troublesome, but can
cause compression of adjacent tissue, can bleed, ulcerate, get
infected, thrombosed and calcified and even undergo sacromatous
changes.
Ligation of feeding vessel or its embolisation causes shrinkage
of the haemangioma. Haemangioma is radiosensitive but
complications like skin necrosis and haemorrhage may occur.

Glomangioma (Glomus tumor)


Glomangioma is a benign and circumscribed tumor arising from
the glomus body. Glomus body is the special arteriovenous
anastomosis surrounded by smooth muscle cells and glomus cells,
encompassed in a network of nerve fibers, most abundantly present
in nail bed. The glomus body regulates skin temperature.
Glomangioma is a bluish or red slowly growing circumscribed
swelling, rarely exceeding 1 cm. It causes burning/stabbing/
lancinating pain broughtout spontaneously or by pressure. It is to
be differentiated from, sub-ungal granuloma, subungal melanoma,
squamous papilloma, etc. Excision is the treatment.

Hamartoma
It is a developmental malformation containing disorganised tissues
belonging to the organ of origin. They grow until the physiological
growth continues. They are very common in lungs and liver.
Common example’s are haemangioma, neurofibroma, lymphan-
gioma, etc.

Neuroma
Neuromas may arise from primitive neuroblast (neuroblastoma)
or ceromaffin tissue of adrenal medulla (pheochromocytoma).
Adrenal neuroblastoma is a common malignant tumor of childhood
having tendency for early blood borne metastasis. Tumors of
SKIN AND SUBCUTANEOUS TISSUE 5

ganglion cells give rise to ganglioneuromas which are relatively


benign and occur at any age.

Neurofibroma
They arise from the supporting connective tissue of the nerve fibre
(the endoneurium) and can be
1. Localized (solitary neurofibroma)
2. Generealised (von Recklinghausen’s disease),
3. Plexiform.
Neurofibroma is smooth firm subcutaneous swelling with good
mobility across the direction of nerve fiber. It may often become
painful and tender. Because of pressure or strectching of the
concerned nerve, there may be tingling and numbness and often
muscle weakness. It may undergo cystic or sarcomatous changes
though very rarely. Besides subcutaneous tissue of upper limb and
abdomen, solitary neurofibromas may arise from dorsal root
ganglia and intracranial nerves. They should be distinguished from
lipoma, fibroma and haemangioma, etc. Compete excision is
curative but partial excision may encourage sarcomatous changes.
The nerve concerned may need limited resection with end to end
anastomosis.
In von Recklinghau sen’s disease there are multiple neurofibro-
mas involving peripheral nerves, cranial nerves (acoustic neuroma)
and spinal nerves (dumb bell neuroma). It is autosomal dominant
with male dominance. Multiple nodules of various sizes occur,
sessile or peduncul ated with soft to hard consistency. Cafe-au-lait
pigmentation, and kyphoscoliosis may be associated; sarcomatous
changes may occur. Since, involvement is extensive excision is only
undertaken when there is mechanical dyscomfort, neurological
abnormality or suspicion of sarcomatious changes.
In plexiform neurofibromatosis there is myxofibromatious
degeneration of the endoneurium of the affected nerve which
becomes enormously thickened. The overlying skin may become
thick, folded and pigmented often drawn into pendulous folds.
Mostly it involves branches of trigeminal but can also involve in
the limbs. On palpation it resembles tortuous thrombosed veins.
Elephantiasis neurofibromatosa is a serve form of plexiform
neurofibromatosis, resembling filarial elephantiasis but the
lymphatics are normal. Mulluscum fibrosum is another presentation
of neurofibromatosis with small multiple cutaneous nodules in
connection with terminal filaments of cutaneous nerves.
6 SHORT NOTES AND SHORT CASES IN SURGERY

Appearance of pain, sudden increase in size, fixity to surroun-


ding, increase in vascularity or paralysis of the affected nerve are
pointers to sacromatous changes.
Amputation neuroma is a painful growth of sevred nerve fiber
following limb amputation. Acoustic neuroma arises from
vestibular component of 8th cranial nerve in internal auditory
meatus causing deafness, vertigo, tinnitus and headache. Growth
into CP angle may cause dysfunction of facial, trigeminal and
abducent nerves.

Hypertrophied Scar
In hyperetrophied scar there is proliferation of mature fiborblasts
without concomitant proliferation of blood vessels. During healing
secondary infection or excessive tension prodispose to it. Scars
crossing the skin creases are particularly vulnerable. Such a scar is
glossy, raised above skin surface and nonitchy.

Keloid
In keloid there is proliferation of immature fibroblasts, collagen
fibrils and immature blood vessels. It follows surgical incisions,
pin prick, or burn wound. It is often familial. Negros are more
vulnreable and in them it may occur spontaneously. Due to
abundance of immature blood vessels the keloid (literally mean
claw like) may blanch, ooze, itch and may be bluish. It is raised,
lobulated and firm, with its claw like processes infringing upon
the surrounding healthy skin. Spontaneously occuring keloids on
chest may have a butterfly shape. Keloid may bleed, ulcerate or
may undergo malignant change (Marjolin’s ulcer).
Intralesional injection of hydrocortisons, vitamin A and hyaluro-
nidase may cause regression of keloid. Deep X-ray therapy also in
hibits proliferation of fibroblasts and immature blood vessels.
However excision with skin grafting may be required. In that case
preoperative radiotherapy to keloid site and postoperative radio-
therapy to both skin donor site as well as recipient site are required.
The later minimises the recurrence rate of keloid at donor and
recipient sites.

Epithelioma (Squamous Cell Carcinoma)


It is a malignant tumor arising from prickle cell layer of epidermis
and the squamous epithelium of mouth, tongue, pharynx, larynx,
SKIN AND SUBCUTANEOUS TISSUE 7

esophagus, vagina. Occasionally, it may arise from columnar cells


undergoing metaplasia in gallbladder, bronchus, cardiac end of
stomach. It is more malignant and rapidly growing in comparision
to basal cell carcinoma. It may arise de novo or in premalignant
lesions like senile (solar) keratosis, Bowen’s disease, leukoplakia,
lupus vulgaris, xeroderema pigmentosun, radiation dermatitis. Old
scars or keloids, chronic discharging sinus, venous ulcers may also
develop it (Marjolin’s ulcer). Kangri cancer of abdomen (in Kasmir)
and kang cancer of buttocks, heel and elbows (in Tibetans) are
nothing but squamous cell carcinomas due to heat. Prolonged expo-
sure to contact carcinogens (tar, dye) may also predispose to it.
It can be ulcereative or proliferative. To start with there is
thickening of skin or small nodule that breaks down to form an
ulcer that refuses to heal. The ulcer has rolled out everted edge,
indurated base fixed to deeper structures and unhealthy floor
covered with grayish white slough. Histopathologically there are
cell nests with central keratin surrounded by peripheral cells in
concentric manner giving on onion peel appearance. These cells
have hyperchromatic nuclei, loss of polarity, mitotic figures (signs
of malignancy). Well-formed cell nests indicate high degree of
differentiation and slow growth; but poor response to radiotherapy.
When cell nests are not well developed the cancer is rapid growing.
Ill differentiated but sensitive to radiotherapy. Cell nests may be
absent in esophageal cancer but occasionally may be present in
non-squemous cancer like teratoma of tastis and pleomorphic
adenoma of parotids.
Spread occurs by locally invasion, lymphatic permeation and
embolism but haematogenous spread is rare. Junctional legions of
skin and mulous membrane like lip, vagina and perianal regions
are the most vulnerable sites and all ulcerative lesions at these sites
be biopsied to exclude epithelioma.
Treatment is by radiotherapy or surgery. Small early lesions
without lymphnode metastasis or deeper invasion can be treated
by deep X-ray therapy, electron beam or radium / cesium implants.
Larger lesions with deeper invasion and lesions on slalp or near to
eyes need surgical excision with 1cm of safety margin. Mobile
infiltrated nodes can be dissected and removed but fixed hard
nodes need paliative radiotherapy.
Marjolin’s ulcer a low grade squamous carcinoma is painless, slow
growing and locally invasive. Lymphnode metstasis is absent and
it is radio resistant (both the features due to avascularity). It may
8 SHORT NOTES AND SHORT CASES IN SURGERY

develop over keloid, long-standing venous ulcer/radiation induced


ulcer and chronically discharging sinus. Excision with good margin
of safety is the treatment.

Basal Cell Carcinoma (Rodent Ulcer)


The tumor arises from basal cells of epidermis. It may also arise
from hair follicles, sweat and sebaceous glands. Face is the most
favored site, most lesions occuring above the line joining angle of
mouth to earlobule-usually near inner and outer canthi of eyes,
nose and nasolabial folds. Males in late middle age are the victims.
Sunlight exposure as in agricultural workers, fishermen is a
predisposing factor. The tumor is frequently multiple. Each lesion
may plaque like, nodule or ulcerative. The edge of the ulcer is
raised, beaded and rounded but not everted. The ulcer base is
indurated.
Histopathology shows closely packed islands of uniformly
packed polyhedral basophilic cells without mitotic figures. The
peripheral cells have palisade arrangement and are more deeply
staining. Spread is by direct infiltration; blood borne and lymphatic
metastasis are rare.
Treatment is radiotherapy for small superficial lesions and
surgical excision for large, deeply penetrating lesions or lesions
close to the eyes. Laser ablation, cryoablation and local trifluorouri-
dine are less dependable.
Basisquamous carcinoma is one where changes of squamous
carcinoma are seen at the margins of basal cell cancer. It is more
often seen in skin damaged by radiotherapy. Surgical excision is
curative.

Melanoma
Melanoma arises from melnanocytes or melanoblasts presentive
in basal layer of skin. Melanin is sulphur containing black pigment
synthesized from thyrosine in melanocytes of skin, choroid,
leptomeninges, substantia nigra, etc. Melanin protects against
sunlight. MSH, an anterior pituitary hormone controls synthesis
of melanin.
Melanoma may be benign (pigmented naevus) or malignant.
Naevus means localised cutaneous malformations including moles
and birth marks. Pigmented naevi may be present at birth but may
SKIN AND SUBCUTANEOUS TISSUE 9

also occur in childhood and later. The clinicopathological variety


of pigmented naevi include—
1. Junctional naevus
2. Intradermal naevus
3. Compound naevus
4. Blue naevus
5. Freckles
6. Juvenile melanoma.
Presence of proliferating melanocytes in the dermis characterises
intradermal naevus, their presence and proliferation at dermo-
epidermal junction characterizes junctional naevus. A compound
naevus has both dermal and junctional components. Junctional
naevus is smooth, flat, hair free light brown to dark brownish-black
lesion having maximum propensity to turn malignant. Intradermal
naevus is the most common type and is an elevated dome shaped
smooth warty hairy lesion, most common to face. Its malignancy
potential is nil. Blue naevus is a special type of intradermal naevus,
hair less flat, smooth and shinny lesion. It is commonly seen on
face dorsum of hand and feet, and buttock of baby (mongolian
spots). Compound naevus has junctional and intra-dermal
components. It is round elevated or eliplical lesion, light to dark
brown in colour. In adolescents it often grows, becomes
papillomatous and darker. Freckle is a premelanomatous
dermatosis that appears in face and neck of elderly persons. The
oral mucosa and conjunctiva can be involved. It has tendency for
enlargement and malignant transformation. It is worth rembering
that the congenital hairy mole has no malignancy potential;
malignancy may occur in giant congenital moles; pigmented naevi
appearing during puberty show junctional activity and can
undergo malignant transformation where as malignant change is
rare in the pigmented naevi appearing on face of middleaged.
Evidence of malignancy in a mole is:
1. Increasing size
2. Irregular edge
3. Increasing thickness
4. Irregular or increasing pigmentation
5. Appearance of pigmented halo or satelite nodules
6. Scaling, crusting, itching, ulceration
7. Loss of hair from a previous hairy lesion
8. Regional lymphadenopathy.
10 SHORT NOTES AND SHORT CASES IN SURGERY

Naevi need surgical excision if situated in potentially dangerous


areas like palm, sole, genitalia and mucous membrane. Similarly
the ones subjected to repeated trauma need excision. Any
suspicious naevus be completely excised with 5 cm of saftey and
be histopathologically studied.

Malignant Melanoma
Though rare (only 3% of all malignant skin tumors), it is most
malignant skin tumor with tendency for wide metastasis. Ninety
per cent of them arise in pre-existing naevus but 10 per cent arise
de novo. It can arise in any part of body but frequently in the lower
extremity of female and arm and chest of males. Fair skinned people
in 20-40 years are more vulnerable. Melanoma at sole of foot is
common in coloured races. It may occur in eyes, beneath the nail,
in the meninges and intestine.
Clinically malignant me lanoma can be of 4 types—leatigo-
maligna melanoma (LMM), supereficial spreading melanoma
(SSM), nodular melanoma (NM), and acral lentigenous melanoma
(ALM). LMM is least common and least malignant but SSM is most
common and less aggressive. SSM presents as a flat irregular
spreading pigmentation with variegated colour pattern of brown,
black and blue. NM is less common but more malignant; nearly all
mucosal, genital and oral melanomas are nodular. Vertical growth
characterizes NM; itching and ulceration are frequent. ALM is a
variety of SSM occuring in palm, sole and under the nail with worse
prognosis.
Clark’s level of skin invasion serves as a guide to risk of malig-
nancy. Similarly Breslow staging system in equally dependable:
Breslow staging
Stage 1 < 0.75 mm
Stage 2 0.76-1.5 mm
Stage 3 1.51-3 mm and
Stage 4 > 3 mm clarks
Clarke’s levels
Level 1 tumor confined to epideremis
Level 2 tumor invades basement membrane
Level 3 tumor extends to the interface between papillary
and reticular dermis
Level 4 tumor extends to reticular dermis,
Level 5 tumor extends into subcutanous fat. Level 1 has
low risk; level 2 and 3 (0.76-1.5 mm) intermediate
SKIN AND SUBCUTANEOUS TISSUE 11

risk with 25 per cent incidence of metastasis and


level 4 and 5 (1.51 and more deep lesions) high
risk with 60 per cent incidence of metastasis.
Spontaneous regression may occur in SSM and LMM and it may
be partial or complete. Amelanotic melanoma is one where the
melanocytes fail to synthesize melanin. Such tumors are difficult
to be distinguished from carcinoma and sarcoma. However, these
cells remain positive for dihydroxy phenylalanine. Metastasis is
usually on local extension and lymphatic permeation/embolism.
Blood borne metastasis occurs in advance disease to lungs, liver,
brain and bone.
Melanoma is radio resistant, hence excision is the treatment.
Tumors less than 0.75 mm thick are cured with excision since
lymphnode metastasis is rare. More thicker lesions need wide
excision with a safety margin 75 mm. The draining lymphnodes
need block dissection if enlarged or are positive for tumor cells.
Block dissection be avoided when tumor is more than 4mm thick
since long-term survival is rare. Malignant melanoma lesion or
suspected lymphnodes never be biopsied for risk of tumor spillage;
rather excisional biopsy be performed. Subungal malignant
melanoma is treated by disarticulation of the digit. Ocular
melanoma have no lymphatic metastasis but have haematogenous
spread.
Metastatic lesions are also radioresistant and relatively
chemoresistant. Endolymphatic therapy with 13I or 32P followed
by block dissection of lymphnodes is often effective. BCU and
vincry stine are often used in palliation. Radiotherapy is often
employed for cerebral and bone metastasis. Regional perfusion
with cytotoxic drugs (melphalen) is often effective in inoperable
limb melanomas.
Prognosis depends upon level of invasion, and tumor thickness.
5 years survival in low risk patients is 95-100 percent; intermediate
risk 85-95 percent and 15-5 percent in high-risk patients. Women
have overall 10 percent survival advantage.
Pigmented lesions of skin other than malignant melanoma
include seborrheic keratosis, pigmented basal cell carcinoma,
pyogenic granuloma, subungal haematoma, Kaposis sarcoma,
glomus tumor, histiocytoma, kerato-acanthoma, and mastocytoma.
2
Sinus and Fistula

Pilonidal Sinus
Pilonidal sinus is a subcutaneous track containing hairs or their
microscopic fragments. They are found most often in the natal cleft
overlying the coccyx. They may however occur in interdigital clefts.
the umbilicus, the axilla and face. They are thought to represent
foreign body granuloma, the foreign body being the hair that
penetrates the skin through abrasions, sweat glands or hair follicles.
The sinus is lined by stratified squamous epithelium and contains
dead hairs, granulation tissue, epithelial scales and debris but no
hair follicle and sebaceous gland. Jeep drivers and barbers have
pilonidal sinus in their digital webspaces.
In the natal cleft, the sinus may be represented by a skin dimple,
often with a tuft of hair projecting from its mouth. The snus may
have one or multiple openings with serosanguinous or purulent
discharge. Pain and tendernes at the tip of coccyx or repeated
abscess formation at that site needs probing for exclusions of
pilonidal sinus. Pilonidal sinus at this site be differentiated from
fistula in ano and sinus due to secondary infection of postanal
dermoid.
Treatment is by any of following methods
1. Excision and primary closure
2. Excision and healing by secondary intention
3. Marsupialization, Simple pure phenol injection into the sinus
tract followed by squeezing out the phenol, done 2-3 times
clears the tracts and enhances healing.

Thyroglossal Fistula
Though this fistula is mostly acquired, the persistence of thyro-
glossal tract is a congenital anomaly. The fistula occurs due to
SINUS AND FISTULA 13

bursting/ incision of thyroglossal cyst or its incomplete removal.


The tract is lined by squamous epithelium and discharges a serous
or mucoid fluid. The fistulous opening is in the midline of neck
usually below the hyoid bone. (hence called median fistula of the
neck). The fistulous opening moves up on protrusion of tongue. A
crescentic fold of skin with concavity downward overlies the
fistula’s opening. The fistula may close intermittently with
recurrence of inflammation and pain. Treatment is complete excision
of the fistulous tract as close as possible to the base of tongue. The
central portion of hyoid bone may have to be sacrificed.

Branchial Fistula
Branchial fistula may be congenital or acquired. The congenital
variety is most common and is due to either failure of fusion of
second branchial arch with 6th branchial arch or disappearance of
the membrane connecting these two arches. The acquired one is
due to bursting of the infected branchial cyst or inardvetent incision
over the infected cyst mistaking it for an abscess.
The fistulous tract may be complete or incomplete. The
incomplete form is most common. It does not communciate with
the cavity of the pharynx and remains blind at the inner end against
the lateral pharyngial wall-hence often termed branchial sinus. In
the complete form the internal opening is in intratonsilar cleft or in
the pyriform fossa. Traced from below the tract pierces the deep
fascia at the upper border of thyroid cartilage, then passes in the
fork of common carotid and internal carotid vessels and ascends
up deep to stylomandibular ligament and hypoglossal nerve but
superficial to glossopharyngial nerve and stylopharyageus muscle.
The external opening lies at the anterior border of sterno mastoid
at the junction of lower one-third and upper two-third. Branchial
fistula is often termed lateral fistula of neck. The fistulous tract is
lined by squamous or pseudo-stratified ciliated columnar epithe-
lium deep to which is a layer of lymphoid tissue. The discharge is
mucoid or mucopurulent.
The fistulous tract needs excision in toto after performing a
sinogram with radio-opaque dye. During excision the carotid
vessels and the 12th and 9th cranial nerevs are to be carefully
protected. During dissection one may encounter the cervical auricle,
a hood of skin containing yellow or elastic fibrocartilage in relation
14 SHORT NOTES AND SHORT CASES IN SURGERY

to external opening; or branchial cartilage embeded in the skin deep


to external opening.

Fistula in Ano
Most anorectal fistulas originate in the analcrypts at the anorectal
junction. Crypsitis leads to abscess formation and its rupture
producing fistula. The fistula may be subcutaneous, submucosal,
intramuscular or submuscular. It may be anterior, posterior, single,
complex or horse shoe shaped. Besides pyogenic infection; trauma,
diverticulitis, rectal cancer, Crohn’s disease, ulcerative colitis,
intestinal tuberenlosis, granuloma inguinale can present as perianal
fistula. Park has classified the anal fistulas as intersphincteric,
transphincteric and supralevator. The internal opening may be below
the anorectal ring or above it. According Salmon-Goodsal rule - when
the external opening is anterior to transverse plane, the internal
opening is usually in a crypt just opposite the external opening.
When the external opening is posterior to transverse plane, the
internal opening is in a crypt in the posteior mid line. When there
is anterior as well as posterior opening of same fistula, internal
opening is in posterior midline. Anorectal fistulas in infants are
congenital, anterior, straight and superficial. The external openings
may be multiple in Crohn’s disease, ulcerative colitis, tuberculous
proctitis, granuloma inguinale, bilharziasis and colloid carcinoma
of rectum.
The patient complains of intermittent or constant serous/
mucopurulent discharge or recurrent perianal abscess. In Crohn’s
disease and tuberculosis the discharge in watery. In rectal palpation
the internal opening may feel as a dimple or nodule surrounded
by area of induration. The cord like tract may be felt. The external
opening may present as small elevation with pouting granulation
tissue. The fistulous tract is lined by granulation tissue.
The fistulous tract can be probed for its path and depth. For
complex elusive fistulas fistulography is essential. Tuberculous and
fistula lacks induration and its external opening are multiple and
ragged with serous discharge. Perianal fistula should be differen-
tiated from pilonidal sinus, hidradenitis (inflammation of apocrine
sweat glands) and ruptured rectal dermoid.
Small acute fistulas may heal spontaneously but most fistulas
require unroofing the entire fistulous tract with excision of internal
SINUS AND FISTULA 15

opening to ensure that the healing occurs from within outwords.


Fecal incontinence may occur if anal sphincter is dissected. A two
stage operation is done when fistula passes deep to entire anorectal
ring. When deeper portions of sphincter and levators are left intact
incontinence is unlikely to occur.
3
Swellings in Neck

The swellings of the neck can be those situated on or near to midline


and those situated laterally. The midline cystic swellings include
ronula, dermoid, thyroglossal cyst, cold abscess in space of Burnes
and cyst in relation to isthmus of hyoid bone. Midline solid swellings
are those of enlarged lymphnodes and thyroid swellings. Lateral
neck swellings can be present in submandibular triangle, carotid
triangle or posterior triangle. Swellings in submandibular triangle
are—plunging ranula, submandibular gland cyst/tumor/
sialothiasis. Swellings in carotid triangle are branchial cyst, carotid
aneurysm, laryngocele, cold abscess carotid body tumor,
steronomastoid tumor, enlarged lateral, to be of hyroid. Swellings
in posterior triangle are cystic hygroma, lymphnode swelling/cold
abscess, solitary lymphatic cyst, pharyngeal pouch, subclavian
anurysm and cerlvical rib.
Swellings common to skin like sebaceous cyst, lipoma, fibroma,
neurofibroma and haemangioma can occur any where in neck.

Ranula
Ranula is a soft bluish mucous retention cyst arising from mucous
glands in floor of month and the under surface of tongue. (glands
of Blandin and Nuhn). The cystwall is lined by columnar or cuboidal
epithelium and the cyst contains viscid or jelly like fluid. Floctuation
and transillumination are positive. The globular swelling may vary
from 1-5 cm in diameter. It may have prolongation in the neck
passing along posterior border of mylohyoid muscle to appear in
submandibular triangle. This plunging ranula is best appreciated
by bidigital palpation. Patient may give history of bursting of ranula
with subsequent reformation.
SWELLINGS IN NECK 17

Ranula should be differentiated from


1. Sublingual dermoid, a midline swelling which is transillumi-
nation negative
2. Haemangioma which is compressible and transillumination
negative.
Complete excision is ideal but difficult. Partial excision with
marsupia lisation is the alternative. Plunging ranula is better
approached through incision in the neck.

Sublingual Dermoid
This is a form of congenital sequestration dermoid arising due to
inclusion of the surface ectoderm. It usually appears between
10-25 years of age. The cyst lining is by sqenmous epithelium,
containing cheesy material but never hair. Sublingual dermoid can
be median and lateral in position, each being suprahyoid or
infrahyoid in position. The median infrahyoid dermoid appears as
a midline cystic swelling below symphysis menti-giving double
chin appearance. It may be confused with thyroglossal cyst, lymph-
node or ectopic thyroid. The lateral infrahyoid variety appears as a
cystic swelling in submandibular region to be confused with
plunging ranula, enlarged sub-mandibular gland or lymphnode.
Complete removal is curative.

Thyroglossal Cyst
The thyroglossal duct extends from foramen cecum to isthmus of
thyroid gland passing through genioglossus and then either infront,
behind or through the hyoid bone. The duct completely atrophies
in postnatal life except for the lower part that forms pyramid and
isthmns of thyroid gland. Ectopic thyroid tissue may be present
along the course of the duct. When present in the region of tongue,
it is called lingual thyroid. Often it is the only thyroid tissue present.
When a portion of duct remains unobliterated, it gives rise to thyro-
glossal cyst presenting in the floor of mouth, supra and subhyoid
region, infront of thyroid or cricoid cartilage.
The cyst wall is lined by columnar, cuboidal or squamous
epithelium surrounded by a layer of lymphoid tissue. The cyst
content is a transparent thick jelly like fluid often containing
cholesterol crystals. The cyst is seen in children, commonly girls as
a midline cystic swelling of neck moving up with deglutition and
tongue protrusion. The cyst may be confused with dermoid,
18 SHORT NOTES AND SHORT CASES IN SURGERY

subhyoid bursal cyst. Thyroid swellings, subthyroid bursal cyst,


laryngocele and enlarge pretracheal glands also move with
deglutition but not with protrusion of tongue.
Subhyoid bursal cyst is a midline transversely elongated discoid
shaped cystic swelling below hyoid containing clear or turbid fluid.
It is positive for transillumnation and is tender unlike thyroglossal
cyst.
Thyroglossal cyst is vulnerable for infection and formation of
thyroglossal fistula and risk of malignancy. Hence excision is
mandatory.

Branchial Cyst
It is a cystic swelling arising from persistent cervical sinus which is
formed on fusion of overgrowing second branchial arch with sixth
branchial arch. The cyst wall is lined by squamous epithelium and
contains a cheesy material rich in cholesterol coystals. Though
congenital the cyst appears by 20-25 years of age as a painless lump
in upper lateral part of the neck. It is ovoid in shape lying beneath
the middle of sternomastoid muscle and bulging forward along its
anterior border into the carotid triangle. It is transillumiration
negative but positive for floctuation with limited mobility.
The cyst is to be differentiated from other swellings in this region
like cervical dermoid, plunging ranula, cystic hygroma, carotid
body tumor, or enlarged lymphnode. Recurrent infection and fistula
formation are common. Hence, complete removal is necessary with
due care in not injuring the carotid arteries and IX and XII cranial
nerves.

Cystic Hygroma
A cystic hygroma is a collection of lymphatic sacs containing clear
colourless lymph. It is a congenital malformation affecting
lymphatic channels and is regarded as a hamartoma. It may be
found at birth, often obstructing labor or appears within first few
years of life. The swelling is moltilobular and multiloculated, the
larger cyst located peripherally and the smaller ones centrally. Each
cyst is lined by a single layer of columar epithelium. The cyst fluid
does not coagulate. The most common site for cystic hygroma is
the root of neck in posterior triangle. Other sites include axilla,
inguinal region, meliastinum and oral cavity. The cyst when present
SWELLINGS IN NECK 19

in root of neck becomes very prominent when the child cries.


Transillumination is brillianthy positive so also floctuation. When
present in mediastinum, it can press upon esophagus, bronchus
and great vessels. The cyst may get infected to become an abscess.
It should be differentiated from branchial cyst, cold abscess and
solitary lymph cyst that usually develops in supraclavicular region
in adults.
Though spontaneous regression is known sudden increase in
size can cause respiratory compromise. Complete excision of the
cyst can be undertaken and the finger like projections from the cyst
wall invading surrounding structures be carefully dissected.
Consrevative treatment includes aspiration followed by injection
of sclerosants but infection remains a danger.
The cysts that contain cholesterol crystal are:
1. Branchial cyst
2. Cystic hygroma
3. Thyroglossal cyst
4. Dental cyst
5. Dentigerous cyst
6. Old hydrocele (some times).

Pharyngeal Pouch
Pharyngeal pouch is a pulsion or pressure diverticulum of pharynx.
The pharyageal mucosa herniates through the Killian’s Dehiscence,
the potential area of weakness between the upper oblique fibers
and lower horizontal fibers of the inferior constrictor. The oblique
fibers are propulsive in function, is supplied by accessory nerve
and the horizontal fibers are constrictive in nature. When the oblique
fibers contract and propel the food, the horizontal fibers relax and
open the esophagus. Simultaneous contraction of both the fibers,
raises pressure within pharynx to cause pulson diverticulum.
When the diverticulum is small patient complains of only foreign
body sensation in throat but once it becomes larger, there is
regurgitation, sense of suffocation and mild dysphagia. Larger
pouch may cause dysphagia, visible swelling behind sternmastoid
and features of repeated aspiration (lung abscess pneumonitis). The
soft swelling is deep with smooth surface, impalpable edges and
can often be partially reduced. Barium study is diagnostic. Excision
of the pouch followed by cricopharyngcal myotomy is curative.
20 SHORT NOTES AND SHORT CASES IN SURGERY

Laryngocele
Herniation of the laryngeal mucous membrane through thyrohyoid
membrane produces larynglcele. It is mostly acquired and unilateral
found mostly in professional trumpet blowers and parients of
chronic cough. When the sac enlarges it gives rise to a visible
swelling in the neck over the thyroid cartilage. Valsalva maneuver
makes the swelling more prominent. It has boggy feel and is
resonant to percussion. The swelling moves up during swallowing.
Hoarseness may be present. Complete excision of the sac is curative.

Carotidbody Tumor
Otherwise known as chemodectoma (potato tumor) it arises from
chemoreceptor cells of carotid body situated at the bifurcation of
common carotid artery. Such chemoreceptor organs are also present
as:
1. Aortic bodies in ascending aorta
2. Glomus jugulare in bulb of internal jugular vein
3. In association with 9th and 10th cranial nerves.
These organs are sensitive to changes in pH and O2-CO2 tension
in blood.
Chemodectoma is regarded as non-chromaffin paraganglioma
akin to glomus tumor of skin and neuroblastoma of adrenals. It is
locally malignant with regional lymphnode metastasis in 20% cases.
The capsule is well formed and cutsurface has dense fibrous septa.
Consistency may be hard to spongy. It usually presents as a slowly
growing painless lump in upper anterolateral neck in people of 40-
60 years. Compression of internal carotid causes fainting attacks.
Though initially spherical, it becomes irregular as it grows. It is
movable from side to side. It should be differentiated from sterno-
mastoid tumor and enlarged lymphnode. It is radioresistant hence
surgical removal is warranted. Often complete removal entails
resection of a portion of carotid vessels followed by venous graft.

Sterenomastoid Tumor
Commonly seen in newborn it appears as a hard fusiform tender
swelling in the middle of sterenomastoid muscle. With time the
swelling disappears but the neck tilts to the side of the swelling
with turning of chin to opposite side. The cause of the swelling is
SWELLINGS IN NECK 21

elusive and include ischaemia, trauma, venous thrombosis, etc.


Unless the deformity in corrected early there is thickening and
shortening of deep cervical fascia, the scaleni muscles and the
vessels with secondary scoliosis of thoracic vertebra. Cervical
hemivertebra in often associated.
Manipulation of neck daily once the tumor is discovered before
contracture has developed can prevent torticollis. When torticollis
has already developed subcutaneous tenotomy at the lower end
(sterenal head), open division of both the heads at lower end or
open division of upper end of sterenomastoid can correct the
deformity. However the internal jugular vein, phrenic nerve and
accessory spinal nerves must be carefully dissected. After surgery
correction must be maintained by wearing the torticollis harness
for 6 months with active and passive physiotherapy.

Cold Abscess
Cold abscess may present as a soft floctuant swelling in posterior
triangle, when it is due to caries spine. In softening of tuberculous
lymphnodes, the swelling is in upper half of anterior triangle.
Tubercular lymphadenitis evolves through 3 phases-discrete tender
nodes, matted nodes due to periadenitis and finally softening and
liquefaction forming cold abscess that may burst forming a
tubercular sinus. In caries of cervical spine the pus first lies deep to
prevertebral fascia and then may travel
1. Behind carotid sheath to reach posterior triangle
2. Downwards to mediastinum
3. Forwards to form retrophanyngeal abscess.
From posterior triangle the pus can travel into axilla along the
axillary sheath which is the prolongation of prevertebral fascia and
then along the course of brachial artery. The swelling is painless
and there in no local signs of inflammation. If there is caries of
spine-neck movements may be restricted and painful.
The cold abscess be distinguished from other cystic swellings
like branchial cyst and cystic hygroma. Antituberculous therapy
helps in resolution of small abscess and matted lymphnodes.
Lymphnodes defying resolution with drugs can be resected enblock.
Larger abscess may need aspiration at the higher-level through
healthy skin to prevent sinus formation. Associated cervical spine
disease needs immobilisation.
22 SHORT NOTES AND SHORT CASES IN SURGERY

Collar stud abscess is a bilocular abscess with an intervening


narrow channel of communication. It can be pyogenic or tubercular.
Pyogenic abscess is commonly found on palm lying superficial and
deep to palmar fascia. Tuberculous form is usually found in neck
due to caseation necrosis of tubercular lymphnode.
Degeneration of nodular leprosy, and gumma and actinomycosis
also can present as cold abscess.

Cervical Adenopathy
Lymphnodes in neck are arranged in two groups-superficial and
deep, the latter lying deep to deep cervical fascia and divided into
vertical and circular groups. The vertical group comprises the
1. Jugulo diagastric nodes
2. Juguloomohyoid nodes and
3. Supraclavicalar nodes (lying between two heads of sterno-
mastoid). Tonsils drain to jugulodigastric nodes and tongue to
jugulo-omohyoid nodes. Left supraclavicular nodes are enlarged
in metastasis from gastric cancer, testicular tumor, esophageal
cancer and right supraclavicular nodes in bronchial malignancy.
The circular group consists of the submental, submandibular,
preauricular, postauricular and occepital nodes.

Causes
Acute—pyogenic, acute leukemia, infectious mononucleosis
Chronic—
a. infections—tuberculosis, syphilis (secondary), sarcoidosis,
brucellosis
b. maligant—lymphoma (HL, NHL), CLL, mmtastasis
c. autoimmsune— `SLE, juvenile rheumatoid arthritis
Enlargement of cervical lymphnodes is by far the most common
cause of neck swelling. In pyogenic lymphadenitis the lymph nodes
are firm, tender but in tuberculosis they are firm non-tender and
often matted. In Hodgkin’s disease they are descrete and rubery.
Metastatic lymphnodes are hard pain less and fixed. In syphilis
the lymphnodes are discrete shooty and non-tender.
In dealing with a case of cervical lymphadenopathy one must
examine the oral covity, laxynx, lungs, abdomen and testes besides
a thorough general examination. However a lymphnode FNAC or
excision biopsy is most diagnostic. In children FNAC/biopsy most
SWELLINGS IN NECK 23

often shows reactionary hyperplasia, a non-pecific finding


commonly associated with infection of oral cavity.

Parotid Swelling
Parotid swellings obliterate the furrow behind the ramus of
mandible and often push up the lobule of the ear, if large. While
examining parotid swelling, its relation to masseter (relaxed and
taught), facial nerve function, movement of temperomandibular
joint, status of lymphnodes in neck and parotid duct orifice in the
mouth must be examined and the parotid duct be palpated.
Neuroma of facial nerve, adamantinoma of mandible, swelling of
masseter muscle (rhabdomyoma) preauricular lymphnodes can be
confused with parotid swelling. Preauricular lymphnodes are
ovoid, firm, smooth and freely movable swelling infront of tragus.
Seventyfive percent of neoplasms of salivary gland occur in
parotid and eighty percent of parotid tumors are benign and most
of them (80%) are pleomorphic adenomas. The remaining 20 percent
are malignant. 15 percent of salivary tumors occur in submandi-
bular gland and 60 percent of them are benign (pleomorphic
adenoma in 90 percent). Ten percent of salivary tumors occur in
sublingual and other minor salivary glands of palate, cheek and
lips. Forty percent of them are benign; exclusively pleomorphic
adenoma.
Parotid tumors can be epithelial and non-epithelial (haeman-
gioma, lymphangioma). Epithelial tumors are of 4 types-adenomas
(monormorphic or pleomorphic), mucoepidermoid tumors, acinic
cell tumors and carcinomas. All parotid tumors are radioresistant
and deny any biopsy evaluation for fear of facial nerve damage.

Adenolymphoma (Warthin’s Tumor)


It is the monomorphic benign epithelial tumor arising from
periparotid lymphoid tissue. It is a cystic tumor often, multiple or
bilateral, situated outside the parotid capsule or just beneath the
parotid capsule. Age of occurence is fourth decade and above with
male preponderance. The swelling is oval with smooth surface and
is freely mobile without involvement of facial nerve. It can be
confused with pleomorphic adenoma but in 99mTC pertechnate scan
it appears as a “hot spot” unlike pleomorphic adenoma that gives
“cold spot”. Superficial parotidectomy is curative.
24 SHORT NOTES AND SHORT CASES IN SURGERY

Pleomorphic Adenoma
It is a potentially malignant tumor with pleomorphic stroma
containing lymphoid, myxomatous, fibrous, pseudocartilaginous
and epithelial elements. It is a slow growing painless rounded
swelling with firm elastic to rubbery hard consistency and free
mobility. When malignancy ensues, facial palsy and fixity of tumor
to masseter may be seen. Patey’s operation, i.e. superficial
parotidectomy with preservation of facial nerve is the answer.
Sudden increase in size, nodularity of surface, fixity of skin over
the tumor, restricted jaw movements and prominence of veins over
the swelling and hard enlarged cervical nodes are pointers towards
malignant transformation.

Carcinoma of Parotid
The carcinoma may start de novo or is superimposed on pleomorphic
adenoma. It is a disease of elderly and lymphnodes are involved in
15 percent cases. Treatment is with total parotidectomy with
preservation of facial nerve or radical parotidectomy with sacrifice
of facial nerve and enblock dissection of cervical lymphnodes of
that side. Inoperable tumors need palliative chemoradiotherapy.
Partial removal of tumor with implantation of radium needles can
also be tried. Local recurrence and in operable cervical nodes need
radiotherapy. Perfusion of cyclophosphamide through superficial
temporal artery can bring about regression of tumor.

Parotid Fistula
Parotid fistula may be external or internal. It may be a gland fistula
or duct fistula. Causes include rupture of parotid abscess, injury to
parotid or following superficial parotidectomy. When the external
fistula is connected to the major duct, there is outpouring of parotid
secretions during eating, swelling at thought of food with
excoriations of the skin. A sialogram with lipiodol can detect if the
fistula is in relation to main duct or a ductule. When connected
with main duct reconstruction is required failing which resection
of auriculotemporal nerve (secretomotor fibers) or complete paro-
tidectomy with preservation of facial nerve may be planned. Fistula
connected with a ductule may need excision or parasy mupatho-
lytics like probanthine 50 mg every 6 hours.
SWELLINGS IN NECK 25

Auriculotemporal Syndrome (Frey’s Syndrome)


It follows injury to auriculo temporal nerve during parotid surgery.
In it, the cheek becomes red, hot and painful during eating. There is
cutaneous hyperesthesia infront and above the ear during shaving.
It is likely that the secretomotor fibers overgrow into the cutaneous
element of the nereve due to aberrant regeneration. Though the
symptoms pass off spontaneously in few years avulsion of auriculo
temporal nerve infront of auricle abolishes the symptoms.

Sialolithiasis
Stones are 50 times more common in submandibular gland and its
duct (Wharton’s duct) than in the parotid or its duct (Stensen’s
duct). This is because parotid duct has a straight course, dependent
drainage and parotid secretion is less viscid. Wharton’s duct has a
long curved and upward course and is hooked by lringual nerve
and the submandibular secretion is more viscid.
The radio-opaque calculus is mainly composed of phosphate
and carbonate of calcium and magnesium. The patient complains
of pain and swelling in submandibular region aggravated by food,
classically by sucking a lemon. This salivery colic is often interpreted
as toothache. Impacted stone in the duct may irritate the lingual
nerve with pain in the tongue. The enlarged submandibular gland
has firm or rubbery hard consistency with palpable anterior,
posterior and inferior margins an almond or ovoid shape best
appreciated by bidigital palpation. During examination of floor of
mouth, the swollen edematous and congested orifice can often be
seen when stone is impacted in the duct. In such patients, instillation
of lemon juice to mouth will fail to show salivary secretion coming
out from concerned submandibular duct orifice. X-ray usually
demonstrates. The submandibular calculus.
When the stone is in the duct, incision can be made directly on
the long axis of duct at the site of stone, the stone is removed and
the wound left unsutured. When the stone is within substance of
the gland, excision of the gland is undertaken.
Minor salivary glands are present in hard palate, lips and cheeks,
tongue, pharynx, larynx, etc. Ectopic salivary gland may be present
in eyelid, lacrimal gland, PNS, middle ear, nose, etc. Salivary tumors
at these sites are asymptomatic usually seen in children below 5
years of age, often recur after excision, hence need adjuvant
radiotherapy.
26 SHORT NOTES AND SHORT CASES IN SURGERY

Sjogren’s Syndrome
It is an autoinmune disorder with destruction of salivary tissue
and its replacement with lymptoid tissue. It is commonly seen in
female above 40 years of age, is bilateral, symmetrical and painless
with dryness of mouth and eyes. Systemic manifestations include
polyarthritis, keratoconjunctivitis, scleroderma and polyarteritis
nodosa. X-ray may show small areas of calafication, commonly the
parotids. Treatment is symptomatic with sialogogues. Mikulicz
desease is a variant of Sjogren’s disease where salivary and lacrimal
glands are enlarged but without any systemic manifestations.
Steroids and X-ray therapy may be beneficial. Surgery is reserved
for massive deforming glandular enlargement.

Jaw Swellings
Swellings of jaw may arise from bone, tooth germs, and mucoperi-
osteum. Those arising from bone include osteoma and fibrous
dysplasia, carcinoma and sarcoma; those arising from tooth germs
(odontomes) include adamantinoma, dental and dentigerous cysts;
and those arising from micoperiosteum are the epulis which can
be granulomatus, fibrous, myeloid, sacromatous and carcinoma-
tions.

Epulis
Fibrous epulis is the most common. Chronic dental sepsis is
considered an important predisposing factor. It may be sessile or
pedunculated. Lower jaw in young women, particularly around
neck of canine or premolar is the most frequent site. The overlying
mucous membrane is healthy but bright pink or grey. The adjacent
teeth are irregularly placed and loosened and the tumor may
protrude in between the irregular teeth. Excision is rewarding but
there is tendency for recurrence. Grannlomatous epulis forms around
carious tooth or at the site of chronic irritation by the denture. It is
a soft rapidly growing bright red growth that bleeds on touch with
offensive smell and enlarged draining lymphnodes. Antibiotics,
scraping of granulation tissue and extraction of carious tooth,
change of ill fitting denture bring about relief. Myeloid epulis is
pathologically an osteoclastoma arising from mucoperiosteum of
the gum common to young. It is rapidly growing smooth lobulated
soft mass, plum coloured that bleeds and ulcerates. Egg shell
SWELLINGS IN NECK 27

crackling may be appreciated. X-ray shows soap bubble or honey


comb appearance. Since locally malignant, thorough curretage with
filling of the cavity with bone chips or radical excision of the
swelling with bone grafting may be undertaken. Carcinomatous
epulis is an epithelioma arising from gum mucosa. It may appear
as a fungating ulcerative growth. Biopsy is diagnostic and surgery
is the treatment.

Adamantinoma
Adamantinoma arises from ameloblasts, the primitive enamel
forming cells. It is a painless slow growing, benign but locally
malignant solid tumor arising around molar teeth. With growth it
undergoes multicentric cystic degeneration causing expansion of
the jaw, usually the outer table of mandible. It should not be
confused with osteoclastoma and giant cell reparative granuloma
of jaw. Age of occurence is between 10-35 years and females are
commonly affected. The tumor is slow growing, painless and often
soft and floctuant due to cystic degeneration. Generally there is no
missing teeth. The tumor may attain large size causing ugly facial
deformity, malalignment and malocculsion of neighboring teeth.
X-ray shows soap bubble or honey comb appearance. Treatment is
evacuation and curettage or excision with bone grafting. Besides
jaw, adamantinoma can arise in pituitary stalk and tibia.

Dental Cyst
It is an unilocular cyst arising in connection with the root of a
normally eurrupted but chronically infected and usually pulpless
tooth. The lining of the cyst wall in with sqamous epithelium and
the cyst contains cholesterol crystals and giant cells, often expanding
the alveolus. The upper incisor or canine are the frequent sites.
X-ray shows an oval or round radiolucent area in relatioan to root
of the tooth. Treatment is complete excision of the cyst wall along
with the diseased tooth and obliteration of the cavity by soft tissue.

Dentigerous Cyst
It is a unilocular cyst arising in relation to non-erupted permanent
tooth. The cyst arises from follicle of the developing tooth. The lining
of the cyst wall is usually fibrous. The unerrupted tooth is embeded
within the cyst, commonly the upper or lower third molar with
truncated roots. The cyst content is a viscid fluid with cholesterol
28 SHORT NOTES AND SHORT CASES IN SURGERY

crystals and giant cells. There is an expansion of the outer table of


the jaw even causing egg shell crackling. X-ray shows the radio-
lucent area with crown of the tooth inside it. There may be
pseudotrabeculation or soap bubble appearance. Treatment is
excision of cyst wall with obliteration of the cavity by soft tissue.

Giant Celled Reparative Granuloma


It is a condition arising due to occurence of haemorrhage within
the bone marrow of jaw bones, commonly mandible. The lesion
contains opaque semisolid dark red material. The swelling is
painless and common to females. X-ray shows round or oval
translucency with expansion and thinning of cortex to be confused
with adamantinoma, osteoclastoma and brown tumor of hyper
parathyroidism. Curettage of the cavity is curative.

Actinomycosis of Jaw
Actionmycosis is a chromic suppurative granuloma occuring
particularly in region of lower jaw, caused by Actinomyces israelli
an anaerobic gram-positive organism, whose colonies appear as
sulphur granules. Actinomycosis is an endogenous infection
gaining access to tissues through defects in mucous membrane by
ulceration or trauma. Clinical forms of actinomycosis are
cervicofacial (65%), ileocaecal-(25%) and pulmonary (10%).
Infection spreads by continuity and contiguity. Infection evokes
granulomatous and fibrous tissue reaction with multiple dischar-
ging sinuses. Lymphatic spread does not occur and regional
lymphadenitis if any is due to secondary infection. Spread through
blood may occur to liver via portalvein from ileocaecal lesion. From
cervicofcaial lesion and pulmonary disease systemic dissemination
may occur to other organs.
In cervico facial from lower jaw is commonly involved with
formation of brawny hard indurated painless swelling with
multiple sinuses discharging sulphur granules. X-ray may show
multiple sclerosed bony cavities with sequestra. Penicillin 4
megaunits daily for 3 months is curative. Tetracyclim is the
alternative. Incision and drainage (not excision) and in resistant
cases radiotherapy may be tried.
Madura foot is caused by nocardia, an aerobic, gram-positive,
often acid fast organism usually found in soil. It is common to
tropical countries and is particularly endemic to South India,
SWELLINGS IN NECK 29

particularly in farmers who walk barefooted. In early disease firm


nodules surrounded by vesicles appear. The nodules liquefy
discharging pus through multiple sinuses. The discharging granules
may be black, yellow or red. The disease or invades deeper tissues
but lymphatic and blood borne metastasis is rare.
Treatment is incision and drainage under cover of long-term
penicillin and dapsone but many cases may require amputation.
4
Lesions of Oral Cavity

Cystic lesions of oral cavity include ranula, dermoid and mucous


cysts. Mucous cysts are retention cysts, pale pink in colour with
smooth surface. Repeated trauma leads to their rupure but
recurrence is common. Hence, excision under local anaesthesia or
cryosurgroy be preferred. When multiple, injections of boiliog water
can cause sclerosis and elimination of the cysts.

Leukoplakia
Leukoplakia literally means a white patch on squamous epithelium.
It is common in mouth, larynx, glans penis and vulva. Predisposing
factors include smoking, sepsis, spices, syphilis, ill fitting dentures
etc. Microscopically there is hyperkeratosis, vacuolation of cells in
middle layer with hyperplasia of basal layers. Leukoplakia is
premalignant and neoplastic changes occur in 25 per cent cases.
Leukoplakia has a gray white appearance with cracks and fissures
on its surface. The lesion is difficult to rub off. Neoplastic change
usually occur within the fissures. Thickening, reddning, bleeding
and pain indicate chances of malignant transformation. Leukoplakia
evolves is 4 stages—
• Stage I—gray transparent thin patch.
• Stage II—white opaque patch with cracks and fissueres.
• Stage III—warty excrescences and nodules with areas of
desquamation giving red, smooth, shinny appearance.
• Stage IV—carcinomatous changes. Biopsy is essential and
excision is curative.

Ulcers on Tongue
Ulcers on tongue can be aphthous ulcers, traumatic ulcers, infective
ulcers (HSV), tubercular, syphilitic and malignant ulcers. Traumatic
LESIONS OF ORAL CAVITY 31

ulcers are common on the margin commonly posteriorly, where as


malignant ulcers are also at the margin but in enterior two-third.
Tuberculous ulcers can occur at any site-tip, margin, dorsum.
Traumatic, aphthous and tubercular ulcers are painful, while others
are painless. Aphthous and tuberculous ulcers are often multiple.
Syphilitic ulcer is common on dorsum of tongue, is painless with
punched out margin and its floor is covered by washed leather
slough. Induration of base of ulcer indicates malignancy so also
rolled out and everted edges.

Carcinoma of Tongue
Squamous cell cancinoma of tongue accounts for more than half of
intraoral carcinomas. Predisposing factors include smoking, alcohol,
oral sepsis, spices leukoplakia, submucous fibrosis, syphilis, Plumer
Vinson syndrome, etc. Eighty percent of tongue cancers occur in
anterior two-third and 20 percent in posterior third. Out of the
anteriorly occuring cancers-10 per cent our on tip, 50 percent on
margins, 10 percent ventral surface, 10 percent on dorsal surface.
The growth can be ulcereative, warty/papillomatious, plaque
nodular, crack/fissure with induration. Posterior third carcinoma
is usually of following types-lymphoepithelioma, basal cell
carcinoma or transitional cell carcinoma. Spread is by local
extension, lymphatic and rarely haematogenous. Lymphatic spread
from tip is to submental nodes, from margins to submandibular
nodes and posterior third to jugulo-digastric nodes. From submental
and submandibular nodes it spreads to jugulo-omohyoid nodes.
Carcinoma when situated in extreme posterior prosition of tongue
may have haematogenous spread.
Any patient having chronic painless ulcer in mouth, leukoplakia
or submucous fibrosis need biopsy followup. Appearance of pain,
bleeding, fixation of tongue, dysphagia, voice changes, local
induration or hard fixed lymphnodes in neck are pointers towards
malignancy.
Treatment is either with radiotherapy or surgery. Anterior two-
third cancer less than 1 cm be excised with good margin of safety.
When more than 1 cm, radiotherapy be third first failing which
partial, subtotal or hemiglossectomy be performed. When growth
reaches within 2 cm of mandible, henimandibulectomy may be
advised. Cancer of the posterior third of tongue irrespective of size
32 SHORT NOTES AND SHORT CASES IN SURGERY

be treated with teletherapy. Mobile hard lympnnodes need block


dissection of neck but hard fixed nodes are treated by radiation.
Interstitial irradiation by radium needles kept for 7-10 days deliver
between 6000-8000 r and suffice for anterior large lesions.
Prognosis is dependent on site and stage of cancer and
involvement of lymphnodes. Early anterior lesions have 5 year
survival of 50-60 per cent but in late stages this reduces to 15 per
cent. If nodes are involved or lesion is in posterior third 5 years
survival is only 15 per cent.

Carcinoma Lip
Upto 93 per cent of lip carcinomas occur in lower lip, 2 per cent
occuring at angles of mouth and 5 per cent in upper lip. Site of
lesion may be at the junction of skin and mucous membrane, in the
exposed mucous membrane or in the mucous membrane on the
inner surface of the lip. Predisposing factors include old age, tobacco
cheeing, papilloma, leukoplakia and fissure in angle of mouth.
Carcinoma of lip may present as a nodule or fissure, warty growth,
an infiltrative induration or typical malignant squamous carcinoma.
Spread is by local extension or often to draining lymphnodes but
very rarely by blood stream. Carcinoma of lip be differentiated from
simple papilliferous wart, syphilitic chancre and keratoacanthoma.
In early cases without lymph node in volvement treatment of choice
is radiotherapy. Surgery is indicated in advanced disease involving
lymphnodes, jawbones or in cancer superimposed on leukoplakia
and syphilitic sore. Palpable but mobile nodes if positive for
metastasis need block dissection but once fixed, palliation with
radiotherapy is the answer.
5
Varicose Veins

Elongated, dilated and tortuous veins are termed as varicose veins.


Varicosity is common in the leg veins, pampiniform plexus and at
the sites of portosystemic anastomosis as in portal hypertension.

Saphenous Varicosity
The venous system of leg consists of the superficial veins, the deep
veins and the perforating veins. The long and short saphenous veins
with their tributaries belong to the superficial group, placed
between the skin and deep fascia. They contain numerous valves.
Their middle coat is much thicker consisting mostly of smooth
muscle. These veins are poorly supported and are at low pressure.
Normally the blood from superficial system flows to deep system
of veins except in sole (and palm) where blood from deep system
flows into superficial system.
Veins of deep saphenous system accompany the main arteries
and are well supported by calf and thigh muscles. They comprise
of femoral and popliteal veins, veins accompanying anterior tibial,
posterior tibial and peroneal arteries and the valveless soleal
venous system. These deep veins are also provided with valves.
The perforators communicate between the superficial and deep
veins piercing through the deep fascia. They have valves only at
their origin and at their entrance to to deep veins. They normally
allow blood to pass from superficial to deep veins and comprise
of:
1. Three medial perforators, situated along posterior border of
tibia 2", 4" and 6" above medial malleolus
2. Three lateral perforators situated 2", 5" and 7" above lateral
malleolus along posterior border of fibula
3. Central perforators, one or two is number connecting the short
saphenous system to veins of soleus and gastrocnemius
muscles.
34 SHORT NOTES AND SHORT CASES IN SURGERY

During rest the pressure driving blood to heart from the venous
system is the systemic venous pressure transmitted from arterial
tree. During walking this is further augmented by the calf pump.
Varicose veins develop when blood leaks from high pressure deep
venous system to the poorly supported superficial saphenous
system due to incompetency of valves in perforators or weakened
valves in superficial saphenous system that fail to direct flow of
blood upwards against gravity.

Aetiology
Exact cause of varicosity is not definitely known but factors like
ageing, obesity, prolonged standing and vigorous muscular efforts
(as in athletes) can predispose to varicosity. In most cases however
congenital incompetence or weakness of the valves is incriminated.
Pregnancy predisposes to varicosity as the gravid uterus presses
upon the vena caval system obstructing venous flow or the high
level of progesterone and relaxin excessively relax the venous
system weakening the valvlar mechanism. Damage to valves by
thrombophlebitis or pressure from pelvic/abdominal tumor are
responsible in some cases. Varicosity in young should prompt for
diseaseses like congenital/traumatic a-v fistula and extensive
cavernous haemangioma.

Symptoms and Signs


Though many patients of varicose vein are asymptomatic, aching
pain on prolonged standing or at the end of the work, night cramps
on retiring to bed are often complained of. While in arterial
insufficiency the cramp and weakness increase on walking, they
are relieved with walking in patients of varicosity. Mild ankle
edema, pigmentation of skin around ankle often with dry scaly
eczematous lesion may be found.
Signs of varicosity are the evident dilated tortuous veins, and
the presence of saphena varix, a saccular dilatation of upper end of
long saphenous vein at the saphenous opening. It has a cough
impulse and the swelling disappears when the patient lies down.
Saphena varix may often be confused with femoral hernia. The
varix however has a fluid thrill which can be felt over the swelling
when the long saphenous vein down the thigh is tapped.
VARICOSE VEINS 35

Tests of Varicosity
The tests are meant for diagnosis as well as to plan the appropriate
treatment. They indicate as to which venous system is involved,
which perforators are incompetent and the patency status of deep
veins. Long saphenous system varicosity is evident from visible
dilated tortuous veins infront of medial mallelolus ascending upto
saphenous opening. Short saphenous varicosity shows up as
dilated and tortuous veins at the back of the leg converging on
popliteal fossa.
Schwastz test is the perception of fluid thrill at saphenous
opening. In cough impulse test a fluid thrill is palpable so also a
bruit on auscultation below the saphenous opening when the
patient coughs. Trendelenberg test is to detect incompetency of
saphenofemoral junction. In this test the lying patient elevates the
affected leg to empty the varicose veins; then the thumb is placed
firmly on the saphenous opening and patient is asked to stand up.
If the veins gradually fills from below incompetency of perforators
is certain. Filling of vein from above downwords on release of
thumb implies incompetence of the valve at saphenofemoral
junction. In Prat’s test elastic compression bandage is applied from
toe to upper thigh. A tourniquet is then applied on upper thigh
above the elastic bandage. With tourniquet in position the bandage
is unwinded in downward direction. In perforator incompetency,
a visible varix will appear at the site where the perforator is
incompetent. Perthe’s test is employed to test patency of deep veins.
A tourniquet is tightly tied around upper thigh and patient is asked
to walk for 5-10 minutes. If the varicose veins shrink, it implies
patency of deep veins and competency of perforators. When the
varicose veins remain unchanged or become more distended it
implies that the deep veins or perforators are incompetent.
Complications of varicose veins include pigmentation, eczema,
ulcerations, haemorrhage, thrombophleitis periostitis, venous
calcifications and foot deformity, usually talipes equinus due to
continued faulty habit of walking on toes to get relief. Venous ulcer
is a vexing problem and may need skin grafting. Carcinomatous
changes may occur in chronic venous ulcers.
Treatment is palliative or curative. Palliative treatment like use of
crepe bandage and elastic stockings, regular calf strengthening
36 SHORT NOTES AND SHORT CASES IN SURGERY

exercises, foot elevation during sleep, avoidance of prolonged


standing hold good in aged, during pregnancy and puerperium
and associated thrombophlebitis or deep vein thrombosis. Injection
of sclerosants like hypertonic salt/glucose, quinine, urethane,
tetradecyl sulphate/ethanolamine oleate may be done in recurrent
varicosities despite surgery, varicosity confined to below knee, and
vulval varicosity. 1ml of sclerosant is injected at various sites (point
of maximum varicosity, blow out sites) to a total of 10-15 ml in one
sitting. Surgical methods include ligation alone, ligation and
stripping and ligation of the incompetent perforators. Stripping is
usually not practised in short saphenous varicosity but it is ligated
in popliteal fossa. Ligation of long saphenous vein at sapheno
femoral junction along with stripping of long saphenous vein is
the Trendelenberg operation, commonly practised for longsaphe-
rous varicosity.

Leg Ulcers
Chronic leg ulcers in three-fourth cases is due to venous insuffi-
ciency. In the rest the causes may be traumatic, infective, diabetes,
neueropathic, ischaemic (sickle cell disease), neoplastic, etc.
Venous ulcer is always associated with venous incompetence
leading to venous stasis and local tissue anoxia. Venous stasis and
venous hypertension cause capillary stasis, pericellular edema and
poor oxygen transfer. Deposition of haemosiderin pigments at
caprillary stasis sites causes pigmentation. The skin becomes thin
and easily breaks off with minor trauma. The ulcer does not heal
because of anoxia and may turn malignant (Marjolin’s ulcer).
Infective ulcers can be pyogenic or gummatous, the latter occuring
in tertiary stage of syphilis as a result of break down of chronic
granuloma of muscle. Neuropathic ulcers occur due to undue trauma
or pressure in an area of sensory loss as in patients of polyneuro-
pathy (alcoholic, diabetic, leprosy, tabes, syringomyelia, etc). In
diabetes mellitus multiple factors predispose to leg ulcers like
athero-sclerosis, suceptibility to infection, diabetic neuropathy, etc.
The toes are commonly affected with often wet gangrene. Cryopathic
ulcers refer to cold injuries like chilblain and frost bite that cause
iscraemic injury as well as direct cell necrosis due denaturation
proteins and halt in enzyme function.
VARICOSE VEINS 37

Site of leg ulcer varies according to pathology. Venous ulcers occur


typically on medial aspect of lower third of leg. Ischaemic ulcers
involve toes, dorsum of foot, or heel. They are very painful and
patient has history of intermittent claudication. Peripheral pules
are weak and the skin has ischaemic changes like loss of hair,
thinning and fissuring of nails. Neuropathic ulcers also occur on
sole or heel, are painless but peripheral pulses are usually normal.
Malignant ulcer has rolled out and everted margin. Gummatous
ulcer is common on outer side of leg commonly around knee. It is
painless but is often multiple. Ulcer in lower leg is common in
haemoglobinopathies particularly sickle cell disease. This is due to
occlusion of small vessel vasospasm, hyperviscosity, etc. Ulcers can
occur in bacterial endocarditis due to septic emboli. Malnutrition
can often be associated with leg ulcers.

Lymphedema
Lymphedema is interstitial edema due to obstruction of lymphatic
flow. In primary lymphedema there is aplasia/hypoplasia or
varicosity of lymph channels. When varicosity extends to
para-aortic/pelvic lymph trunks, intestinal lymph may back flow
into limb lymph channels to cause vesicles beneath the skin. Primary
lymphedema is of 3 types—lymphedema congenita when the limb
swelling is present at birth or appears shortly ofterwards. When
familial it is known as Milroy’s disease. In lymphedema praecox, the
most common form, the swelling appears in childhood or adoles-
cence. In lymphedema tarda the swelling appears in adult life.
In secondary lymphedema, the lymph channels are either
obstructed due to filariasis or are fibrosed following irradiation,
Infiltration of lymphnodes by malignant metastasis or their block
dissection in early metastasis can lead to lymphstasis and
lymphedema. Accumulation of protein rich fluid over-time leads
to thickening of skin and overgrowth of fibrous tissue.
Limb swelling due to adiposis dolorosa (painful accumulation of
fat particularly around thigh, buttock and ankles), a-v fistula
(congenital or traumatic) and erythrocyanosis frigida (hypoplastic
microcirculation in female adolescents with leg swelling) should
not be confused with lymphedema. Similarly venous hypertension
as in congestive cardiac failure, vena caval obstruction, nephrotic
syndrome, hypoproteinemia, hepatic dysfunction can cause pitting
limb edema. Solid edema can be seen in myxedema, cellulitis, gas
38 SHORT NOTES AND SHORT CASES IN SURGERY

gangrese; contusion can also cause swelling of a limb. In deep vein


thrombosis the leg is swollen and painful.
Lymphangiography helps to outline the lymph channels and
can pinpoint the pathology. Surgical treatment of lymphedema is
varied. When lymphangiography shows chyle reflux from abdo-
minal lymphaties, ligation of lymphchannels at inguinal region is
rewarding. Tackling of the thick fibrosed limb tissue by reducing
operation with/or without skin grafting is often undertaken.
Creation of new lymphatic pathway by lymphovenous anastomosis
can be attempted in secondary lymphedema. However, surgical
outcome for lymphedema is poor. Often amputation is required
for severe disfigurement, lymph leakage, ulceration and physical
incapacitation.
6
Hernia

Hernia is an abnormal protrusion of a part or whole of viscus


through an opening in the wall of the cavity in which it is contained.
Abdominal hernias can be external or internal. Common external
hernias include inguinal (direct and indirect), umbilical, femoral,
incisional and the rarely encountered epigastric, lumbar, obturator,
gluteal and spigelian hernia.
Weakness of abdominal wall or increase in intra-abdominal
pressure are the precipitating factors for hernia. Abdominal wall
weakness can be congenital or acquired. Congenital weakness is
due to persistence of processus vaginalis (causes indirect inguinal
hernia in male), patent canal of Nuck (causes indirect inguinal
hernia in female). Acquired weakness is due to obesity, wasting
diseases, surgical wound with poor healing or faulty technique.
Increased intra-abdominal pressure is due to chronic constipation,
chronic cough, ascites, repeated parturition, etc.
A hernial sac consists of a mouth or neck, body and fundus. The
neck is the most constricted part and the fundus is the most
redundant or dependent part. The contents of hernia can be omentum
(omentocele), loop of intistine (enterocele). Meckle’s diver ticulum
(Littre’s hernia), a part of circumference of intestinal wall (Richter’s
hernia), ‘W’ shaped loop of intestine (Maydl’s hernia), a part of
bladder wall, ovary with or without fallopian tube. All external
hernias have characteristic expansile impulse in coughing - both
visible and palpable.
Complications of hernia include
1. Irreducibility
2. Obstruction
3. Strangulation
4. Incarceration
5. Torsion of the content.
40 SHORT NOTES AND SHORT CASES IN SURGERY

Irreducibility can be due to adhesion between the sac and its


content, narrowing of the neck, growth of omentum within the
hernial sac or retention and solidification of faeces in the intestinal
loop within the sac. Irreducibility can be accompanied by intestinal
obstruction with vomiting and absolute constipation but the hernia
remains painless and non-tender. Strangulation refers to compro-
mised circulation in the hernial content manifesting with features
of intestinal obstruction, absent cough impulse, irreducible painful
and tender hernial content. Then gangrene supervenes with escape
of colonic bacteria to invade systemic circulation to cause gram-
negative septic shock.

Inguinal hernia
Inguinal canal is an oblique fibro muscular tunnel of 4 cm length
lying above and parallel to inner half of inguinal ligament. It passes
downwards and medially from internal inguinal ring to external
inguinal ring. It transmits spermatic cord in male and round
ligament of uterus in female. Its anterior wall is formed by skin and
superficial fascia, external oblique aponeurosis, and internal
oblique muscle (in lateral third only). The posterior wall is formed
by fascia transversalis, conjoint tendon (in medial half) and reflected
part of inguinal ligament (in medial third). The floor is formed by
upper grooved surface of inguinal ligament and the lacunar
ligament (in medial third) while roof is formed by the arched fibers
of internal oblique and transversus abdominis muscles.
The superficial inguinal ring is a slit in the external oblique
aponeurosis and it lies 1 cm above and lateral to the pubic tubercle.
It transmits the spermatic cord (male) or round ligament (female),
the ilio inguinal nerve, genital branch of genitofemoral nerve. The
external oblique aponeurosis extends over the spermatic cord as
external spermatic fascia. Deep inguinal ring is placed 1 cm above
the mid point between anterior superior iliac spine and symphysis
pubis or 1 cm above the point where femoral artery passes under
the inguinal ligament. It transmits the spermatic cord or round
ligament. The fascia transversalis extends with the spermatic cord
as internal spermatic fascia. The processus vaginalis when present
passes through. The inferior epigastric artery coarscs up along the
medial side of the ring. Hesselbach’s triangle lies on the posterior
wall of inguinal canal bounded medially by outer border of rectus
abdominis, laterally by inferior epigastric artery and inferiorly by
HERNIA 41

inguinal ligament. It is bisected by the lateral umbilical ligament


which is obliterated hypogastric artery. Its floor is formed by fascia
transversalis and it contains the ilioinguinal nerve and the round
ligament of uterus/spermatic cord.
The natural mechanisms preventing inguinal hernia include
1. Obliquity of the canal
2. Shuttere action of arched fibers of internal oblique and
transversus abdominis
3. Sliding valve like action of the U shaped internal ring
4. Plugging action of spermatic cord due to contraction of cremas-
teric muscle.
Indirect inguinal hernia can be congenital due to persistence of
patent processus vaginalis or it can be acquired, appearing at any
age in adult life. The hernic may be incomplete (bubonocele) limited
to inguinal canal due to closure of processus vaginalis at superficial
inguinal ring. It can be funicular type where the processus vaginalis
is closed at its lower end, so that the sac of the hernia is separate
from sac of tunica vaginalis. When the hernia descends down to
bottom of scrotum covering the testis infront and at the sides it is
called vaginal or complete hernia. Both vaginal and funicular
hernias appear as inguinoscrotal swellings.
Coverings of indirect inguinal hernia include skin, dartos muscles,
external oblique aponeurosis/external spermatic fascia, cremasteric
muscle, internal spermatic fascia and the peritoneal covering.
Direct inguinal hernia protrudes directly through the posterior
wall of inguinal canal in the Hesselbach’s triangle. The neck of the
saclies medial to the inferior epigastric artery, where as in indirect
inguinal hernia it lies lateral. Direct inguinal hernia is common to
elderly and because of its wide neck-strangulation is rare. A direct
hernia seldom comes out through the superficial inguinal ring.
Uncomplicated inguinal hernia is an inguinal or inguinoscrotal
swelling with visible and palpable expansile impulse on coughing.
When it contains gut, it is soft, elastic and floctuant, resonant to
percussion. When it contains omentum the consistency is firm and
dloughy with dull on percusion and non-floctuant. Direct hernia
reduces directly backwards but indirect hernia reduces in upward,
backward and lateral direction.
An enterocele reduces with gurgling second. Coughing and
straining or even standing causes hernial swelling to reappear.
42 SHORT NOTES AND SHORT CASES IN SURGERY

When after reduction, the deep ring is occluded with thumb and
patient is asked to cough, indirect inguinal hernia fails to reappear
but not direct inguinal hernia. Invagination test is done to differen-
tiate an early indirect inguinal hernia from femoral hernia. The
index finger is invaginated through scrotum into external inguinal
ring. When patient is asked to cough thrust is felt at tip of finger in
incomplete inguinal hernia. Like inguinal hernia groin swelling can
also be due to femoral hernia, saphena varix, femoral artery
aneurysm, encysted hydrocle of cord, undescended or ectopic testis,
enlarged inguinal lymph nodes, psoas abscess, lipoma of cord etc.
In femoral hernia the swelling is below and lateral to pubic
tubercle but in inguinal hernia is above and medial to pubic tubercle.
Enlarged inguinal nodes are often multiple and without cough
impulse. Saphena varix has a positive fluid thrill, it is soft and
compressible disappearing on elevation of the limb. Aneurysm has
expansile impulse often with audible bruit. Encysted hydrocele of
cord is an elongated smooth tense cystic swelling. Transillumination
is positive but it is non-reducible and cough impulse is absent.
Lipoma of cord has similar features but is trans-illumination
negative. Psoas abscess is a floctuant swelling without any impulse
on coughing. It may be partially reducible. Tuberculosis of hip/
spine may be present.
Besides hernias other inguino scrotal swellings to be considered
are infantile hydrocele, congenital hydrocele, varicocele and lymph
varix. In infantile hydrocele the swelling is floctuant, translucent
and irreducible with absent cough impulse. Congenital hydrocele
is floctuant and translucent but reduces very slowly. Varicocele feels
like of bag of worms and on coughing there is palpable fluid thrill.
On lying down, if scortun is elevated varicocele gradually
disappears but on standing it reappears, filling up from bottom of
the scrotum.
Treatment of uncomplicated inguinal hernia is conservative or
surgery. Infants, old patients or those refusing surgery can be
prescribed to use truss that prevents descent of hernia but does not
cure. When hernia is associated with undescended testis truss be
never used. Truss be not prescribed when hernia is irreducible or
patient has chronic constipation, chronic bronchitis, prostatism, etc
that consistently increase intra-abdominal pressure. Truss can be
“rat tailed” or “adder headed” and should fit snugly over the
inguinal canal. It should be constantly used except during sleep.
HERNIA 43

Early surgery be advised in indirect inguinal hernia since chances


of obstruction and strangulation are more because of the narrow
neck. In direct inguinal hernia operation can be delayed since above
complications are rare because of wide neck. Before surgery the
factors raising intra-abdominal pressure be adequately addressed.
Surgery consists of
1. herniotomy alone
2. herniotomy plus hernirrhaphy
3. herniotomy plus hernioplasty.
Selection of a method depends upon patient’s age, type of
hernia and condition of musculature of the inguinal canal.
Herniotomy is excision of the sac only after reducing the hernia
and transfixing its neck. It is done in children and young adults
with good musculature. Herniorrhaphy is reconstruction of
posterior wall of inguinal canal by approximating the conjoint
tendon and muscle with the curved edge of inguinal ligament with
non-absorbable suture like silk or nylon. This is called Bassini’s
repair. To ease tension on conjoint tendon and muscle one may add
Tanner’s slide operation where a curved release incision is put on
anterior rectus sheath above the conjoint tendon. The patulous
internal ring can be plicated, and the fascia transversalis is often
plicated in direct inguinal hernia. In Halstead modification the
spermatic cord is exteriorised by suturing external oblique
aponeurosis behind it and. Willys-Andrews modification the cord
is sandwitched between the upper and lower flaps of external
oblique aponeurosis. In herniplasty the posterior wall of the
iaguinal canal is reinforced by prolene or stainless steel mesh or
autogenous tissue like a strip of fascia lata, a strip of external
oblique aponeurosis, a flap from anterior rectus sheath (Blood
good) or a strip of skin from the margin of hernial incision wound.
In direct inguinal hernia, herniotomy is not performed but the
sac is inverted after reduction of the content followed by plication
of fascia transversalis. Then the posterior wall of inguinal canal is
augmented by herniorrhaphy or herenioplasty. Orchidectomy with
obliteration of inguinal canal can be done in very elderly patients
with weak abdominal musculature.
Complications of inguinal hernia repair include injury to inferior
epigastric vessels, external iliac vein, vasdeferens, contents of the
sac, inardvertent suturing of ilio-inguinal nerve, damage to
testicular artery (leads to testicular atrophy), epidermoid cyst (when
44 SHORT NOTES AND SHORT CASES IN SURGERY

skin flap is used for herenioplasty), wound infection, injury to


bladder and finally recurrence.
Recurrence rate after successful repair ranges between 3-7 per
cent and most recurrences occur by end of first year. Wound
infection, inadequate haemostasis, failure to dissect the sac right
upto neck, inadequate repair of posterior wall of inguinal canal or
failure to recognise coexistence of both direct and indirect inguinal
hernias are responsible for recurrence so also persistence of preci-
pitating factors and early resumption of activity after surgery.

Femoral Hernia
In femoral hernia extraperitoneal fat and peritoneum with or
without abdominal content pass through the femoral ring to enter
into femoral canal. Femoral canal is the innermost compartment
of femoral sheath, the outer most compartment contains the femoral
artery and femoral branch of genito femoral nerve and the middle
compartment contains the femoral vein. The femoral canal is 2 cm
long and conical in shape with base directed upwards to the femoral
ring. Boundaries of femoral ring include—inguinal ligament
anteriorly, pectineal line of pubis and pectineal ligament
posteriorly, crescentic edge of lacunar ligament medially and a
fibrous septum separating it from femoral vein laterally. The ring
is closed above by condensed extra peritoneal tissue (septum
crurale) being pierced by lymphatic vessels. The femoral canal
contains fibrofatty tissue and Cloquet’s node at the femoral ring.
A branch of inferior epigastric artery often courses along the
crescentic margin of the lacunar ligament, and if the lacunar liga-
ment needs division to reduce femoral hernia, the artery will be
damaged.
Femoral hernias are always acquired and occur in middle aged
to elderly females, more so on the right side. The sac reflects
upwards after reaching the lower end of femoral canal (fossa ovalis)
since the fascia scarpa joins fascia lata at this site. Hence the sac is
retort shaped and the hernial content often overlie inguinal
ligament. The contents of sac are usually a part of omentum, loop
of small bowel or a part of circumference of small bowel (Richter
hernia). Femoral hernia is more prone to obstruction and strangu-
lation because of the rigid structures around femoral ring.
Femoral hernia be differentiated from bubonocele, saphena varix,
psoas abscess, ruptured adductor longus muscle, etc. Treatment of
HERNIA 45

femoral hernia is always surgery i.e. herniotomy with closure of


femoral ring by suturing inguinal ligament to the pectineal ligament
or the conjoint tendon to pectineal ligament. The surgical approach
to femoral hernia is through subinguinal approach (Lockwood’s),
inguinal approach (Lotheissen’s) and suprainguinal (Mc Evedy’s).
Each procedure has its own merits and demerits. While Lockwood’s
is the most simple one for small uncomplicated femoral hernia,
Mc Evedy’s is best for large gangrenous bowel and easy repair of
femoral ring.

Umbilical Hernia
Umbilical hernia is of 4 types—exomphalos, congenital umbilical
hernia, infantile umbilical hernia, adult umbilical hernia.
Exomphalos refers to protrusion of midgut through umbilicus due
to its failure to return to abdominal cavity associated with failure
of development of anterior abdominal wall. The protruding gutis
covered by a sac and the sac is likely to be ruptured during delivery.
The sac has outer layer of amniotic membrane, middle layer of
Wharton’s jelly and inner layer of peritoneum. The contents of sac
may vary from entire midgut to caecum and lower ileum.
Depending upon size of sac exomphalos can be minor and major.
In exomphalos minor, the sac is small and the umbilical cord is
attached to its summit. The sac may contain Meckel’s diverticulum
and loop of lower small bowel. Hence unless discovered, these
organs may be inardvertently included in ligature and cut across
during severance of cord. Twisting of umbilical cord reduces the
contents of sac into abdominal cavity. This can be followed by
severance of cord and strapping of abdominal wall for 2-4 weeks.
In exomphalos major the sac is longer and the cord is attached
to its base in inferior aspect. The sac contains small as well as large
intestine and nearly always a part of liver. Unless immdiate surgery
is done the sac may burst. Skin flaps are created by undermining
the subcutaneous tissue on either side in the flanks to cover the
sac. If required lateral release incisions can be made. If patient
survives, the ensuing large ventral hernia can be repaired later.
Intrauterine epithelialisation of exomphalos minor leads to
congenital umbilical hernia. Infantile umbilical hernia occurs
through weak umbilical cicatrix. It is more common to male child
and becomes prominent during crying and straining. A small gap
in umbilicus can always be palpated but obstruction and
46 SHORT NOTES AND SHORT CASES IN SURGERY

strangulation are rare herniorrhaphy is indicated if hernia persists


after 2 years of age. The defect in the alba is closed by overlapping
the rectus sheath with interrupted silk mattress sutures. Adult
umbilical hernia is rather paraumbilical, commonly occuring in
obese middle aged women who have increased intra-abdominal
pressure. The hernia occurs through weak area in linea alba. The
sac is usually multiloculated and the superficial layer of rectus
sheath is stretched over the protruding sac. The contents of the sac
are adherent to each other and to the sac and hence are often
irreducible. The neck of the sac is also narrow accounting for irredu-
cibility. Even after reduction the matted omentum can be felt as a
doughy mass. On palpation the defect in the linea alba can be felt
as a firm fibrous edge. Incarceration is more common than
obstruction. Treatment is always surgical, i.e. hereniotomy
followed by repair of the gap in linea alba by overlapping the flaps
of the anterior rectus sheath either from each side (Wells) or from
above and below (Wayo) with interrupted sutures.

Epigastric Hernia
It is the herniation of preperitoneal fat through a defect in linea
alba between xiphoid process and umbilicus. It is common to
manual laborers in middle age. The weak spots in linea alba at the
site of entering of blood vessels serves as the portal for exit of
preperitoneal fat during sudden strain. With time the extruded fat
drags a pouch of peritoneum. The mouth of the sac is very narrow
and does not allow any hollow viscus to enter into it. Hence the
sac is usually empty or may contain a small portion of greater
omentum. Patient is either asymptomatic or has epigastric pain at
the site worsened with physical activity. The epigastric pain may
simulate that due to peptic ulcer or gallbladder disease. Exami-
nation reveals of firm epigastic swelling, usually irreducible with-
out cough impulse. Treatment in symptomatic patients is surgical.

Lumbar Hernia
Lumbar hernia can be primary or secondary. In primary form the
hernia protrudes through the triangle of Petit whose base is formed
by iliac crest, the floor by the internal oblique and transversus
abdominis muscles, the anterior border by posterior free margin
of external oblique and the posterior border by anterior margin of
HERNIA 47

latissimus dorsi. The hernial sac has a wide neck and the content is
right or left colon. The secondary form usually follows operation
for kidney disease, hence a form of incisional hernia or follows
poliomyelitis that weakens the anterolateral abdominal muscles
(phantom hernia). The swelling is in lumbar region just above the
iliac crest, is reducible, soft and with positive cough impulse. It
can be confused with a lipoma-(firm solid swelling, non-reducible,
no cough impulse) or paravertebral cold abscess (soft cystic
floctuant non-reducible, absent cough impulse). Treatment is
surgical with herniotomy and augmentation hernioplasty by
dacron mesh or tentalum gauze.

Spigelian Hernia
Here the hernia protrudes through linea semilunaris along the
lateral border of rectus muscle at the level of the arcuate line. It
appears as a small tender swelling to one side of lower abdominal
wall along the lateral margin of rectus.

Incisonal Hernia
Here the peritoneal sac herniates through an acquired scar in the
abdominal wall, caused by trauma or surgical procedure. Scar
tissue is inelastic and can be stretched easily when subjected to
constant strain. Obese, diabetics, malnourished and those with
chronic cough/constipation are at particular risk. Mc Burney’s
incision for appendicectomy causing injury to ilio inguinal nerve,
Kocher’s subcostal incision for cholecystectomy causing injury to
ninth and tenth thoracic nerves, midline infraumbilical incision
for cesarean delivery are particularly liable to cause incisional
hernia. Defective closure of abdominal wound, wound sepsis,
wound haematoma, premature removal of suture or wound dehis-
cence are equally responsible.
The sac in incisional hernia is big, multiloculated and the
overlying skin is thinned out and adherent to the sac. Midline
incisional hernia usually reduces spontaneously but hernia in
lateral part of abdomen where the muscular defect is relatively
small and irregular is likely to be strangulated. The midline infra-
umbilical incision is frequently followed by incisional hernia
because below linea semicircularis the linea alba is thin and narrow,
and posterior rectus sheath is deficient. The hernial bulge is
48 SHORT NOTES AND SHORT CASES IN SURGERY

conspicuous on standing, coughing and straining with expansile


impulse on coughing. It reduces spontaneously.
Treatment is both conservative and surgical. The former is by
properly fitting corset, operative repair is by Mayo’s double breast
technique, by muscle pedicle flap, repair in layers (Cattle’s
technique), inlay or patch grafting, and Keel operation. Tensor fascia
lata and rectus femoris are preferred in muscle pedicle flap repair.
Cattle’s technique aims at anatomical restoration and is suitable
for small hernia with minimal scarring. In inlay or patch grafting
the defect in the abdominal wall is bridged across by implanting a
sheet of tantalum gauze/dacron/marlex/polypropylene mesh
tacked beneath the recti.

Sciatic Hernia
It refers to outpouching of abdominal contents through greater
sciatic foramen. Diagnosis is often made only after incarceration
or strangulation. Repair of the defect using flap from superficial
fascia or pyriformis muscle through abdominal approach is
necessary.

Obturator Hernia
Herniation through obturator canal at the upper border of obturator
membrane is common to women. Pain and paresthesia in pelvis
radiating to anteromedial aspect of thigh is due to compression of
obturator nerve. Small bowel obstruction is frequent. Preoperative
diagnosis is impossible without CT scan. Surgical closure through
suprapubic approach is essential.

Perineal Hernia
Here the hernia protrudes through the muscles and the fascia of
perineal floor. It may be primary or may follow abdomino perineal
resection of rectum. The hernia may be anterior or posterior to
trnsverse perineal muscles. Presentation is with perineal bulge often
causing dysuria. Repair can be made by abdomino perineal
approach using marlex mesh or flaps from gracilis or gluteus.

Interparietal Hernia
In it the hernial sac insinuates between layers of abdominal wall.
Strangulation is more likely. Ultrasound or lateral film of abdomen
HERNIA 49

will disclose bowel loops in abdominal wall; surgical repair is


curative.

Internal Hernias
The 4 major categories of internal hernia are paraduodenal hernia,
hernia into foramen of Winslow, mesenteric hernias and omental
hernias. Presentation is with chronic subacute intestinal obstruction
or digestive complaints. Small bowel series may show the trapped
bowel. Surgical decompression is required.

Diaphragmatic Hernia
Postero lateral foramen of Bochdalek hernias are farmore common
than substernal foramen of Morgagni hernias. Eighty to ninety per
cent are left sided. Foramen of Morgagni hernia is very rarely symp-
tomatic in neonatal period. Incomplete development of posterior
diaphragm leads to persistence of pleuroperitoneal communication
and abdominal content find their way to chest with underdevelop-
ment of lungs and a small abdominal cavity. The infant presents
with progressive respiratory distress, retraction of chest and
cyanosis. The mediastinum is shifted usually to right and heart
sounds are more clear on right chest. Ipsilateral chest is dull on
percussion, breath sounds are diminished and bowel sounds may
be heard in chest. Respiratory compromise may cause early death
but those with minimal pulmonary dysfunction can survive for
days to months. Chest radiograph demonstrates multiple loops of
airfilled intestine within the hemithorax. However, if X-ray is done
soon after birth, prior to any air entering bowel, the hemithorax
may look opaque with mediastinal displacement. Nasogastric
suction decreases/prevents further bowel distension and
respiratory distress. Hyperventilation with controlled alkalosis may
improve oxygenation. Hypoxia induced pulmonary arterial spasm
can lead to pulmonary hypertension and further hypoxia in a cyclic
fashion. Such infants can be better rescued by extracorporal
membrane oxygenator (ECMO). Cannulation of right atrium
through jugular vein and the aortic arch through right common
carotid artery can divert 80 per cent of cardiac out put to the
oxygenator thus reversing the pulmonary hypertension. The hernia
can be repaired by subcostal approach but trans thoracic approach
may be preferred for older children or those with recurrence. The
50 SHORT NOTES AND SHORT CASES IN SURGERY

attenuated posterior rim of diaphragm is sutured to anterior


diaphragmatic muscle after returning the intestinges into abdomen.
When abdomen is too small, it may not accomodate the intestines,
and in that case the skin is only closed creating a ventral hernia so
that pressure on inferior vena cava and diaphragm is relieved. The
ventral hernia is corrected 2 weeks later. Even after successful
diaphragmatic repair the infant is in danger of sudden deterioration.
The risk of alveolar rupture and life-threatening pneumothorax is
high. Hence after repair, ventilatory assistance with high rates but
low pressure be continued. These infants have tendency to develop
increasing pulmonary vascular resistance and persistent fetal
circulation. Tolazoline may decrease pulmonary hyperetension so
also nitrus oxide, prostacycline and sildenafil.
7
Testis, Scrotum and Penis

Cryptorchidism
In cryptochidism the testis is absent from scrotum. The testis
develops retroperitoncally in the lumbar region from the genital
ridge on the medial aspect of mesonephros. It is then attached to
posterior abdominal wall by a fold of peritoneum the mesorchium
that contains the testicular artery and branches of D10 and D12
nerves. The fibromuscular gubernaculum is attached to lower pole
of testis and bottom of the scrotum whose contraction brings down
the testis to scrotal sac. In 7th month of fetal life testis is at deep
inguinal ring, by 8th month in superficial inguinal ring and shortly
before birth in the scrotum. Factors that hinder with the scheduled
testicular descent include retroperitoneal adhesions, short vas
deferens/papminiform flexus/testicular vessels or gonadal
dysgenesis that makes testis insensitive to gonadotrophins. When
testicular descent is arrested along normal path of descent it it called
undescended testis but when deviated from normal path of descent
it is called ectopic testis.
An undescended testis fails to develop normally. It becomes
soft and flabby. It is more liable for torsion due to long mesorchium.
Though the interstitial cells develop normally, the germinal
epithelium is destroyed. Hence, in bilateral cases sterility is usual
despite normal androgen levels.
Nearly, 4 percent of full-term infants have undiscended testis
and in 80 per cent cases this is unilateral. It is more common on
right side. Retractile testis is one which is pulled up from scrotum
due to overeactive cremaster muscle. It gradually comes back to
its normal position by puberty, is well developed in structure and
function. Ectopic testis is less common than incompletely
descended testis and can be found in supereficial inguinal pouch
(above and lateral to superficial inguinal ring and superficial to
52 SHORT NOTES AND SHORT CASES IN SURGERY

external oblique aponeurosis-commonest site), root of penis,


perineum, femoral triangle in order of frequency. The ectopic
positions may be explained by presence of accesspry gubernacular
tails of Lockwood that misdirect the testis during descent. The
ectopic testis is well developed with good hormonal function and
spermatogenesis. The scrotum is well developed. An ectopic testis
becomes more prominent when the abdominal muscles are tensed
as it lies on the taut external ablique oponeurosis. On the contrary
the undescended testis in inguinal canal becomes less prominent
and impalpable.
Undescended testis is likely to have following complications—
atrophy, torsion, herniation, inflammation and malignant changes.
10 percent of testicular tumors occur in undescended testis and
chances of malignant change in undescended testis is 50 times
higher than in fully descended testis. Though all types of malignant
testicular tumors may occur in undescended testis seminoma is
the most common. Either indirect inguinal hernia or interstitial
hernia is associated with undescended testis. The appearance of a
lump in the line of testicular descent, whether within the abdomen
or in inguinal canal should arouse suspicion fo malignant change
in undescended testis.
Surgery in the treatment for undescended testis. Fixation of testis
in the scrotum (orchidopexy) or removal of testis (orchidectomy)
are done. Orchidopexy improves chances of fertility, reduces risk
of torsion. It however does not reduce risk of malignancy but
facilitates its easy and early detection. All testicles be brought into
scrotum by 5 years to improve spermatogenesis and reduce the
incidenced of malignancy. When delayed uptil puberty, it may not
be possible to lengthen the spermatic cord and bring down the
testis to scrotum. There are various techniques of orchidopexy, each
having its merits and demerits. The procedure can be done in single
stage or two stages. The testis is placed in contralateral scrotum
via the median septum (Ombredanne’s technique) or in the dartos
pouch on same side. In Keetley Torek technique the testis is placed
into a pocket superficial to facia lata of upper inner thighs and
after 3 months the testis is replaced into the scrotum. Lengthening
of spermatic cord is achieved by incising the fascia transversalis
medially from deep inguinal ring and dividing the inferior
epigastric vessels which makes the course of spermatic cord more
straight. In Fowler-Stephen’s procedure the testicular vessels are
TESTIS, SCROTUM AND PENIS 53

sacrificed, relying on the cremasteric vessels to prevent infarction


of testis.
Hormone therapy is advocated when undescended testis is in
inguinal canal and in retractile testis Beta hCG and GnRH are used
but they may precipitate precocious puberty. Since ectopic testis
coexists with inguinal hernia in many cases surgery has been the
gold standard treatment for cryptorchidism

Cysts of Epididymis
During development the wolfian body disappears but the wolfian
duct persist as the epididymis and vasdeferens. A cyst of
epididymis may arise due to cystic degeneration of the vestigeal
remnants in epididymis like paradidymis, appendix of epididymis.
Though congenital in origin it is usually found in middle life. The
cyst is multilocular usually situated at the head of epididymis,
hence above and behind the testis. Floctuation is positive and the
cyst is brilliantly transilluminant with chinese lantern or marble
floor appearance. Palpation may give feel of bunch of grapes and
aspiration yields clear fluid free of sperms. Small cysts be left as
such but larger cysts causing dyscomfort need excision. However
blockage of sperm passage may occur following surgery.

Spermatocele
Spermatocele is an unilocular retention cyst derived from sperm
conducting channels of epididymis. The swelling lies above and
behind the testis, is soft and cystic. Though floctuation is positive
transillumination is poorly positive. Aspiration yields barley water
like fluid containing spermatozoa. Larger cyst causing dyscomfort
may need aspiration or excision.
Processus vaginalis is the peritoneal diverticulum dragged down
by the testis during its descent. It becomes occulded soon after
birth at the deepinguinal ring and just above the testis. The part
between the two occlusions is the funicular process which gets
obliterated forming a fibrous cord. The part of the sac in relation
to testis is the tunica vaginalis.

Hydrocele
Hydrocele refers to abnormal collection of fluid in tunica vaginalis
of testis or in the funicular process. When the funicular process
54 SHORT NOTES AND SHORT CASES IN SURGERY

remains patent peritoneal fluid finds its way to tunice vaginalis


forming a communicating hydrocele which is commonly congeni-
tal.
Hydrocele can be primary or idiopathic and secondary; due to
trauma to testis, epididymorchitis, lymphatic obstruction or
testicular malignancy. Besides congenital hydrocele, infants can
have funicular hydrocele (the patent processus vaginalis is closed
off above testis); infantile hydrocele (fluid accumulation in tunica
vaginalis and funicular process upto deep inguinal ring where the
processus vaginalis is closed). The patent procesus vaginalis usually
closes by 12-18 months of age. So congenital hydrocele persisting
beyond 2 years needs surgical treatment.

Vagital Hydrocele
It is a scrotal swelling due to accumulation of fluid between the
parietal and visceral layers of tunica vaginalis. The fluid is straw
or amber coloured. It contains inorganic salts, traces of albumin
and fibrinogen. In long-standing cases it may contain tyrosine and
albumin crystals.
Since the swelling is scrotal, one can get above the swelling unlike
inguinal hernias which are inguin scrotal swellings and one can
not get above the swellings. The testis can not be felt separately.
Floctuation and transillumination are positive. When transillumi-
nation is negative thick scrotal skin or tunica and testicular tumor
are likely else the fluid is turbid. Vaginal hydrocele be differentiated
from chylocele, haematocele, encysted hydrocele of cord and cysts
of epididymis.
Complications of hydrocele include infection, rupture,
haematocele testicular atrophy, hernia of hydrocele sac through
dartos and calcification of the sac. Big hydroceles are socially
embarassing and hinder with physical activity.
Treatment of hydrocele is surgery but tapping, aspiration with
injection of sclerosing agents can be done depending upon urgency.
Surgery however offers best chances of cure and recurrence is
unlikely. Jaboulay’s operation is the time tested one for small and
medium sized hydroceles where the vaginal sac is everted behind
the testis and epididymis and is stitched with a running suture so
that the secreting surface is made inside out. Excision of the sac
may be necessary in very large hydrocele, haematocele, chylocele
where the sac is thick leaving behind a margin of 2 cm by the side
TESTIS, SCROTUM AND PENIS 55

of the testis. A continuous mattress suture along the margin stops


bleeding. Redundant scrotal skin may be excised to reduce the size
of scrotum. Lords procedure is often practised in small hydrocele
where the edge of the sac are plicated around the periphery with
interrupted sutures. Tapping brings immediate relief but
complication do occur like infection, haematocele, injury to testis
and high recurrence.
Hydroceles in middle aged developing slowly need ultrasound
to document the type of fluid, thickness fo tunica and the status of
testis. Testicular tumors may masquerade as hydrocele.

Varicocele
Varicocele refers to dilatation, elongation and tortuosity of the veins
of pampiniform plexus. Often the testicular veins are normal and
varicosity is confined to cremasteric veins that anastomose freely
with testicular veins. The pampi inform plexus consists of 3 groups
of veins-i.e. veins of testis epididymis, veins of vasdeferens and
veins of cremasteric muscle. The venous plexus surrounding the
vas has 15-20 venous channels that gradually reduce in number
during ascent to form the single testicular vein in the abdomen.
The right testicular vein to inferior vena cava at an acute angle
where as the left testicular vein drains to left renal vein in a right
angle. Testicular veins have valves near termination. The
cremasteric veins drains into inferior epigastric veins. The pamping
form plexus is believed to form a heat exchanges system that
reduces scrotal temperature and aids in spermatogenesis.
Varicocele can be primary or idiopathic where no definite cause
is established. Nearly 95 per cent of varicocele are of this group.
Secondary varicocele is due to obstruction to testicular venous
return as in retroperitoneal tumor, fibrosis or venous thrombosis.
Most primary varicocele (95 %) occur on left side because the left
testicular vein is longer, joins left renal vein at right angle, is often
compressed by loaded left colon. Venous return from left testicular
vein is hindered due to sandwitching of left renal vein between
abdominal aorta and trunk of superior mesenteric artery; arching
over by left testicular artery over left testicular vein and growth of
tumor tissue in hyper nephroma into left renal vein. Recent onset
of varicocele in middle aged calls for exclusion of (left)
hypernephroma.
56 SHORT NOTES AND SHORT CASES IN SURGERY

Varicocele is one of the causes of male subfertility or infertility


as spermatogensis in the affected testis is hampered due to raised
temperature consequent to stagnation of blood in the varicocele.
Most patients have a feeling of dragging on effected side. Palpation
gives the impression of bag of worms. Cough impulse is positive.
The scrotal swelling reduces when patient lies down and the
scrotum is elevated. The affected side testis may be some what
smaller and softer. On standing the varicocele fills up from bottom
and the swelling reappears. Secondary varicocele does not reduce
on lying down and elevation of scrotum.
Small varicocele need reassurance and scrotal support. Larger
symptomatic ones or those causing subfertility need surgery. The
aim of operation is to reduce the number of veins in pampiniform
plexus to minimum. This can be done by abdominal extraperitoneal
approach (Palomo), inguinal approach or scrotal approach. In the
classical inguinal approach the inguinal canal is opened, the
vasdeferens together with its artery and 1-3 veins are separated
from the main mass of dilated veins, and 5cm of isolated dilated
veins are removed between two clamps. The two clamps are then
approximated and the two ends are tied togtether. In scrotal
approach the tortuous varicose veins are divided and ligated
individually leaving few to maintain venous drainage. In Palomo
operation, the testicular vessels are exposed through an oblique
incision given 3 cm above deep inguinal ring, the testicular vein is
identified and divided between ligatures. However it can aggravate
varicocele in 10 per cent cases. By and large Palomo method is safe
and simple, the cremasteric vein continues to drain the testis. Even
if testicular artery is erroneously divided artery to vas and cremas-
teric artery can provide adequate collateral circulation to testis.

Phimosis
Phimosis is a disorder in which the prepuce can not be retracted
back to expose the glans and external meatus. In children upto 3
years the prepuce may be normally adherent to glans penis but
the external meatus is clearly seen. Phimosis may follow
balanoposthitis, penile trauma and penile malignancy spreading
from corona glandis. Phimosis in aged and of recent onset calls for
exclusion of penile malignancy.
Patient can have difficulty in micturition, intercourse and
recurrent balanitis causing pain and purulent discharge. Ballooning
TESTIS, SCROTUM AND PENIS 57

of prepuce during micturition may be seen. Calculus may form in


the prepucial sac. Patient may have paraphimosis, a complication,
where the retracted prepuce refuses to go forward and forms a
constricting ring around corona glandis impeding venous return.
The prepucial skin distal to constricting band becomes swollen and
edematious. In neglected cases arterial occlusion can cause necrosis
of glans.
Excision of the prepucial skin (circumcision) under LA/GA is
the treatment. A continuous absorbable suture on the foreskin
achieves haemostasis. The frenal artery must be carefully secured
to prevent postoperative haemorrhage. Paraphimosis is an
emergency and the constricting band be divided along with
circumcision. If a manual reduction offer hyaluronidase injection
at 9’ and 3 o’clok position on the band fails.

Carcinoma of Penis
More than 50 per cent cases fo carcinoma penis are associated with
phimosis. Chronic irritation by accumulated smegma in prepucial
sac is responsible. Hence, circumcision after birth confers complete
protection against cancer penis. Jews are completely immune to
cancer penis as in them circumcison is ritual soon after birth. In
muslims, the ritual is at 4-9 years and hence they run some risk.
Recurrent attacks of balano posthitis, long-standing venereal warts,
leukoplakia of glans Paget’s disease of penis, Bowen’s disease,
erythroplasia of Queyart and balanitis xerotica also predispose to
penile cancer. In most cases (97 percent) the penile cancer is of
squamous cell carcinoma, rarely it is adenocarcinoma when arising
from Tyson’s glands, the smegma secreting gland on eitherside of
phrenum. Very rarely melanoma, basal cell carcinoma mesenchymal
and lymphoreticular tumors may occur in penis.
The lesion usually occurs on glans and the inner surface of the
prepuce near its reflection from corona. Macroscopically the growth
can be ulcerative, fungative (cauliflower type), fissure type or
plaque like. The growth is very slow and remains localised for many
months. Early spread to body of penis is prevented by Buck’s fascia
but once the fascia is penetrated dissemination is rapid. Urethra
mostly remains uninvolved even in late cases. Lymphatic spread is
earliest to superficial and deep inguinal nodes bilaterally. Glans
penis drains to Cloquet’s nodes in femoral canal. Blood spread
though rare can involve lungs and liver.
58 SHORT NOTES AND SHORT CASES IN SURGERY

Diagnosis is evident from ulcerative or fungative growth with


serosanguinous discharge. The raised everted edge, indurated base
and friable surface bleeding on touch are characteristic. Biopsy is
diagnostic. However pelvic CT, bone scan and chest X-ray are
required to delineate spread of the disease.

Staging
Stage I Tumor confined to glans or prepuce
Stage II Tumor involves shaft of penis
Stage III Involvement of inguinal lymphnodes
Stage IV Distant metastasis
Radiotherapy is indicated for small (< 3 cm) lesions on glans in
young patients as it preserves penile structure. Carcinoma
supervening on leukoplakia and syphilis be always treated with
surgery. Radiotherapy may cause urethral stricture and sterility
Recurrences are common after radiotherapy and infected tumors
respond poorly. Larger invasive anaplastic growth need partial
amputation with 2 cm margin of safety. When growth has involved
proximal shaft total amputation with perineal urethrostomy may
be warranted. The scrotum and testis are preserved but the testes
are ablated to kill the sexual desire because of a small penile stump
or no stump at all.
8
Anal Canal and Rectum

Anal Fissure
Anal fissure is an elongated crack or ulcer situated at anal verge or
in long axis of lower anal canal. Irritant diarrhoeal stool, prolonged
constipation and too much tightening of anal canal secondary to
anxiety are the three most precipitating factors. Other causes
include cathartic abuse, chronic diarrhoea, avulsion of analvalve
but very often no definite cause can be identified. The fissure is
very painful because of its location below microcutaneous junction.
The most common site is the posterior medial because of acute
angulation between rectum and anal canal. At the lower end of
fissure there is a tag of skin, the sentinel pile and at the upper end
an enlarged papilla may be present. Patient presents with recurrent
attacks of sharp agonising anal pain during defaecation that lasts
for hours after the act. The stool may be streaked with blood
corresponding to the site of fissure.
Conserveative treatment of fissure is with stool softners, high
fiber diet and steroid suppository along with 5 percent xylocaine
local application. Anal dilatation under local or general anaesthesia
may be rewarding. Non-responders and chronic fissure need
surgical intervention in the form of lateral internal sphincterotomy
or fissuretomy with posterior midline sphincterotomy.

Haemorrhoids
Haemorrhoids are dilated veins originating in the sub-epithelial
plexus above the pectinate line at 3, 7, 11 o’clock positions, the
areas fed by three terminal branches of superior haemorrhoidal
artery. The collecting radicals of the superior rectal vein lie
unsupported in submucous connective tissue and as they pass
through sphincteric tissue are liable for constriction due to
60 SHORT NOTES AND SHORT CASES IN SURGERY

sphincteric contraction during defaecation. Too much of straining


due to constipation, urinary obstruction precipitate haemorrhoid.
Portal hypertension, compression of superior rectal vein by
pregnant uterus (pregnancy piles) or thrombosis of superior rectal
vein in rectal carcinoma are few secondary causes of haemorrhoids.
Internal haemorrhoids can be graded according to degree of
prolapse. First degree the haemorrhoids descend down beyond the
dentate line on straining and mostly manifest with painless rectal
bleeding. In second degree they prolapse beyond anus during
straining but reduce spontaneously. Third degree prolapse needs
mannual replacement.
External haemorrhoids occur below the dentate line in the tissues
beneath epithelium of anal canal and the skin around anal canal
and drain to systemic circulation. Bleeding is first symptom of
internal haemorrhoids. The blood is bright red unmixed with stool.
Dyscomfort and pain occur when there is thrombosis with edema
and inflormmation. Severe pain is a feature of external haemorrhoid
thrombosis. Proctoscopic examination is necessary to see the
pinkish blue soft and non-tender masses at 3, 7, 11 o’clock position
that become more prominent on straining. Complication of
haemorrhoids include profuse bleeding, thrombosis, strangulation
ulceration, gangrene fibrosis and often portal pyemia with liver
abscess. Prolapsed internal haemorrhoids are vulnerable for
thrombosis and infarction with gangrene and ulceration. Often
thrombosis leads to fibrosis and the haemorrhoid looks white with
firm consistency.
Treatment is dependent upon symptomatology and degree of
prolapse. Anal dilatatioan under GA can cure first degree piles.
Increased water intakes, stool softners and high fiber intake are
essentials. Barron’s bands can be of value in first and second degree
piles but ensuing gangrene causes intense pain. Injection of
sclerosants like 5 percent phenol in almond oil is a good alternative
provided correctly injected into submucosal space. Freezing of
haemorrhoid with cryoprobe (NO2, CO2) can cause necrosis and
sloughing out of the haemorrhoidal mass but profuse rectal
discharge is often trouble some. Haemorrhoidectomy is indicated
for third degree internal haemorrhoids, intereno-external piles and
fibrosed haemorrhoids. Photocoagulation of piles using an infrared
source is also curative.
ANAL CANAL AND RECTUM 61

Failure of surgery is due to appearance of haemorrhoids at


secondary branchings of superior haemorrhoidal veins. The painful
strangulated internal piles may call for emergent haemorrhoidec-
tomy. Thrombosed external haemorrhoids are painful tense bluish
elevations beneath skin of anal canal. They may occur in healthy
and have no relation with internal piles. When pain is severe,
evacuation of thrombus or haemorhoidectomy may be done under
LA. Anal stricture may follow surgery for haemorrhoids.
Drug treatment of internal piles is often disappinting flavenoids
(Doflon), venusmin and calcium dobesilate can control bleeding.
Local application of ointments containing heparinoids, hydrocon-
tisone allantoin, aminobenzoates are in vogue. They donot cure
the disease but certainly control bleeding.

Rectal Prolapse
Rectal prolapse can be complete or partial. In partial prolapse, the
rectal mucous membrane and sub-mucosa protrude outside the
anus for half to one-half inches. When palpated between thumb
and fingers, the double layer of mucous membrane is appreciated.
In complete prolapse the protrusion consists of all layers of rectal
wall and on palpation, the prolapse feels much thicker. Rectal
prolapse be differentiated from prolapsed haemorrhoids, prolapsed
rectal polyp and ileocecal intussusception.
The normal rectal support consists of mesentery posteriorly,
peritoneal folds, fascial attachments, the rectal curvature and the
levator muscles. The puborectalis sling maintains an acute angle
between rectum and anal canal at resting state.
In infants and children rectal prolapse is usually congenital and
is due to lack of skeletal support and fixation. In adults it is due to
injury to levators, diseases of cauda equina, increased intra
abdominal pressure, etc. Initially prolapse occurs with straining
and reduces spontaneously but with time the prolapse is constant,
causing weakening of anal sphincter and incontinence. A complete
neurological examination, sigmoidoscopy and barium enema are
essential so also assessment of anal sphincter tone.
Conservative treatment consists of stool softeners to reduce
straining and submucosal sclerotherapy to induce scarring and thus
promote fixation. In Thiersch procedure a loop of stainless steel wire
as synthetic mesh is placed submucosally. In Ripstein procedure,
62 SHORT NOTES AND SHORT CASES IN SURGERY

teflon, ivalon or mersilene mesh graft fixes upper rectum to


presacral fascia. In Well’s operation a sheet of polyvinyl sponge is
placed between rectum and sacrum. Another approach is excision
of the redundant prolapsed rectum and formation of a low anterior
anastomosis between the colon and lower rectum.

Proctitis
Imflammation of rectum can be due to gonocococci, chlamydia,
HSV or HIV. Gonococcal and chlamydial proctitis are common to
homosexual males. In female the infection can spread to rectum
from gonococcal/chlamydial vaginitis/cervicitis via lymphatics.
The infection can be subclinical. In gonococcal proctitis proctoscopy
reveals hyperemic and edematous rectal mucosa with purulent
discharge. Rectal stricture is a common sequel of chlamydial
proctitis. Free test and LGV complement fixation test are diagnostic.
While gonococci are sensitive to peniciullin quinolones,
cephalosporin, chlamydia are sensitive to tetracyclines. Early rectal
stricture can be dilated by finger or dilator but complete obstruction
may demand colostomy. HSV II causes anorectal herpes. Pain and
soreness around anus are usual. Examination reveals erythematous
red areas with small group of vesicles or shallow ulcers. Treatment
is with oral acyclovir. HIV rectal infection occurs in advanced
disease and is often due to MAC organism or CMV.

Condyloma
Anal warts are caused by human papilloma virus. Itching, bleeding
and irritation are common. The disease may range from few small
warts to extensive fungating mass. It should not be confused with
the condycoma lata, the flat eroded pink to gray white infectious
lesions of secondary syshilis. Treatment is with trichloracetic acid/
podophylon lotion, CO2 snow or laser excision Lesional interferon/
iodoxy uridine cream may be of help.
9
Diseases of Intestines

Appendicitis
Appendix lies 2.5 cm below the ileocecal valve and all the taeniae
coli converge at the base of appendix. Though mobile it may be
fixed retrocecally in upto 16 per cent cases. It contains clusters of
lymphoid follicles that are liable for hypertrophy, obstructing the
lumen and initiating acute appendicitis. In two-third cases of acute
appendicitis, there is obstruction in proximal lumen by fibrous
band, fecolith, parasite or foreign body. In rest intraluminal
obstruction is not apparent. Obstruction of lumen leads swelling
of appendix, compromise in its circulation, gangrene and per-
foration.
Clinical manifestations of acute appendicitis are protean. It may
simulate any acute abdominal condition and in turn may be
mimicked by a variety of conditions. However, progression of signs
and symptoms are the rule unlike floctuating signs and symptoms
of some other acute abdominal illnesses. The pain of acute
appendicitis first is felt around umbilicus, is a continuous ache
which gradually localises to right iliac fossa. Mild nausea and few
bouts of vomiting are not unusual. Fever if present is mild. Bowel
sounds are normal. Examination reveals tenderness in right iliac
fossa and rebound tenderness classically referred to right lower
quadrant (features of involvement of parietal peritoneum). Pelvic
appendicitis may present with diarrhoea and rectal examination
reveals pelvic tenderness in absence of abdominal pain and
tenderness. In retroceeal or retroileal appendicitis pain is poorly
localised but urinary frequency and haematuria may be there due
to irritation of ureter. A long appendix may reach other parts of
abdomen with confusing presentation.
Mild polymorphonuclear leucocytosis is usual. Plain X-ray
abdomen may show airfluid levels, localized ileus, altered right
64 SHORT NOTES AND SHORT CASES IN SURGERY

psoas shadow or increased soft tissue density in right iliac fossa.


In 55 per cent cases ultrasound may show the dilated and swollen
appendix. CT scan of abdomen is often diagnostic, particularly
when there is peri appendiceal abscess.
Complications include perforation, appendicular abscess,
pyelophlebitis and paralytic ileus due to peritonitis. Perforation is
accompanied by more severe pain and high fever. Microscopic
perforation of gangrenous appendix causes localized peritonitis
and the periappendiceal infection is walled off by omentum and
adjacent viscera with formation of a mass. Generalized peritonitis
is marked by board like rigidity, adynamic ileus and abdominal
distention. Pyelophlebitis is marked by chills, low grade fever,
jaundice and later hepatic abscess. CT scan best demonstrates the
thrombus and gas in portal vein.
Appendectomy is the treatment of choice before formatiaon of
mass due periappendiceal infection. In that case antibiotics are
given and elective appendectomy is done 6-12 weeks later.
Appendicular abscess may be drained udner ultrasound guidance.
Pyelophlebitis needs vigorous antibiotic therapy. Appendictomy
is done through Mc Burney’s incision. Tip of cecum is delivered
through the incision and mesoappendix is divided. The base of
appendix is clamped, ligated and appendix is removed. The stump
is inverted with a purse string suture of fine silk.
Appendicitis may go unrecognised in children, elderly and
pregnant patients and 75 per cent of them have already perforation
by time of presentation. Hence in them greater vigil is required.
Amoebic typhlitis may mimic appendicitis but rebound tenderness
is absent and progression is unusual. Acute epiploic appendagitis
is often mistaken for appendicitis since it presents with pain, fever,
leukocytosis and often a mass. Surgical excision of the offending
epiploicale is warranted. Salpingitis, rupture/torsion of ovarian
cyst, red degeneration of fibroid, acute mesenteric lympladenitis,
ruptured ectopic pregnancy, ureteric colic may mimic acute appen-
dicitis and need careful exclusion. A pelvic/vaginal examination
is always mandatory.

Mucocele
Mucocele of appendix is cystic dilatation of appendix due to
accumulation of mucin. Chronic obstruction of proxinal lumen by
fibrotic band is the usual cause and appendectomy is the treatment
DISEASES OF INTESTINES 65

of choice. Cyst adenoma can cause mucocele but the walls are lined
by columnar epithelium with papillary projections. Appendectomy
is curative provided wall infiltration and metastasis are absent.
Excised appendix always be subjected to histopathological
examination for evidence of maligancy.

Small-Intestinal Obstruction
Obstruction is the most common surgical disorder of small
intestine. Adhesions account for 60 per cent cases, external hernias
for 15 per cent, neoplasms for 15 per cent and the rest by gallstones,
worms, intussusception, tuberculosis, Crohn’s disease, etc. Internal
hernias (obturator foramen of Winslow retroperitioneal fossae, hole
inmesentery/transverse mesocolon can cause obstruction when
incarcerated. Among retroperitoneal fossae, the para duodenal,
ileocecal and intersigmoid are the important ones. Congenital
bands and intussusception are common to children. In the latter
there is invagination of one intistinal loop into another and this is
brought about by an intraluminal polyp or mass.
The small bowel proximal to the point of obstruction distends
with pooled intestinal secretions and gas. Swallowed air is the
major source of gaseous distention as the nitrogen is not well
absorbed by mucosa. Bacterial fermentation occurs later and adds
to volume of the gas. The rising intraluminal pressure impedes
venous return causing edema of bowel wall and loss of fluid from
serosal surface to peritoneal cavity. Vomiting ensues and the
dehydration can kill even in absence of strangulation, bowel
gangrene and generalised peritonitis. Strangulation means artetrial
occlusion and this does not occur in simple obstruction. Closed
loop obstruction where the lumen is occluded in atleast two places
(hernia, volvulus) strangulation is a potential danger. The
gangrenous bowel eventually perforates with generalised
peritonitis. The gram-negative intestinal bacteria and their product
may enter circulation to produce septic shock. The gangerenous
bowel may bleed into peritoned cavity.

Signs and Symptoms


Proximal small bowel obstruction presents as profuse vomiting
and upper abdominal pain/dyscomfort. Mid or distal intestinal
obstruction causes cramping periumbilical or poorly localised
66 SHORT NOTES AND SHORT CASES IN SURGERY

abdominal pain vomiting follows pain after an interval that varies


will level of obstruction - more distal the obstruction vomiting is
late and is often feculent. Absolute constipation is the rule without
passage of stool and flatus. Abdominal distention is minimal in
proximal obstruction but pronounced in distal obstruction. The
step ladder peristalsis may be visible and bowel sounds are
exaggerated coinciding with attack of cramping pain.
Standing plain abdominal films show a ladder like pattern of
dilated small bowel loops with air and fluid levels. These features
may be minimal to absent in early obstruction, proximal
obstruction, closed loop obstruction or when the fluid filled loops
contain little gas. Appearance of high fever, shock, blood stained
vomitus, abdominal rigidity, gas within bowel wall or within
intrahepatic branch of portal vein are pointers towards strangu-
lation. Ultrasound may detect intussusception from presence of
two concentric bowel walls. Atleast one-third of strangulation are
unsuspected before operation.

Differential Diagnosis
Mesenteric vascular occlusion, acutipancreatitis, paralytic ileus can
mimic intestinal obstruction. Besides pain, blood tinged diarrhoea
is common in mesenteric vascular occlusion. Pain of acute
pancreatitis is more severe, constant, radiates to back and patient
prefers to sit and bend forward for relief. In paralytic ileus pain is
constant and diffuse and bowel sounds are absent.

Treatment
Partial obstruction where flatus and feces continue to pass can be
managed by decompression with Ryles tube and maintenance of
water and electrolyte balance. In complete obstruction patient is
prepared for surgery-longer the duration of obstruction, longer
the duration of preparation. Nasogastric suction be immediately
started to relieve distention, vomiting and further swallowing of
air. Fluid losses are isotonic, hence normal saline is the best
replacement fluid. Some degree of hypokalemia is the rule and
needs correction. When strangulation is suspected broad spectrum
antibiotic and blood transfusion are essential. Once patient is
adequately rehydrated and vital signs are stable, abdomen be
opened to relieve obstruction. The affected loop should be wrapped
DISEASES OF INTESTINES 67

in warm saline soaked packs and be inspected for colour,


mesenteric pulsation, and peristalsis. Viable bowel contracts on
pin prick and fluoresce under UV light when fluorescein is enjected
into peripheral vein. Gangreous loops be resected and end to end
anastomosis is made. When extirpation of obstructing lesion is not
possible anastomosis of proximal small bowel to distal small/large
bowel beyond obstruction be made. When adhesion is dense
enough to hinder mobilisation of intestine for anastomosis,
decompression gastrostomy may be done and anastomosis may
be attempted few months later.
After bowel surgery, stomach motility returns within 12-24
hours and intestinal motility within 48-72 hours. Hence, IV fluids
with nil orally be continued for 72 hours, the amount of fluid being
adequate to cause urine output of 1ml/minute. 1 bottle of Ringer
lactate, 2 bottles of normal saline and 2 bottles of 5 per cent daily
usually suffice for average adult. Hypokalemia delays return of
bowel motility and be especially looked for.

Volvulus
Volvulus is the rotation of segment of the intestine on an axis
formed by its mesentery. Volulus of the colon involves the cecum
in 50 per cent, sigmoid colon in 45 per cent, transverse colon in 3%
and splenic flexure in 2 per cent. In areas of high residue diet,
volvulus is the most frequent cause of large bowel obstruction.
Cecum is vulnerable becasue of poor embryogenic fixation.
Obstruction of lumen only occurs when the rotation is of 180°, when
it reaches 360°, veins are occluded, gangrne and perforation
supervene.
Cecal vulvulus causes severe interemittent colicky pain,
obstipation and feculent vomiting. Plain X-ray shows hugely
dilated ovoid cecum. In the initial stages there is a single fluid level
which is confused with gastric dilatation but gastric aspiration does
not alter the X-ray picture. In sigmoid volvulus there is generalised
lower abdominal distention. X-ray shows greatly distended bowel
loops with loss of haustral markings and of coffee bean shape. In
cecal volvulus the concavity points towards right lower quadrant
and in sigmoid vulvulus it points towards left lower quadrant. In
barium enema a bird’s beak deformity may be seen. Cecal volvulus
is common to female and sigmoid volvulus to male. Cecal volvulus
always occurs in clockwise fashion. Narrow attachment of pelvic
68 SHORT NOTES AND SHORT CASES IN SURGERY

mesocolon, long pelvic mesocolon, overloaded colon and adhesive


bands due to peridiverticulitis predispose to sigmoid volvulus.
Onset of sigmoid volvulus is always abrupt, often coming on when
patient is straining for stool.

Treatment
Colonoscopic derotation and decompression may be attempted in
sigmoid volvulus but not in those with strangulation and perfora-
tion. However, recurence is 50 per cent, hence laparotomy is the
ultimate answer. The loop is untwisted and the colon is deflated
by the rectal tube. If the bowel is viable resection is done and imme-
diate anastomosis is made in good risk patients, else ileostomy
(cecal volvulus) or colostomy is done and anastomosis is deferred
for a latter date. Suture fixation of cecum to parietal peritoneum
(cecopexy) prevents recurrence of cecal volvulus. Emergency
sigmoid colon resection for gangrenous volvulus has a mortality
of 30-40 per cent.
Volvulus of mid-gut occurs in neonates due to arrested
derotation, in which the cecum remains in left hypochondrium and
the transduodenal band of Ladd causes intistinal obstruction. The
onset is very rapid with vomiting, distention and dehydration.
X-ray shows the double stomach appearance due to distended
stomach and upper duodenum. Immediate laparotomy with
derotation and excision of the transduodenal band are essential.
Volvulus of stomach is often encounte where the stomach
rotates around its two fixed ends the cardia and pylorus. The
rotation can occur in vertical or horizonal direction. Most
commonly the greater curvature with the colon turn upwards to
lie under cupola of left diaphragm. Patient has epigastric pain and
retching and food intake is reduced. Barium meal is diagnostic
and surgery is the only alternative. The gastrocolic omentum be
completely divided to free the greater curvature which is fixed to
duodeno jejunal flexure. Any diaphragmatic hiatus if present be
closed.

Intussusception
Intussusception is invagination of one portion of gut into another,
immediately adjacent, usually proximal into distal. Very rarely it
can be retrograde also. A polyp, papilliferous carcinoma, sub-
mucous lipoma or inverted Meckel’s diverticulum are often
DISEASES OF INTESTINES 69

responsible. In infants is mostly idiopathic and occurs during


weaning. The last 50 cm of small intestine is involved which has
maximum aggregation of Peyer’s patches. Nearly in three fourth
of cases the intussusception is ileo-colic, in 12 per cent it is ileo-ileo
colic and in 5 per cent ileo-ileal. A definite cause can only be dis-
covered in half the patients. The outer tube is called intussuscpiens,
the inner and middle tube the intussusceptum. The blood supply
to inner tube is most likely to be impaired. Early gangrene occurs
in ileocecal form because of pressure exerted by ileocecal valve.
The typical patient is a male child of 6-9 months of age with
sudden paroxysms of abdominal pain and often vomiting. Each
pary xysm of pain is attended by facial pallor. The red currant jelly
stool occurs later. During pain one can feel the sausage shaped
lump to harden. If the intussusception travels far down, its apex
can be felt during per rectal examination. When mesentery is too
long, the intussusception may protrude through the anus.
Gradually the paroxysmal pain becomes continuous and acute
intestinal obstruction occurs in 24-36 hours. Barium enema shows
the “claw sign” in ileo-colic form.
It showld be differentiated from acute enterocolitis, and prolapse
of rectum. Intussusception in adolescence is nearly always caused
by inverted Meckel’s diverticulum.
Reduction of intussusception by hydrostatic pressure (saline or
barium) are unreliable. Hence operative reduction is best through
a right lower paramedian incision.

Paralytic Ileus
It is due to failure of neuromuscular mechanism affecting myenteric
plexus (Aurerbach) and the submucous plexus (Meissner). This
results in a collection of fluid and gas in the intestine with vomiting,
distention, absent bowel sound and failure to pass flatus. It may
follow generalised peritonitis, abdominal surgery, retroperitoneal
haemorrhage, fracture spine/rib, uraemia and hypokalemia. There
is no colic or abdominal pain. Percussion note is tympanitic. X-ray
shows gas filled dilated intestinal loops with multiple fluid levels.
Overflow of fluid from one intestinal segment so other often gives
to high pitched tinkling sounds.
Treatment is nasogastric suction, correction of precipitating
factors, maintenance of fluid and electrolyte balance, bowel rest
70 SHORT NOTES AND SHORT CASES IN SURGERY

and small dosen of morphine/pethidine to allay anxiety. Abdomi-


nal girth be measured at umbilicus every 4 hours to know of the
progression. As recovery occurs segmentation returns before
peristalsis. This may cause wind pains but is soon followed by
passage of flatus. When paralytic ileus is prolonged and threatens
life, laparotomy be performed and bowel be decompressed.

Intestinal Pseudo-obstruction
In pseudo-obstruction symptoms and signs of obstruction are
present in absence of organic obstructive lesion. Though mostly
idopathic, diabeties, scleroderma, myxedema, phenothiazines,
visceiral myopathy, radiation injury can precipitate it. Attacks are
recurrent and treatment is palliative.

Polyposis
Polyps can be sessile or pedunculated, benign or malignant,
mucosal, sub mucosal or muscular. Polyps are detected in routine
barium enema in 5 per cent of patients. About 50 per cent of polyps
occur in rectosigmoid junction and 50 per cent of sufferers have
more than one polyp.

Type Histology
Hamartoma Juvenile polyp
Peutz-Jigher’s polyposis
Inflammatory Benign lymphoid polyp
Inflammatory pseudopolyp
Neoplastic Tubular adenoma
Tubulo villous adenoma
Villous adenoma
Carcinoma
Unclassified Hyperplastic (metaplastic polyp)

Hyperplastic, hamartomatous and inflammatory polyps have


no malignancy potential. Malignancy potential of an adenomatous
polyp depends upon its size, growth pattern and degree of atypia.
Malignancy potential of tubular adenoma is 5 per cent, tubulo-
villous adenoma is 22 per cent and villous adenoma is 40 per cent.
Sessile lesions are most likely to become malignant. Cancer is found
DISEASES OF INTESTINES 71

in 45 per cent of adenomas bigger than 2 cm Tubular adenomas


account for 75 per cent cases of all neoplastic polyps. Most
adenocarcinomas of colon evolve from neoplastic polyps. Larger
the polyp, more likely for it to produce symptoms. Intermittent
rectal bleeding is common. Large villous adenomas produce
copious mucus. Double contrast barium enema as well as
colonoscopy can detect most polyps. Intermittent rectal bleeding
in children is often due to rectal polyp. Familial juvenile polyposis
coli is autosomal dominant and involves rectum, colon, and even
small bowel and stomach. Cancer developers before 40 years of
age in most potients, even though the polyps are hamartomatous.
Rectal bleeding, intussusception, protein loss may draw attention.
Total colectomy with mucosal proctectomy and ileoanal
anastomosis eliminates chances of cancer. Familial adenomatous
polyposis coli is also autosomal dominant where entire large bowel
is studded with polyps. Bleeding and diarrhoea are the presenting
symptoms. Nearly two out of three patients who present because
of symptoms have already developed cancer. Total proctocolec-
tomy, colectomy with ileorectal anastomosis, ileo anal anastomosis,
ileal rescrvoir/ileostomy bag/continent ileostomy are the options
for these patients. In Gardner’s syndrome, there is osteoma of
mandible and skull with inherited colonic polyposis. Risk of malig-
nant transformation of the polyps is very high. In Peutz Jegher’s
syndrome polyps occur in stomach, small bowel and colon with
pigmentation of buccal mucosa and lips. Malignancy potential of
polyps is very low.
Metaplastic polyps are small sessile pinkish in colour and are
harmless. Pseudopolyps are actually edematous bosses of colonic
mucosa in inflammatory bowel disease.
Polyps can be removed by snare during colonoscopy/sigmoido-
scopy. Small rectal polyp, the cherry red tumor can be avulsed by
finger but large soft velvety lesions in rectum are usually villous
adenomes with high malignancy potential. They need complete
excision under direct vision. No, rectal tumor should be removed
until pressibility of a proximeal carcinoma has been completely
ruled out.

Colon Cancer
Colon cancer is a major problem of prosperous societies. In USA it
is second most leading cause of cancer death. Six per cent of
72 SHORT NOTES AND SHORT CASES IN SURGERY

Americans will develop colon cancer and 40 per cent of them are
likely to die of the disease. High-fat and deficient fiber intake are
incriminated. Dietary fat enhances the bile acid systhesis and thus
increases level of sterols in the intestine. Colon bacteria convert
these sterols into secondary bile acids which are thought to be
carcinogenic. A fiber deficient diet reduces fecal bulk and thus
causes high concentration and prolonged contact of these
carcinogens. Most colorectal cancers are adenocarcinomas which
form bulky exophytic masses or annular constricting lesions.
Approximately half of the lesions are located in rectosigmoid
region, twenty per cent in ascending colon, 10 per cent in transverse
colon and and 15 per cent in descending colon. ten-to-fifteen per
cent of them are mucin producing with early distal spread and
hence poor prognosis.
Majority of colon cancers arise from malignant transformation
of adenomatous polyp especially if > 1 cm in size, multiple with
villous histology. Positive family history is present in 25 per cent
cases. Hereditary non-polyposis cancer colon has early onset,
proximal dominance. Ulcerative colitis and Crohn’s disease carry
cumulative risk of colon cancer of 5-10 per cent at 20 years and 20
per cent after 30 years of disease. Cholecystectomy and gastric
surgery increase the propensity for colon cancer. First degree
relatives of patients with colon cancer have 2-3 fold increased risk.
Adenocarcinomas grow slowly with doubling time of 130 days.
Thus it takes 5 years for the tumor to reach a size to produce symp-
toms. Cancers of right colon are exophytic and right colon being
more distensible, symptoms occur late. Unexplained microcytic
anaemia in elderly demands search for colon cancer. Lesions of
left colon are annular and because of small diameter of left colon
and solid fecal consistency obstructive symptoms develop early,
often with constipation alternating with diarrhoea. Rectal cancers
cause bleeding, tenesmus, urgency and sense of incomplete
evacuation.
Barium enema (double contrast) and colonoscopy can detect
most colon cancers. Fifty-to-seventy per cent colon cancers are even
within the reach of 60 cm flexible sigmoidoscope. Intra-rectal US
and MRI can provide informatioan about depth of invasion of rectal
wall and involvement of pararectal lymphnodes. CEA is raised but
has no diagnostic value. If it fails to fall after surgery or rises after
brief fall, incomplete removal/recurrence can be thought of.
DISEASES OF INTESTINES 73

Treatment is by wide resection with regional lymphnode


dissection for staging which guides about adjuvant therapy.

Duke’s Staging
Stage Extent 5 year survival
A Lesion limited 100 per cent
to mucosa
B1 Invades muscularis 60 per cent
propria, negative nodes
B2 Invades subserosa 53 per cent
negative nodes
C Bowel wall penetration
1-3 pericolic nodes positive 42 per cent
C2 4 or more pericolic nodes positive 22 per cent
Adjuvant therapy with 5FU, irinotecan and pelvic radiation is
given in node positive patients.

Prevention of Colon Cancer


Prolonged regular use of NSAID or aspirin > 325 mg twice weekly
causes 30-50 per cent decrease in incidence of colon cancers.
Calcium carbonate 8 gm/day and folate therapy also modestly
reduce the risk of colorectal cancer. Vitamin E is similarly helpful
but not other antioxidants. Fiber supplement, low meat and low-
fat diet with plenty of vegetables and fruits may be helpful.
Sensitivity of double contrast barium enema is 70-90 per cent
for polyps > 1 cm and 55-85 per cent for early colorectal cancer.
However CT colonography has better sensitivity for detection of
small polyps and early colon cancer.
10
Diseases of Breast

Fibrocystic Disease
It is the most frequent breast lesion in women of 30-50 years of age
probably induced by estrogen. Histologically, there are cystic
changes, papillomatosis, adenosis, fibrosis and ductal epithelial
hyperplasia. Painful, often multiple, usually bilateral masses in
breast are common so also quick floctuation in size of the masses.
Pain and size of mass characteristically increase before menstrua-
tion. There may be nipple discharge. FNAC is diagnostic and
confirms cystic nature of mass. If the mass continues to persist,
obtained fluid is bloody or no fluid is obtained, it be excised.
Danazol 100-200 mg twice daily and vitamin E 400-600 mg daily
have been tried with variable success. Chances of malignancy is
high when breast epithelicum shows atypia or proliferation.

Fibroadenosis
It is a round or avoid rubbery, discrete, relatively mobile non-tender
mass 1-5 cm in diameter, common to young women, often multiple
(in 10-15 percent). FNAC is diagnostic. Excisional biopsy is required
when FNAC diagnosis is uncertain.

Phyllodes Tumor
It is a fibroadenoma like tumor with cellular stroma that grows
rapidly to attain large size. It can be benign or malignant. Hence
mastectomy is safe but when benignity is certain local excision
with a safety margin is adequate. Lymphnode dissection is not
required since when it undergone sarcomatous changes, it
metastasizes to lungs, not axillary nodes.
DISEASES OF BREAST 75

Fat Necrosis
Fat necrosis produces a mass with nipple/skin retraction mimic-
king carcinoma. History of trauma is only present in half the cases.
Needle biopsy is adequate.

Cancer Breast
The risk of breast cancer increases with age. Mammary dysplasia,
early menarche (< 12 years), late menopause (> 50 years), late first
pregnancy, BRCA1 or BRCA2 mutation, mulliparity and history of
breast cancer in mother and sister are the risk factors. The relatively
long preclinical growth phase, and the tendency of breast cancer
to metastasize much before clinical recognition, demand effective
screening and mass awareness of the disease. Oral contraceptives
donot increase its risk so also HRT in postmenopausal women. In
over 90 percent cases the neoplasm arises from ductal epithelium.
Except the insitu cancers, histologic subtype (colloid, medulary,
schirous) have no bearing on prognosis. However most patients
donot have an identifiable risk factor.
The usual presentation is with a painless breast lump. Breast pain,
nipple discharge, erosion and retraction, itching and shrinking of
breast are less common. The mass is firm to hard in consistency
with poorly delineted margins. Skin puckering and pea-u-de
orange occur late due to skin infiltration. 60 percent of lesions occur
in upper outer quadrant, 15 percent each in upper inner and lower
outer quadrant, 5 percent in lower inner graduant and 5 percent
around the areola. Lesions < 1 cm may be difficult to feel, yet can
be discovered by the patient. Microscopic metastasis is present in
axillary nodes in 30 percent cases with clinically negative nodes (<
1 cm in size, mobile). Woman with cancer in one breast has risk to
develop cancer in other breast at 1-2 percent per year.
Clinical judgement of breast cancer is fallacious since 60 percent
of lesions clinically thought to be cancer turn out to be benign on
biopsy and 30% believed to be benign turn out to be malignant.
Hence FNAC is mandatory. Mammography is most useful for
detection of early breast cancer and can diagnose 2 years prior to
its clinical recognition. Clustered polymorphic microcalcification
is the diagnostic clue particularly when having V, Y or branched
configuration. Mass when present has irrigular illdefined borders.
Ultrasound only differentiates cystic from solid lesion. A cyst in
76 SHORT NOTES AND SHORT CASES IN SURGERY

US yielding non-bloody liquid is surely benign. All non-palpable


mammographic densities be examined by US to determine if cystic
or solid. US can guide needle biopsy too. Open biopsy is performed
in highly suspicious lesions in OT, and if frozen section is positive,
definitive surgery is done immediately or within 1-2 weeks. 99mTc
phosphate or phosphonate bone scan and chest X-ray are essential
to exclude metastasis. Hypercalcemia is occasional. CEA, CA 15-3,
CA 27-29 may be used as markers for recurrent disease. MR
mammography has better yield than X-ray mammography.
Most breast cancers (80-90%) arise from epithelium of
intermediate sized ducts or from epithelium of terminal ducts
(lobular—6-8%).

Paget’s Carcinoma
The basic lesion is usually an infiltrating well differentiated ductal
carcinoma or carcinoma in situ involving ducts of nipple. The first
symptom is itching and burning of nipple with erosion and
ulceration. Tumor mass is usually not palpable. Biopsy of the erosion
yields the diagnosis. Lesion is frequently mistaken for dermatitis
or bacterial infection. When lesion consists of nipple changes only
axillary node metastasis is < 5 percent and prognosis is excellent.
When mass is also palpable chances of axillary node metastasis is
high with poor prognosis. However Paget’s cancer constitutes only
1 percent of all breast cancer.

Inflammatory Carcinoma
It is constitutes 3 percent of all breast cancers and is the most malig-
nant. It presents as a painful rapidly growing mass. The overlying
skin is warm, erythematous and edematous. Often there is no
distinct mass as the tumor infiltrates the breast diffusely. Metastasis
occur, early and widely.
Breast cancer can complicate pregnancy and the diagnosis is
delayed due to physiologic changes in the breast that obscure the
lesion. Pregnancy however is not a contraindication for operation.
Bilateral breast cancer occurs in 1 percent and is mostly familial.
Non-invasive breast cancer (ductal carcinoma in situ (DCIS) and
lobular carcinoma in situ (LCIS) account for 4-6 percent of all breast
cancers. While 40-60 percent cases DCIS develop invasive cancer
later in same breast, 20 percent cases LCIS develop invasive cancer
DISEASES OF BREAST 77

in either breast. Tamoxifen prophylaxis in the both the forms can


prevent development of invasive cancer.

Receptor Status
Knowledge about the receptor status of tumor cells is essential for
planning adjuvant therapy and treatment of advanced disease.
Upto 60 percent of patients with estrogen receptors and advanced
metastatic disease will respond to hormonal manipulation.
Similarly 80 percent of patients with metastastic progesterone
receptor positive tumors improve with hormonal manipulation.
Receptors donot bear any relationship to resposne to chemo-
therapy. Receptor status may change after radiotherapy, chemo-
therapy. Hence estrogen progesterone receptor assay be done for
every breast cancer at the initial diagnosis.

Staging
Tis Carcinoma in situ, DCIS, LCIS, Paget’s disease of
nipple without mass.
T1a Tumor > 0.1 cm but < 0.5 cm in greater dimension
T1b Tumor > 0.5 cm but < 1 cm in greater dimension
T1c Tumor > 1 cm but < 2 cm in greater dimension
T2 Tumor > 2 cm but < 5 cm in greater dimension
T3 Tumor > 5 cm in greater dimension
T4 Extension to chest wall, edema, ulceration of skin,
peaud orange, satellite nodule or inflammatory
carcinoma.
N1 Ipsilateral axillary nodes involved but mobile
N2 Ipsilateral axillary nodes involved but fixed
N3 Metastasis to ipsilateral internal mammary nodes.
Stage I T1, N0, M0
Stage IIA T0, N1, M0; T1 N1 M0; T2 N0 M0
Stage IIB T2 N1 M0, T3 N0 M0
Stage IIIA T0 N2 M0; T2 N2 M0, T3, N1/N2, M0
Stage IIIB T4; Any N M0; Any T1 N3 M0
State IV Any T/N with metastasis

Treatment
Treatment may be curative or palliative. Curative treatment is
advised for Stage I and Stage II disease; palliation is for Stage III
78 SHORT NOTES AND SHORT CASES IN SURGERY

and IV disease. Partial mastectomy plus axillary node sampling


followed by radiotherapy is as good as modified radical mastelc-
tomy followed by adjuvant therapy if nodes are positive. After
surgery chemotherapy or hormone therapy is advocated for most
patients to eliminate micrometastasis. CMF regimen (cyclophos-
phamide, 600 mg/m2 IV on day 1 and 8, methotrexate 40 mg/m2
IV on day 1 and 8, and 5FU 600 mg/m2 IV on day 1 and 8) every 4
weeks for 6 months improves survival in node positive premeno-
pausal women. Adriamycin and taxanes (palcitaxe, docetaxel) when
used provide better survival. ER positive postmenopausal women
do better when given tamoxifen 10 mg bid. Premenopausal women
who are ER negative be given chemotherapy irrespective they have
mxillary netastasis or not. Of course if they are ER positive with no
involvement of axillary nodes tamoxifen is the alternative.
Postmenopausal women with negative nodes need not receive
chemotherapy irrespective of receptor status. In practice most
patients are advised adjuvant chemotherapy irrespective of node
status.
Radiotherapy consists of 5000-6000 rads delivered over a month
in 4-5 sittings per week. Postoperative radiotherapy prevents local-
regional recurrence. With 4 or more positive nodes local-regional
recurrence is 15-25 percent and radiotherapy reduces it to 5 percent.
However overall survival is not increased by radiotherapy. Should
a patient who had adjuvant chemotherapy has local recurrence,
radiotherapy then helps in controlling lesion in 60-70 percent cases.
Postoperative radiotherapy should be considered if tumor is > 5
cm, undifferentiated/inflammatory histology.

Local Recurrence
The incidence of local recurrence correlates with tumor size, number
of positive axillary nodes, histology of tumor and its fixation to
deeper tissue/skin. Without axillary node involvement it is only 5
percent. With involvement of supraclavicular or internal mammary
nodes rodiation is the only answer. Local recurrence should always
demand search for bone/visceral matastasis. Edema of hand
follows in 10-30 percent cases following radical / modified radical
mastectomy plus axillary irradiation.

Prognosis
Two-third of patients eventually have distant disease irrespective
of form of primary therapy. Recurrence is high in:
DISEASES OF BREAST 79

1. T2 T3,
2. Negative receptor status.
3. Higher histologic grade,
4. > 5 percent cells in `S’ phase,
5. Presence of lymphatic/vascular invasion
6. High-levels of cathepsin D, HER2/neu oncogene, and epider-
mal growth factor.

Palliative Treatment
Palliative radiotherapy may be advised for locally advanced disease
with distant metastasis in order to control pain, and ulceration and
lymph node symptoms. A small number of patients can be cured
with radiotherapy. Radiotherapy is also useful in isolated bone
metastasis, chest wall recurrence, acute spinal lord compression
and brain metastasis. Hormonal therapy/manipulation is more
helpful in postmenopausal women. It consists of tamoxifen
(antiestrogen) 10 mg twice daily; diethyl stilboestrol (estrogen) 5
mg three times daily; megestrol acetate (progestin) 40 mg 4 times
daily, and aminoglute thimide 250 mg 4 times daily. Response rate
is 60 percent if ER is positive and upto 80 percent if progesterone
receptor is positve as well. Tamoxifen is the agent of choice in
premenopausal women but those who donot respond may undergo
oophorectomy. If none is helpful other methods like aminoglute
thimide (medical adrenalectomy) or progestins may be tried. Post
menopansal women who donot respond to tamoxifen, need
cytotoxic drugs (CMF regime) or other hormonal manipulation.
Anastrozole 1 mg daily has proved effective when hormonal
treatment is unhelpful, and tumor is ER negative; adriamycin/
taxanes are also to be considered. High dose chemotherapy and
autologous bone marrow/stem cell transplantation is drawing
attention for metastatic breast cancer. Trastuzumab, a monoclonal
antibody binding to HER2/neu receptor on cancer cells may be tried.

Survival
5 year survival in:
Stage I 85 percent
Stage II A 70 percent,
Stage IIB 60 percent,
80 SHORT NOTES AND SHORT CASES IN SURGERY

Stage IIIA 55 percent,


Stage IIIB 30 percent and
Stage IV 5-10 percent

Prevention
Raloxifene, a selective estrogen receptor modulator with anti-
estrogen effect on breast and uterus and estrogenic effect on bone
and lipids, 60 mg daily when taken for two and half years by post
menopausal women reduced breast cancer risk by 70 percent.
Chemo-preventive role of tamoxifen (antiestrogen and antiproges-
terone) is less clear.
11
Diseases of Prostate

Prostatic Hyperplasia
Benign prostatic hyperplasia involves periurethral zone of prostate
and all the three elements, i.e. epithelium, smooth muscle and
collagen increase in number. The hyperplastic nodules compress
outer zones of prostate, resulting in formation of surgical capsule.
This boundary separates the transition zone from peripheral zone
and serves as the cleavage plane for enucleation of prostate. BPH
is the most common benign tumor in men. 50 percent of men
between 51-60 years have it and incidence increases to 90 percent
in men over 80 years. Ofcourse symptoms of urinary obstruction
are less frequent. 25 percent of men at 55 years have some obstruc-
tive voiding symptoms which increases to 50 percent at 75 years of
age.
The disease is multifactorial and is under endocrine control.
Decreased apoptosis may be a factor. Castration and administration
of LHRH analogs shrink established BPH. Falling testosterone levels
of aging with increasing estrogen level may play a role.
The symptoms can be obstructive and irritative. Obstructive
symptoms include hesitancy, decreased force and caliber of stream,
sensation of decreased bladder emptying, straining, postvoid
dribbling. Irritative symptoms include urgency, frequency and noc-
turia. The obstructive component can be mechanical and dynamic.
The latter is due to increased prostatic smooth muscle with enhan-
ced adrenergic tone. Prostatic size by digital rectal examination
poorly correlates with the symptoms.
BPH usually results in a smooth, firm, elastic enlargement of
prostate. The rectal mucosa freely glides over the gland and any
induration demands exclusion of prostatic malignancy. Many
patients present with acute retention of urine or haematuria. The
bladder is hypertrophied and increased residual urine may lead to
82 SHORT NOTES AND SHORT CASES IN SURGERY

stone formation and infection. When median lobe is only enlarged


digital rectal examination may be normal. IVP, sonography and
cystoscopy confirm bladder changes secondary to outlet obstruction
(detrusor hypertraphy, diverticula, vesico-ureteric reflux, hydro-
nephrosis).
Other obstructive conditions of lower urinary tract like urethral
stricture, bladder neck contracture, bladder stone and prostatic
malignancy be excluded. PSA study, urodynamic studies and
cystometrograms are helpful in this regard. Carcinoma in situ of
bladder and patients with neurogenic bladder can have similar
presentation.

Treatment
Treatment is medical or surgical. With mild to moderate symptoms,
not much of residual urine and absence of significant detrussor
hypertrophy one should resort to medical therapy. Alpha 1 blockers
like prazosin 1-5 mg twice daily, terazocin 1-10 mg daily, doxazosin
1-8 mg daily or tamsulosin 0.4-0.8 mg daily are effective as they
reduce prostatic smooth muscle tone. 5 alpha reductase inhibitor -
finasteride 5 mg daily can be helpful in 60-80 percent cases. 20
percent reduction in prostate size is excepted at 6 months of therapy.
Several plant products are also used to shrink although their safety
and efficacy is not known (speman, prostina) Transurethral resection
of prostate (TURP) is most popular but the risks include retrograde
ejaculation (75%), impotence (5-10%) and urinary incontinence
(< 1%). Complications include bleeding, bladder neck contracture,
urethral stricture and TURP syndrome, a hypervolemic
hypernatremic state resulting from absorption of irrigating fluid
especially when resection time exceeds 90 minutes. Transurethral
incision of prostate (TUIP) is less traumatizing and is reserved for
small enlargement causing elevation of bladder neck retrograde
ejaculation is low. Trans-urethral needle ablation (TUNA) employs
radiofrequency to cause coagulative necrosis but bladder neck and
median lobe enlargements are not effectively treated by this method.
Transurethral laser induced prostatectomy (TULIP) under transrec-
tal US guidance is another option. Transurethral electro-vaporiza-
tion of prostate high intensity focussed ultrasound and microwave
hyperthermia are some other minimally invasive procedures for
BPH. Transurethral balloon dilatation and stent placement are often
practised in treating BPH.
DISEASES OF PROSTATE 83

Open prostatectomy is required when prostate is too big (> 100


gm). It can be done by suprapubic or retropubic approach.

Carcinoma of Prostate
Ninety-five percent of prostatic cancer is adenocarcinoma. There
is 60 percent increase in prostatic cancer in vasectomised males.
Unlike BPH which arises from periurethral zone, most of carcinoma
(75 %) arises in peripheral zone, only 10 percent have bladder outlet
obstruction and urinary obstruction. Fifty percent of men present
with symptoms of metastasis. A hard nodular prostate with
adherent rectal mucosa on PR examination commonly draws
attention towards possibility of cancer. The incidence of prostatic
cancer increases with age. Where as 30 percent of men between 60-
70 years will have the disease at autopsy, it rises to 67 percent in
men of 80-90 years.
Serum acid phosphatase is increased in 75 percent of patients
with bone metastasis. Those without elevated acid phosphatase
levels have undifferentiated cancer. PSA level is increased in 60
percent of cases: PSA level correlates with volume of prostatic tissue
in BPH and prostate cancer. Measurement of PSA is useful in
(1) detecting prostate cancer (2) staging prostate cancer
(3) monitoring response to treatment, and (4) and detecting
recurrence before it is clinically evident. Normal PSA level is < 4
ng/ml. When level is 4-10 ng/ml, the cancer is mostly localised
and hence potentially curable. However abnormal PSA value has
only a sensitive rate of 67 percent. With bone metastasis levels of
alkaline phosphatase and calcium may increase. Transrectal US
guided prostatic biopsy is more informative, the samples taken from
apex, middle and base. The seminal visicles can be biopsied to know
of local invasion. Transrectal US and MRI are also used to see for
lymphnode, seminal vesicle invasion. Bone scan is a must to exclude
skeletal metastasis. PSA velocity is more informative than absolute
PSA values in suspicion of prostate cancer. PSA velocity > 0.75 ng/
ml A year is associated with more likelyhood of cancer detection.
Since carcinoma of prostate usually arises in external group of
glands, minimally invasive procedures for BPH do not provide
protection against future development of prostate cancer.
Carcinoma arising in lateral lobes involve prostatic urethra early.
Growth arising in posterior lobes tends to grow upwards to involve
seminal vesicles as backward extension is prevented by strong fascia
84 SHORT NOTES AND SHORT CASES IN SURGERY

of Denonvilliers. The frequent skeletal osteoblastic metastasis of


prostate cancer has been attributed to reverse flow from vesical
venous plexus to the emissary veins of pelvic bones during
straining, coughing and sneezing. Prostate cancer particularly
metastastic form is known to cause bleeding from DIC. Bilateral
sciatica in elderly or paraplegia or spontaneous pelvic/hip fracture
should arouse suspicion of prostate cancer.

Treatment
It is to be stressed that only 5 percent of tumors is localized to
prostate at the time of diagnosis; fifty percent have local spread
and in 40 percent metastasis is already present. Radical prosta-
tectomy (removal of prostate, seminal vesicles and ampullae of vas)
is indicated for Stage T1 and T2 (tumor confined to prostate without
capsular involvement) disease, followed by radiotherapy. Patients
with locally advanced disease can be treated with surgery or
radiation combined with androgen deprivation. Metastatic disease
is treated by androgen deprivation (estrogen 1-3 mg daily, LHRH
agonists IM monthly, ketoconazole 400 mg tid; aminoglutethimide
250 mg qid, orchiectomy, and anti androgens - flutamide 250 mg
tid or bicalutamide-50 mg daily). Hormone refractory patients may
be tried with suramin.

Bladder Neck Contracture


It is due to congenital muscular hypertrophy, fibrosis of tissues
around bladder neck or an aftermath of chronic prostatitis/
vulvovaginitis. Symptoms of bladder outlet obstruction with
progression to hydronephrosis and recurrent pyelonephritis.
Intermittent dilatation may be helpful in early disease but most
patients require transurethral resection or an open operation.

Chronic Prostatitis
The predominant organism is E.coli. Smears show bacteria in 40
percent but culture is positive in 70 percent cases. Chlamydia is
responsible in many. Clinical picture is varied. Dull ache in
perineum or rectum is common, increased on sitting on a hard chair.
Referred pain may be felt in the back or in the legs. It may cause
posterior urethritis with burning micturition. Silent prostatitis may
continue with distant manifestations like arthritis, myositis, neuritis,
DISEASES OF PROSTATE 85

conjunctivitis, iritis, etc. Sexual dysfunction in the form of premature


ejaculation and impotence are not uncommon.
Discovery of prostatic threads in urine, and pus cells in prostatic
fluid obtained after prostatic massage is diagnostic. In PR examina-
tion prostate feels often boggy. Urethroscopy may reveal inflammed
posterior urethra; enlarged and edematous verumontonum. Bactrial
prostatitis responds to TMP-SMX, quinolones given for 4-6 weeks.
Chlamydial infection responds best to tetracycline/erythromycin.

Prostatodynia
Though the symptoms mimic chronic prostatitis, the prostate is
normal. It affects young and middle aged men. Voiding dysfunction
or pelvic floor muscle dysfunction are often responsible. Prolonged
voiding difficulty is usual. Urodynamic studies may show detrusor
contraction without urethral relaxation, high urethral pressure,
spasm of urinary sphincter. Alpha blockers like doxazosin 1-8 mg
daily may be of help. Pelvic floor dysfunction may respond to dia-
zepam.
12
Malignancies of Urinary Tract

Hypernephroma (Grawitz tumor)


It is adenocarcinoma arising from proximal tubule cells. Various
cell types (clear, granular, spindle) and histologic patterns (acinar,
papillary, solid) are seen but they donot affect treatment. Peak
incidence is in sixth decade with male-female ratio of 2:1. Sporadic
tumors are more common but cigarette smoking and von Hippel
Lindau syndrome are the risk factors. Tumor is spherical in shape
occupying one or other pole, upper pole being more common. On
section it is characteristically yellow with haemorrhagic areas. As
it grows it spreads to renal veins. Haematologic metastasis to lungs
(canon ball) and bones is common. When it involves renal capsule
lymphnodes in renal hilum and para-aortic group are involved.
60 per cent of patients present with gross or microscopic
haematuria. Flank pain and abdominal mass are present only in
30 per cent cases. Twenty-thirty per cent present with bone pain
and haemoptysis due to metastasis. Polycythemia and prolonged
pyrexia may occur in few. Some have hypertension due to renin
release white others have nephrotic syndrome CT scan is the best
modality for evaluating renal cell carcinomas. MRI and Doppler
can detect invasion of renal vein.
All solid masses in kidney seen in US are renal cell carcinoma
unless proved otherwise. Other solid masses include angio-
myolipoma (fat density in CT), transitional cell carcinoma in renal
pelvis (more centrally located), oncocytoma (preoperative
diagnosis difficult). However 75 per cent of renal tumors are only
renal cell carcinoma.
Radical nephrectomey is the primary treatment for localized
tumor (confined to renal capsule) with 5 year survival of 90-100
per cent. When tumor extends beyond renal capsule with radical
nephrectomy and radiotherapy 5 year survival is 60 per cent. When
MALIGNANCIES OF URINARY TRACT 87

there is distant metastasis-5 year survival drops to 0-15 per cent.


When metastases is single and resectable, radial nephrectomy with
resection of mitastasis achieves 5 year survival of 15-30 per cent.
No, effective chemotherapy is available for metastatic renal cell
carcinoma except viablastin which has short term partial response
rate of 15 per cent. Alfa interferon and IL2 may be tried in these
patients. Tumor vacine and gene therapy are the hopes for future.
Oncocytomas account for 3-5 percent of renal tumors and can not
be distinguished from renal cell carcinoma by imaging. They also
occur in adrenals, salivary glands and in thyroid, parathyroid
glands. Angiomyolipomas contain fat, blood vessels and smooth
muscles, common to patients of tuberous sclerosis. Fat component
in CT is diagnostic. Asymptomatic lesions < 5 cm in diameter donot
require intervention.

Bladder Cancer
More than 95 per cent bladder tumors arise from mucus membrane,
the rest being angioma, myoma, fibroma, endometrioma. Though
efiology is unknown in most cases, benzidine and many other dyes
are incriminated so also bilhirziasis, chronic stone disease and
catheter placement for long. A villous papilloma of bladder has
finger like projections from a single central stalk or is diffuse with
multiple small growths over an area. Low grade transitional cell
carcinomas (malignant villous tumor) also resemble villous
papilloma but the villi are stunted resembling cauliflower, stalk is
small or often sessile, and lymphatic nodules appear around the
growth. Bladder tumor accompanied by cystitis is nearly always
malignant. The bladder wall adjacent to malignant tumor is
edematous and the tumor may ulcerate. Invasion of bladder wall
gives rise to regional and distant metastasis. Solid tumors are
always malignant and grow rapidly through the bladder wall. The
tumor is deep red, lobulated and bleeds on touch. Lymphatic meta-
stasis occurs earlier than malignant villous tumors. The third form
of bladder cancer is a carcinomatous ulcer occasionally arising from
a patch of leukoplakia. It is more common in base of bladder and
trigone. This is the most malignant form. Thirty-five per cent of
bladder cancers occur in lateral wall, 32 per cent in trigone, 10 per
cent in posterior wall and 10 per cent in bladder neck. Carcinoma
not involving serosa/peri vesical fat has no metastasis but may
88 SHORT NOTES AND SHORT CASES IN SURGERY

recur after removal. Adenocarcinoma and squamous cell carcinoma


account for 7-9 per cent of all bladder cancers.
Painless profuse, paroxysml haematuria is the only symptom
in papilloma. Malignant neoplasms also present with intermittent
haematuria which eventually becomes continuous. In carcinoma-
tous ulcer the symptoms are often those of acute cystitis with
painful frequent and blood stained urine. Strangury occurs at the
end of each act. Cystoscopy is the main stay of diagnosis but US is
equally reliable as a screening procedure.
Cystodiathermy through cystoscope is widely employed to
remove benign or low grade malignant tumors. Excision with a
resectoscope can be used when tumor is very large. Intravesical
chemotherapy with thiotepa, mitomycin, doxorubicin and BCG are
equally effective. Muscle infiltrating cancers may require partial
cystectomy. Radical cystectomy entails removal of bladder,
prostate, seminal vesicles with bilateral pelvic node dissection and
removal of uterns, ovaries and anterior vaginal valt in women.
Radiotherapy can be tried in fractions over 6-8 weeks but 10-15
per cent develop bladder, bowel and rectal complications.
Cisplastin based combination chemotherapy may be employed
when tumor is far locally advanced and is unresectable (extension
to pelvic wall) but response rate is 15-30 per cent. It can be used
prior to surgery or after it in locally advanced disease (involvement
of perivesical fat and lymphnodes).

Wilm’s Tumor
Nephroblastoma accounts for 80 per cent of malignant renal
neoplasms in children, the rarer ones being anaplastic sarcoma,
clear cell sarcoma and rhabdoid tumors which all carry poor
prognosis. Associated anomalies with Wilm’s tumor include
aniridia, hemihypertrophy, crypseorchidism, hypospadius and
neuro-fibromatosis. The tumor is large, firm, smooth and does not
extend beyond midline. Hypertension is present in half the cases.
Pain, haematuria and weight loss only occur in 15-20 per cent cases.
Calcification is present in only 10 per cent, is cresent shaped and
peripherally placed, in comparision to neuroblastoma where it is
finely stipled. IVU shows distorsion of calyces. CT and US show
the tumor and lung metastasis is common.
MALIGNANCIES OF URINARY TRACT 89

It should be differentiated from adrenal neuroblastoma which


crosses midline, always symptomatic and metastasizes early to
lungs. Wilm’s tumor is radiosensitive and responsive to
chemotherapy. Immediate nephrectomy be performed with
removal of renal and para aortic lymphnodes. The other kidney
should be explored since bilateral disease is known. Adjuvant
chemo radiotherapy (actinomycin-D, vincrystine) offers improved
results.
Short Cases

Case 1

A 25-year-old male complains of swelling and heaviness in scrotum


for one month. Examination reveals cystic non-tender swelling of
both testicles with positive transillumination and fluctuation tests.
1. What is your diagnosis?
Hydrocele of tunica vaginalis testis.
2. What is hydrocele?
Accumulation of free serous fluid within processus vaginalis,
commonly tunica vaginalis testis.
3. Types of hydrocele
1. Congenital
2. Acquired, which is
a. primary or idiopathic
b. secondary to diseases of testis like trauma, epididymo-
orchitis (filarial) tumor or lymphatic obstruction.
4. Cause of primary hydrocele
It is thought that an imbalance between secretory and
absorptive capacities of layers of tunica vaginalis is
responsible for collection of fluid within the sac. Primary
hydrocele usually develops slowly and becomes large and
tense. Four anatomical varieties of congenital hydrocele are
encountered—vaginal hydrocele, infantile hydrocele,
encysted hydrocele of cord and congenital hydrocele.
5. Characteristic of hydrocele fluid
The fluid is amber coloured with specific gravity of 1022 to
1024. It contains 6 percent albumin, inorganic salts and
fibrinogen. It does not clot unless mixed with blood.
6. Description of different varieties of congenital hydrocele
1. Congenital Hydrocele Proper. Here processus vaginalis
remains patent throughout communicating with abdominal
cavity, but the communicating orifice at inguinal ring is
too small for development of a hernia. The scrotal swelling
CASE – 1 91

disappears when patient lies flat, becoming more promi-


nent on standing and walking. This entity is very rare.
2. Infantile Hydrocele. Here fluid collection is upto internal
abdominal ring but the sac has no connection with
peritoneal cavity. It does not necessarily appear in infants.
3. Funicular Hydrocele (encysted hydrocele cord). It is a smooth
oval swelling confined to cord, often mistaken for
irreducible inguinal hernia.
4. Vaginal Hydrocele. It is the most common form of congenital
hydrocele, where fluid accumulation is limited to tunica
vaginalis testis.
7. Charecteristics of vaginal hydrocele
1. Swelling is scrotal i.e. one can get above the swelling.
2. Testis can not be felt separately.
3. Swelling is not reducible and is uniformly cystic.
4. Transillumination positive.
8. Complications of hydrocele
1. Rupture, usually traumatic but less commonly spontaneous.
2. Haematocele.
3. Infection
4. Atrophy of testis
5. Hernia of hydrocele sac through dartos muscle, usually in
oldstanding cases
6. Calcification of sac.
9. Operations available for vaginal hydrocele
1. Lod’s operation—Everted tunica is bunched into a ‘ruff at
the edge of testis by interrupted sutures; done mainly for
small hydroceles.
2. Jhawer’s and Sharma’s operation—Placement of delivered
testis into fascial layers of scrotum
3. Jaboulay’s operation—Eversion of sac behind testis and
epididymis and placement of continuous suture.
4. Tapping—done in patients for prompt relief of discomfort,
as an alternative to surgery in large hydroceles and poor
surgical candidates.
When hydrocele sac is thick as in secondary hydrocele due to
repeated attacks of filarial epididymitis, the large and thick
tunica vaginalis be excised, along with exuberant scrotal
coverings.
92 SHORT NOTES AND SHORT CASES IN SURGERY

10. Other forms of hydrocele


1. Hydrocele of canal of Nuck occurs in females in relation to
round ligament. The cystic swelling lies wholly or partially
within inguinal canal.
2. Hydrocele—en Bisac—Here the hydrocele has two inter
communicating sacs, one above and one below the neck of
the scrotum,
3. Post-herniorrhaphy—This is due to damage to lymph
channels of tunica vaginalis passing along spermatic cord.
4. Hydrocele of hernial sac.
11. Role of ultrasound in management of hydrocele
An ultrasound is a must in small hydroceles developing insi-
diously in older persons, in order to exclude testicular tumors.
12. Characteristics of filarial hydrocele
Frequently, they are bilateral and the fluid contains chyle, rich
in cholesterol derived from rupture of lymphvarix into the
tunica. In long-standing chyloceles, dense adhesions form
between the scrotum and its contents. Elephantiasis of
scrutum and penis occur in some cases.
13. Diagnosis of haematocele
1. History of testicular trauma, lapping of hydrocele.
2. Tense, tender painful swelling.
3. Poor or absent transillumination.
14. Why all haematoceles be explored?
To exclude testicular rupture due to antecedent trauma or
presence of occult tumor.
15. Can haematocele be painless?
Yes, slow spontaneous non-traumatic bleed into tunica, causes
clotted haematocele which simulates a neoplasm of testis.
16. Other cystic swellings of scrotum
• Cysts of epididymis
• Spermatocele.
17. Origin of cyst of epididymis
The cyst forms due to cystic degeneration of:
• The paradidymis (organ of Giraldus)
• The appendix of epididymis (hydatid of Morgagni)
• The vas aberrans of Haller.
CASE – 1 93

18. Characteristics of cysts of epididymis


They occur in middle age, often bilateral, tense and give the
feeling of a tiny bunch of grapes. They contain crystal clear
fluid and are brilliantly translucent. Being multilocular their
aspiration is useless but excision is attended by infertility due
to blockage of sperm passage.
19. Characteristics of spermatocele
Spermatocele is a unilocular retention cyst of epididymis,
containing barley water like fluid. It is situated above and
behind the body of testis and contains spermatozoa. It is
translucent but softer. Excision is only required in larger
symptomatic form.
20. Causes of acute epididymitis
In young men, it is due to chlamydia and gonococcus but in
elderly, it is due to prostatitis or urinary tract infeclion.
Mumps, many viruses, filariasis, urethral instrumentation,
retrograde passage of urine along vas, post-prostatectomy are
few other causes. Blood borne infection affects globus major
first unlike infection reaching along vas, that affects globus
minor first. Streptococcum, staphylococcum, proteus and E.
coli are the villains of blood borne epididymitis.
21. Features of chronic epididymitis
Ninety per cent of these cases are of tuberculous origin,
commonly rectrograde from infected seminal vesicle. The epi-
didymis feels as a firm, nodular, craggy painless mass. Rectal
examination reveals a thick irregular indurated seminal
vesicle on affected side. Two-third patients have associated
tubercular infection of urinary tract.
Non-tuberculous chronic epididymitis following acute attacks
feels smoother and is larger and is most often secondary to
urethral strictures.
22. What is varicocele?
It is a condition of dilatation, elongation and tortuosity of
pampiniform venous plexus.
23. Venous drainage of testis
The left testicular vein drains into left renal vein at right angle,
while the right testicular vein drains into inferior vena cava
94 SHORT NOTES AND SHORT CASES IN SURGERY

at an acute angle. The cremasteric veins and veins accom-


panying the vas anastomose with testicular veins, thus
providing an alternative pathway for venous drainage from
testis and epididymis. The cremasteric veins drain into
inferior epigastric vein.
24. Aetiology of varicocele
Ninety-five per cent of varicocele occurs on left side due to
reflux from left renal vein. The left testicular artery often
arches over left renal vein to obstruct it. Often the left testicular
vein may be obstructed due to growth of hypernephroma into
renal vein. Often the dilated vessels are veins of cremasteric
plexus, not the pampiniform plexus.
As varicocele is a problem in young adults, probably
testicular congestion due to sexual stimulation and consequent
increased venous drainage is contributory. Loaded pelvic
colon may compress left testicular vein; left renal vein may
be sandwitched between abdominal aorta and superior
mesenleric artery, absence or incompetence of valves in
testicular vein are some other explanations.
25. Infertility and varicocele
Varicocele is one of the treatable surgical causes of infertility.
Pampiniform plexus is believed to constitute a counter current
heat exchanger mechanism to keep scrotal temperature low,
favourable for spermatogenesis. Chronic venous congestion
raises scrotal temperature and hence may reduce spermato-
genesis.
26. Diagnosis of varicocele
• Dragging pain in scrotum extending to groin
• Positive fluid thrill on coughing in primary varicocele
• Reduction in size of swelling on lying down or elevation
of scrotum
• The dilated and tortuous veins give a feeling of ‘bag of
worms’
• Negative floctuation and transillumination.
27. Surgical treatment of varicocele
1. Palomo operation—Division of testicular vein above internal
inguinal ring in the posterior abdominal wall lateral to
external iliac artery.
CASE – 1 95

2. Classical operation—Through inguinal or scrotal approach,


the dilated veins are dissected and divided, keeping few to
maintain venous drainage.
28. Advantages and disadvantages of Palomo operation
Palomo operation has the advantage of simplicity; less chance
of endangering testicular blood supply; but it may aggravate
varicocele in 10 percent cases.
29. What happens if testicular artery is divided high-up in
abdomen?
Testis is likely to get adequate blood supply through
anastomosis of testicular artery with cremasteric artery, a
branch of inferior epigastric artery and artery to vas, a branch
of superior vesical artery.
30. Complications of hydrocele and varicocele surgery
1. Haemorrhage
2. Infection
3. Testicular atrophy
31. Role of sclerosing agents in treatment of vaginal hydrocele
and varicocele
1. Tapping of tunica vaginalis, followed by injection of
sclerosing agents to obliterate the space in tunica has not
found good favour.
2. Injection of sclerosing agents into dilated veins in analogy
of sclerosing agent injection into haemorrhoids has not been
practised.
32. Differentiation between funiculitis (inflammed spermatic
cord, usually filarial) and stangulated inguinal hernia
Funiculitis Strangulated inguinal hernia
Fever, chill, simultaneous Sudden irreducibility of
swelling of cord and testis. existing swelling
Cord, and epididymis are Features of intestinal
thick and tender obstruction
Dull percussion note no Tympanitic note on
features of intestinal percussion
obstruction
Scrotal elevation relieves Scrotal elevation does
pain not reduce pain
Case 2

A 35-year-old male complains of a soft inguinoscrotal swelling on


right side reducing in size in recumbency. Examination reveals
swelling to be completely reducible with positive impulse on
coughing. Deep ring occlusion test is positive.
1. What do you think of this swelling?
Complete indirect inguinal hernia.
2. What is a hernia?
A hernia is a protrusion of a viscus or part of the viscus
through an abnormal opening in the walls of its containing
cavity.
3. Types of inguinal hernia
Direct, indirect and femoral
4. Precipitating factors for inguinal hernias
• Weakness of abdominal wall
• Increase in intra-abdominal pressure
• Persistent processus vaginalis
• Patent canal of Nuck (indirect inguinal hernia in females).
5. Anatomy of inguinal canal
Inguinal canal in an oblique fibromuscular tunnel, about
4 cm long, in the lower part of anterior abdominal wall,
parallel to inguinal ligament. It transmits spermatic cord in
males and round ligament of uterus in females. Its anterior
wall is formed by skin, superficial fascia, external oblique
aponeurosis, and internal oblique muscle (in lateral third
only). The posterior wall is formed by fascia transversalis and
conjoint tendon. The roof is formed by arched fibres of internal
oblique and transversus abdominis muscles and the floor by
upper surfaces of inguinal and lacunar ligaments.
6. Anatomy of inguinal rings
Superficial inguinal ring is a slit in external oblique aponeu-
rosis, 1 cm above and lateral to pubic tubercle. Deep inguinal
CASE – 2 97

ring is situated 1 cm above the midpoint between anterior


superior iliac spine and symphysis pubis. It is a U-shaped
condensation of fascia transversalis being incomplete above.
The inferior epigastric artery courses up along the medial
side of the ring.
7. Natural mechanisms of preventing inguinal hernia
• Obliquity of inguinal canal
• Internal oblique muscle opposite the deep inguinal ring.
• Shuttering action of arched fibres of internal oblique and
transversus abdominis
• Plugging action of spermatic cord due to contraction of
cremasteric muscle
• Sliding valve action of U-shaped deep inguinal ring.
8. Types of indirect inguinal hernia
Vaginal, Funicular and Bubonocele. In vaginal hernia, the
processus vaginalis is patent throughout, the hernia thus
descending down to the bottom of scrotum, covering the testis
in front and on the sides. In funicular type, the processus
vaginalis is closed above the testis, so that the hernial sac is
separate from tunica vaginalis testis and testis can be palpated
separate from the hernia. In bubonocele, processus vaginalis
is closed at the superficial inguinal ring, so that the hernia
lies in the inguinal canal only, appearing as an inguinal
swelling; unlike inguinoscrotal swellings as in vaginal and
funicular types.
9. Coverings of complete indirect inguinal hernia
Skin, superficial fascia, dartos muscle, external spermatic
fascia, cremaster, internal spermatic fascia, processus
vaginalis.
10. What is direct inguinal hernia?
Here, the hernia protrudes directly through the posterior wall
of inguinal canal in the Hesselbach’s triangle. The neck of
the sac lies medial to inferior epigastric artery and is too wide;
hence strangulation is extremely rare.
11. What is Hesselbach’s triangle?
This triangle is situated in the posterior wall of inguinal canal,
bounded medially by outer border of rectus, laterally by
inferior epigastric vessels, floor by fascia transversalis and
98 SHORT NOTES AND SHORT CASES IN SURGERY

below by inguinal ligament. The triangle is bisected by lateral


umbilical ligament, which is obliterated hypogastric artery.
12. Differential diagnosis of inguinoscrotal swelling
• Indirect complete inguinal hernia
• Infantile hydrocele, congenital hydrocele
• Encysted hydrocele of cord
• Varicocele
• Lymphvarix of cord.
13. Differential diagnosis of inguinal swelling
• Inguinal lymphadenopathy
• Saphena varix
• Femoral hernia
• Lipoma of cord
• Aneurysm of femoral artery
• Encysted hydrocele of cord
• Undescended or ectopic testis
• Psoas abscess
• Malgaigne’s bulge (a precursor of direct inguinal hernia)
• Distended psoas bursa
• Rupture of adductor longus.
14. Complications of hernia
• Irreducibility
• Obstruction
• Strangulation, incarceration
• Torsion of omentum when contained in the sac
• Hydrocele of hernial sac.
15. Why hernia becomes irreducible?
Hernia becomes irreducible when
1. There is adhesion between sac and its contents
2. Adhesion between contents themselves
3. Narrowing of neck of sac by fibrosis
4. Retention of faeces within intestine contained in the sac.
16. How to differentiate obstructed hernia from strangulation?
In obstruction, hernia is painless, soft, non-tender with
positive impulse on coughing but when strangulation is
present (arrest of blood supply to the contained intestine
leading to gangrene of bowel), the hernia becomes painful,
tense, tender and irreducible and cough impulse becomes
absent.
CASE – 2 99

17. Incidence of inguinal and femoral hernias


Indirect inguinal hernia is the most common among all age
groups; men, women and children (50%). In women femoral
hernia occupies next position (40%). Femoral hernia is rare
in males.
18. Causes of increased intra-abdominal pressure
• Chronic cough
• Chronic constipation
• Urethral obstruction, prostatic hypertrophy.
19. What is conjoined tendon?
The combined aponeurosis of transversus abdominis and
internal oblique muscles and the transversalis fascia is called
conjoined tendon. It is used to repair the defect in direct
inguinal hernia.
20. Types of repair employed for hernia
• Bassini repair
• McVay repair
• Shouldice repair
• Lichenstein repair
21. Components of hernial surgery
Herniotomy, herniorrhaphy and hernioplasty. Herniotomy means
excision of sac after its neck is transfixed and ligated. It is
done in children and young adults with good abdominal
muscle tone. In direct inguinal hernia the sac is inverted rather
than excised. Herniorrhaphy refers to reconstruction of
posterior wall of inguinal canal by approximating the
conjoined tendon to the curved edge of inguinal ligament by
non-absorbable material. Hernioplasty means reinforcement
of posterior wall of the canal by autogenous tissue (fascia
lata, rectus sheath flap, skin flap) or Marlex mesh.
22. Features of direct inguinal hernia
Ten to fifteen per cent of inguinal hernias are direct and over
half of these are bilateral. Women never practically develop
direct inguinal hernia. Damage to ilioinguinal nerve (e.g. by
previous appendectomy) and raised intra-abdominal pres-
sure are the predisposing factors. The hernia lies behind the
spermatic cord, unlike indirect inguinal hernia, where the
100 SHORT NOTES AND SHORT CASES IN SURGERY

sac is in front of spermatic cord. The protruding mass is


mainly extraperitoneal fat.
23. Recurrence rate after hernial repair
Indirect Direct
Children 1-4% —
Adults 5-10% 15-30%.
24. Sites of hernia recurrence
Internal ring in 50 percent cases. This is due to leaving the
ring too loose or not ligating the sac high enough.
The pubic tubercle in 25 percent cases. This is due to failure
in putting the most medial repair stitch into the periosteum
over the pubic tubercle.
25. Indications for Marlex mesh repair
Weak conjoined tendon, and other tendinous and fascial
layers; recurrent direct hernia, very large direct hernia, very
old people or obese with lax abdominal wall.
26. Disadvantage of Dacron/Marlex mesh repair
Infection is not infrequent. If it occurs the mesh should be
removed.
27. Significance of a hydrocele in a child compared with an
adult
In children, hydroceles are usually accompanied by indirect
inguinal hernia. Hence during hydrocele surgery, the internal
inguinal ring be explored. In a child, if hernia is present on
left side, right sided hernia will be present in 50 percent but if
it is present on right side, bilaterality is only 5 percent.
28. How to differentiate omentocele from enterocele?
In omentocele, peristalsis is not present nor peristalsis sound
is heard; consistency is firm, doughy, and non-floctuant with
dull percussion note, and reduction is usually difficult. The
opposite is true for enterocele.
29. What is femoral hernia?
It is the protrusion of the extraperitoneal fat and omentum
with or without abdominal contents through the femoral
canal. The hernial sac passes through femoral ring to enter
into femoral canal.
CASE – 2 101

30. What is femoral canal?


It is the innermost compartment of femoral sheath. It is 2 cm
long and conical in shape with base upwards and apex
downwards. It extends from femoral ring above upto
saphenous ring below. It is bounded anteriorly by inguinal
ligament, posteriorly by pectineal ligament, medially by
lacunar ligament and laterally by fibrous septum, separating
it from femoral veins. The canal is closed above by condensed
extraperitoneal tissue; pierced by lymphatic vessels. The canal
contains fibrofatty tissue, and lymphnode (Cloquet’s node).
31. Characteristics of femoral hernia
• Round or oval swelling below inner end of inguinal
ligament in elderly fatty female
• Positive cough impulse
• When reducible, it reduces upwards and backwards
• Inguinal canal empty
32. Variants of femoral hernia
1. Laugiere’s femoral hernia—Where the hernia passes through
a gap in lacunar ligament and is nearly always strangulated.
2. Narath’s femoral hernia—The hernia is hidden behind femoral
vessel and occurs in patients with congenital dislocation of
hip and is due to lateral displacement of psoas.
3. Cloquet’s hernia—Here the sac lies under the fascia covering
pectineus muscle.
33. What is sliding hernia?
In this type of hernia, the caecum and appendix on right and
sigmoid colon on left side slide down outside the hernial sac
forming a part of it’s wall and being covered by peritoneum
on the hernial aspect only.
34. What is interstitial hernia?
Here the sac lies between different layers of abdominal wall.
35. What is spigelian hernia?
It is a variety of interparietal hernia, occuring at the level of
arcuate line. It appears as a soft reducible mass below umbi-
licus, lateral to rectus.
36. What is obturator hernia?
Here the hernia passes through obturator canal, commonly
in elderly females. The leg on affected side is kept in semi-
flexed position and movement increases pain, PV and PR
102 SHORT NOTES AND SHORT CASES IN SURGERY

examinations may discover the hernia and once strangulated,


pain is referred to knee.
37. When orchiectomy is advised in hernial repair?
In elderly patients, removal of testis and spermatic cord helps
in effective repair in recurrent inguinal hernia, sliding hernia
and some large direct hernias.
38. What is Richter’s hernia?
Here the hernial sac contains only a portion of the circum-
ference of intestine. It usually complicates femoral hernia and
when strangulation occurs, it is usually missed.
39. What is Saphena varix?
A saphena varix is a saccular enlargement of the termination
of the long-saphenous vein and it is usually accompanied by
other signs of varicose veins in the legs. There is impulse on
coughing and a palpable fluid thrill. The swelling disappears
on lying down.
40. What are internal hernias?
• Para-esophageal hernia; hernia through foramen of
Morgagni
• Hernia through foramen of Bochdalek.
• Hernia through foramen of Winslow.
41. Inguinal hernia vs femoral hernia
Inguinal hernia Femoral hernia
— Common to male — Common to female
— Sac protrudes along — Sac comes out through
inguinal canal femoral ring
— Sac is aove inguinal — Sac is below the medial
ligament end of inguinal ligament
— Sac is above and medial to — Sac is below and lateral
to pubic tubercle to pubic tubercle
— Reduces upword and — Reduces upwards and
laterally backwords.
Case 3

A 20-year-old person has presented with pain in the perineum


while passing stool and occasional passage of fresh blood during
as well as after defaecation.
1. What is the presumptive diagnosis?
Anal fissure.
2. What is anal fissure?
A linear ulcer in the anal canal extending from just below the
dentate line to the margin of anus.
3. Aetiology of anal fissure
It may follow severe bout of diarrhoea or constipation. It
frequently occurs in inflammatory bowel disease and in
immediate postpartum period. Syphilis produces chronic
painless anal fissures. Ninety per cent fissures occur in
posterior midline. In many patients, they are idiopathic. Anal
chancer also presents as a painful ulcer. All patients with anal
venereal disease and admitted homosexual be tested for HIV
infection.
4. Treatment of acute anal fissure
• Five per cent Xylocaine jelly application, followed by anal
dilatation with finger or dilators
• High fibre diet, steroid suppository/cream
• Laxatives (bulk laxatives)
5. Treatment of chronic fissure
• Lateral internal sphincterotomy
• Forcible anal dilatation under GA
• Fissurectomy with posterior midline sphincterotomy.
6. What are haemorrhoids?
Haemorrhoids are veins occuring in relation to anus.
Haemorrhoids can be external or internal. The external variety
is covered by skin, while the internal variety is covered by
mucous membrane. When the two forms occur together they
are called intero external haemorrhoids.
104 SHORT NOTES AND SHORT CASES IN SURGERY

7. What are internal haemorrhoids?


They are dilatation of the internal venous plexus within,
enlarged displaced anal cushions. They primarily occur at 3,
7 or 11, `O’ clock position due to divisions of superior rectal
artery. In between these three primary haemorrhoids, there
are secondary smaller haemorrhoids.
8. Predisposing factors for haemorrhoids
• Heredity: large arterial supply, weak-venous support
• Chronic constipation, straining at stool
• Colitis, diarrhoea, dysentery
• Pregnancy, (compression of superior rectal veins by uterus)
• Coionic cancer (thrombosis and compression of superior
rectal veins
• Raised portal pressure.
9. Grading of internal haemorrhoids
First degree—haemorrhoids that bleed but do not prolapse
outside anal canal.
Second degree—haemorrhoids that prolapse during defaecation
but return or can be replaced manually.
Third degree—haemorrhoids that are permanently prolapsed.
10. What is `arterial pile’?
Occasionally, there is haemangiomatous condition of branches
of superior rectal artery feeding the anal cushions, giving rise
to arterial pile that bleed profusely.
11. Complications of internal haemorrhoids
• Profuse haemorrhage
• Strangulation
• Thrombosis, often ending up in fibrosis
• Ulceration and suppuration
• Gangrene, portal pyemia
12. Treatment for internal piles
1. Sclerosing agents for first degree piles
2. Band ligation for second degree piles
3. Cryosurgery or photocoagulation
4. Haemorrhoidectomy
13. Indications for haemorrhoidectomy
• Third degree piles
• Failure of non-operative treatment for second degree piles
CASE – 3 105

• Fibrosed piles
• Intero external piles.
14. Complications of haemorrhoidectomy
• Early post-operative pain
• Reactionary haemorrhage
• Anal stricture
• Anal fissure
15. What is an anal fistula?
Anal fistula is an abnormal communication between anal
canal and perianal skin.
16. Association between anorectal abscess and anal fistula
Fifty per cent of anorectal abscesses occur secondary to anal
fistulas. Abscess is the acute manifestation, while fistula is
the chronic manifestation.
17. Causes of anorectal abscess
1. Extension of infection from anal gland
2. Extension of a cutaneous boil
3. Blood borne infection
4. Crohn’s disease of rectum, tuberculosis, granuloma
inguinale
5. Diabetes and AIDS
18. Causative agents of anorectal abscess
• E. coli in over 50 percent cases
• Staphylococcus aureus
• Bacteroides
• Streptococci
• Proteus
19. Varieties of anorectal abscess
Perianal—60%
Ischiorectal—30%
Submucous
Pelvirectal
20. Types of anal fistulas
Standard classification Park’s classification
Subcutaneous Intersphincteric
Submucous Transphincteric
106 SHORT NOTES AND SHORT CASES IN SURGERY

High anal Supralevator


Pelvirectal
21. What is Goodsal’s rule?
Fistulas with an external opening in relation to anterior half
of anus tend to be of direct type while fistula with opening or
openings in posterior half of anus usually have curved tracks
connecting to a solitary internal orifice, usually in midline.
22. Characteristics of tuberculous fistula
• Induration around the track is lacking.
• Opening is ragged and margin undermined
• Watery discharge
• Discolouration of surrounding skin
23. Relationship between carcinoma rectum and anal fistula
Colloid carcinoma of rectum is liable to be complicated by
anal fistula. Colloid carcinomatous fistulas can also develop
without a primary neoplasm in the rectum.
24. Causes of anal and rectal stricture
• Posthaemorrhoidectomy
• Senile anal stenosis
• Lymphogranuloma inguinale
• Inflammatory bowel disease
• Endometriosis of rectovaginal septum
• Malignancy of rectum and anal canal.
25. Malignant tumors of anus and anal canal
• Squamous cell carcinoma (most common)
• Basaloid carcinoma (non-keratinising squamous cell carci-
noma)
• Mucoepidermoid carcinoma
• Basal cell carcinoma
• Melanoma
26. Treatment of malignancy of anal canal
• Small localised lesions—Wide resection or radiotherapy
• Lesions with lymphnode metastasis—Abdominoperineal
resection with block dissection of inguinal nodes
Or
• Chemoradiotherapy with 5 FU and mitomycin for 7 days
followed by radiotherapy. If thereafter, residual tumor is still
present abdominoperineal excision is carried out. Tumors
CASE – 3 107

larger than 5 cm have a higher failure rate with chemoradio-


therapy.
27. Causes of anal incontinence
The causes of anal incontinence relate to following:
1. Descent (rectal prolapse, perineal descent)
2. Destruction (malignancy, irradiation)
3. Debility
4. Deficiency (congenital abnormality)
5. Damage (childbirth, haemorrhoidectomy, fissurectomy)
6. Denervation (spinal injury, spina bifida)
7. Dementia
28. Gold standards of surgery around anus
• Tissue from anal fistulous tract should be biopsied
• All anal-rectal strictures should be biopsied
• After haemorrhoidectomy or excision of fissure, anal canal
should be dilated to prevent stricture formation.
29. What is dentate line?
The dentate line is an important landmark both morpholo-
gically and surgically. It represents (1) site of fusion of
proctodaeum and post-allantoic gut (2) position of anal
membrane. It separates plum coloured rectal mucosa from
white (non-pigmented) anal canal skin. The area above is
drained by portal venous system, is insensitive (autonomic
supply) while area below is sensitive (somatic supply) and
drains to systemic venous system.
Case 4

A 20-year-old healthy young male has been rejected by army for


having dilated and tortuous veins in both legs, but the individual
is asymptomatic.
1. What do you think of this case?
A case of varicosity of saphenous system.
2. Definition of varicosity
Dilated, lengthened and tortuous veins.
3. What is venous pump and how it effects varicosity?
Pumping action of muscles within tight fascial compartments
squeeze or milk the blood in the direction insisted upon by
venous valves, i.e. towards the heart.
4. Causes of varicose veins
1. Primary
• Congenital paucity of valves
• Stretching of valve rings
• Wasting or weakness of muscles
• Stretching of deep fascia
2. Secondary
• Obstruction to venous flow by thrombosis, pressure
(tumor, pregnancy)
• Oral contraceptives
3. Fistula varicosities
• Congenital arteriovenous fistula
• Extensive cavernous haemagioma
4. Trauma
5. The main valves affected in varicosity of leg
1. Sapheno femoral lying at the junction of long-saphenous
with common femoral vein
2. Mid thigh communication
3. Saphenopopliteal junction
4. Either side of tibia and fibula, where perforators pierce the
deep fascia.
CASE – 4 109

Blood in the superficial system normally flows into deeper


veins through the perforators.
6. Anatomy of venous drainage of lower extremity
The veins of lower extremity consist of superficial veins, deep
veins, and perforating veins, which join the two. The
superficial veins consist of greater saphenous and lesser
saphenous veins. The greater saphenous vein begins at
dorsum of foot, passes anterior to medial malleolus and
ascends up to join common femoral vein at fossa ovalis. The
lesser saphenous vein begins posterior to lateral malleolus
and ascends in the posterior calf to join the popliteal vein in
popliteal fossa. The deep veins are paired veins accompa-
nying anterior tibial, posterior tibial and peroneal arteries,
that join together to form popliteal vein which ascends
beneath the adductor tendon to become superficial femoral
vein. It is joined by deep femoral vein accompanying femoral
artery to become common femoral vein which continues up
as external iliac vein.
7. Symptoms of venous insufficiency
Aching, swelling, night cramps. Aching and swelling are
more conspicuous towards end of the day.
8. Complications of varicose veins
• Thrombophlebitis
• Venous ulcers
• Pigmentation and chronic eczema
• Calcification of venous wall
• Talipes equinous due to faulty habit of walking on toes to
get relief
• Haemorrhage from rupture of dilated veins
• Lipodermatosclerosis.
9. What is lipodermatosclerosis?
It is a condition, where there is skin thickening, fibrosis and
pigmentation. Progression to ulcer is common.
10. What is venous ulcer and its management?
Venous stasis either due to superficial venous insufficiency
(e.g. varicose veins) or deep venous insufficiency (e.g. deep
vein thrombosis) leads to local anoxia and edema with
lipolysis of subcutaneous fat and ulcer formation. This ulcer
110 SHORT NOTES AND SHORT CASES IN SURGERY

should be differentiated from arteriosclerotic ulcer, ulcer due


to arteritis and syphilis. Malignant transformation may occur.
Treatment is by elevation of leg, calf exercise, elastic bandages,
and finally by operation for varicosity.
11. Clinical assessment of varicosity
• Percussion test
• Compression tests (Trendelenberg test).
In Trendelenberg test, patient lies upon his back and raises
his leg to empty the veins. A venous tourniquet is applied
just below the saphenous opening and patient is asked to
stand up. In a normal person the sapheous veins fill slowly
from below. Now the tourniquet is released. If there is
incompetence of sapheno femoral valve, the veins fill from
above. If veins fill from above inspite the tourniquet in place,
then there is incompetence of perforators.
12. Common investigations of venous disease
1. Venogram for anatomical delineation and functional
venography to delineate direction of flow.
2. Doppler studies and colour duplex imaging
3. Photoplethysmography
13. Treatment of varicose veins
1. Elastic bandage or stocking, leg elevation, active exercise
2. Sclerotherapy (maximum dose at any one time and any one
point is 1 ml).
3. Ligation and division of those veins into which there is leak
from deep system.
4. Stripping of long-saphenous vein.
14. Differentiation of venous ulcer from arterial ulcer
Venous ulcers almost exclusively occur behind medial and
lateral malleoli. There is associated brawny induration and
brownish discolouration. Arterial ulcers occur anywhere in
the leg and are very painful, have a blue surrounding, then
erythematous ring appears. Venous ulcers are superficial but
arterial ulcers often penetrate the fascia.
15. Differentiation of edema of varicosity from lymphedema
Lymphedema is of rubbery consistency, and is non-pitting.
It does not change with leg exercise.
CASE – 4 111

16. Response in venous pressure in ankle with exercise


While walking, normal persons demonstrate rapid decrease
in venous pressure and slow return to normal when exercise
stops. Patients with varicosity show lesser decrease in
pressure with walking but more prompt return to normal
following stoppage of exercise. Patients with deep vein throm-
bosis have little decrease in venous pressure with walking,
while those with venous obstruction have increase in venous
pressure with walking, with slow return to normal.
17. Types of lymphedema
Primary
• Congenital
• Lymphedema precox
Secondary
• Neoplastic invasion
• Surgical excision of lymphnodes
• Radiotherapy
• Parasitic invasion and infections
• Motor paralysis
18. Investigations for lymphedema
• Lymphangiography; CT/MR for inguinal/pelvic nodes
• Isotope lymphangiogram
• Analysis of protein content of edema fluid
19. Surgical treatment of lymphedema
• Swiss roll operation
• Venous lymphatic anastomosis
• Complete excision of all skin, subcutaneous tissue and deep
fascia followed by split skin graft.
20. Complications of lymphedema
• Recurrent lymphangitis and cellulitis (beta-haemolytic
streptococci)
• Hyperkeratosis of skin
• Lymphangiosarcoma
• Protein loosing enteropathy
21. What does ‘Saphenous’ mean?
The word ‘Saphenous’ is of Arabic origin and means; ‘easily
seen’.
112 SHORT NOTES AND SHORT CASES IN SURGERY

22. What is lymphangioma?


Lymphangioma are congenital lymph filled spaces with thin
walled septum. They can be (1) Capillary’ lymphangioma: that
are brownish parpules or wart like excrescence and on
examination with handlens show as small vesicles
(2) Cavernous lymphangioma consisting of masses of lymphatic
cysts (e.g. cystic hygroma neck, axilla or mediastinum.
Case 5

A 30-year-old male has presented with pain in the leg more so


after exercise. He has been a chronic heavy smoker and examination
reveals weak dorsalis pedis pulsation.
1. What is the possible diagnosis?
Thromboangitis obliterans. (Buerger’s disease)
2. What is Buerger’s disease?
It is an occlusive disease of small and medium sized arteries
(plantaris, tibials and radials), thrombophlebitis of superficial
and deep veins and Raynaud’s phenomena, occuring in
young males, commonly heavy smokers. Gangrene of toes
and fingers are common and usually progressive.
3. Differential diagnosis of Raynaud’s phenomenon
• Arteritis (rheumatoid arthritis, SLE, scleroderma, poly-
arteritis nodosa)
• Cervical rib (costoclavicular syndrome)
• Carpal tunnel syndrome.
• Buerger’s disease.
• Cold injury, diabetic vasculopathy.
• Post-poliomyelitis
• Idiopathic (Raynaud’s disease)
• Use of industrial tools (pneumatic drills)
4. What is Raynaud’s phenomenon?
It is a sequence of blanching, dusky cyanosis and red engor-
gement of fingers often accompanied by pain, on immersion
in cold water due to abnormal sensitivity of arterioles to cold.
5. Investigations of a case of Raynaud’s phenomenon
• ESR
• Rheumatoid factor
• Antinuclear factor
• Cryoproteins
• Cold agglutinins
• Skin biopsy
• X-ray cervical spine.
114 SHORT NOTES AND SHORT CASES IN SURGERY

6. Treatment of Buerger’s disease


• Avoidance of cold
• Stoppage of smoking
• Buerger’s exercise
• Vasodilators—Nifedipine
• Lumbar sympathectomy
• Revascularisation using omentum
7. How effective is sympathectomy?
Immediate results of sympathectomy are good, but after few
months, susceptibility to cold returns, although subjective
symptoms are less, so also degree of cyanosis.
8. Distribution of arterial involvement in Buerger’s disease
• Smaller, more peripheral arteries are involved in segmental
distribution.
• In lower limb, disease generally occurs beyond popliteal
artery and in upper limbs, beyond brachial artery.
9. Microscopic features of involved arterial segment
• Extensive intimal proliferation
• Fibroblastic reaction
• Intact basic architecture of artery
• No lipid deposit or calcification
10. Arteriographic findings in Buerger’s disease
• Smooth intimal lining of larger arteries
• Abrupt areas of occlusion (thrombosis) in smaller arteries,
surrounded by extensive collateral circulation (tree root or
spider like)
• Cork screw deformity of small arteries (thrombosis
followed by partial recanalisation).
11. Factors which negate diagnosis of Buerger’s disease in a
patient of arteritis
• Palpable popliteal aneurysm
• Audible bruit over major vessel
• Calcification of peripheral vessels
• Raised blood cholesterol and sugar
• Age of onset after 40 years
• Only involvement of lower extremity.
CASE – 5 115

12. Physical signs of chronic tissue ischaemia in distal-limb


• Loss of hair
• Atrophy of skin
• Brittle nail
• Rubor on dependency
• Ulcer/gangrene
13. What is Takayasu’s arteriopathy?
It is obliterative arteritis of females, involving major arteries
with a relentless course. Balloon angioplasty or vascular
reconstruction are helpful.
14. What is cystic myxomatous degeneration of a vessel?
It is seen in popliteal artery, where, there is accumulation of
clear jelly like substance in the outer layer of the vessel. The
vessel is stiff with disappearance of pulsation and appearance
of claudication on flexion of knee. Arteriography shows, a
smooth narrowing of an otherwise healthy artery and a sharp
kink on bending of knee. Removal of myxomatous material
is curative, but recurrence may occur demanding excision of
arterial segment and vein patch repair.
15. What is acrocyanosis?
It is a painless, paroxysmal disease, affecting young females,
mainly the distal limbs. There is cyanosis with paresthesia.
Sympathectomy offers relief. Cause is unknown and trophic
skin changes are not seen.
16. What is livedo reticularis?
This is a vasospastic condition characterized by a persistent
mottled reddish-blue discolouration of the skin of extremities.
It becomes worse on exposure to cold. Peripheral pulsations
are normal and there is no trophic changes. Vasodilator drugs
and in severe cases sympathectomy are beneficial.
17. What is claudication and its types?
Claudication refers to pain in a limb on exercise or movement.
It is due to ischaemia of muscles or nerves supplying the
muscles. Claudication can be:
i. Arterial claudication as in Buerger’s disease
ii. Venous claudication as in varicosity
iii. Neurogenic claudication as in lumbar canal stenosis.
Case 6

A 30-year-old female has been complaining of gradually worsening


pain in the lower limbs on walking. Examination reveals absence
of femoral pulses and audible bruit below umbilicus.
1. What is your diagnosis?
Aorto arteritis
2. Forms of aorto arteritis
• Idiopathic
• Tubercular
• Atherosclerotic
3. What is Takayasu disease?
Takayasu disease is an idiopathic aortitis involving the
branches of aortic arch, presenting as pulseless disease or
transiant ischaemic attack/vertebrobasilar insufficiency in
young females. There is rich collateral circulation in the
shoulder, chest and neck areas. Steroids may halt the progress
of disease.
4. Symptoms of aortoilial disease
• Pain in the calf, thigh and buttock on exercise
• Impotency in male
5. Pathological finding in idiopathic aortoilial disease
1. Mild atherosclerotic changes in aorta and iliac vessels
(intimal thickening and smooth muscle proliferation)
2. Marked luminal narrowing, due to thrombus formation
upto common femoral arteries. The profunda femoris is
always patent.
3. Ten per cent of these patients have small aortic aneurysm
and nearly 30 percent have concomitant cerebral vessels
involvement.
6. How collateral flow to lower limb is maintained in these
patients?
The principal collaterals are through the lumbar arteries,
anastomosing distally with the branches of gluteal arteries
CASE – 6 117

and profunda femoris arteries. This collateral flow is sufficient


to prevent ischaemia at rest.
7. Cause of necrotic toe coming suddenly in aortoiliac disease
Distal embolization, not extension of thrombosis to digital
vessels. Rest pain, ulceration or gangrene always indicates
additional distal disease. This is particularly true in diabetics,
where disease of tibial arteries is common.
8. Features of acute arterial occlusion
• Pain
• Paralysis ⎫
• Paresthesia
• Pallor
⎬ 5 P’s

• Pulse absent ⎭
9. Manifestations of chronic arterial occlusion
• Intermittent claudication
• Trophic changes with loss of hair, atrophy of subcutaneous
tissue, brittle nails and rubor on dependency.
• Ulcers on trivial trauma, that take long-time to heal.
• Symptoms aggravated by cold
10. Features of atherosclerotic aortitis
• Involvement in 6th to 8th decade
• Intimal thickening, ulceration are more prominent than
thrombosis
• Renal arteries frequently involved, so also popliteals and
femorals.
• Calcification is frequent.
11. Treatment of aortoiliac discase
In 80% of patients, the course is benign, the claudication
improving with time or remaining stable. Only 2.5 percent of
claudicators develop gangrene. However, when claudication
hampers active life, following steps may be employed.
1. Balloon angioplasty, stenting or atherectomy
2. Thromboendarterectomy and bypass graft
3. Extra-anatomical bypass graft
4. Lumbar sympathectomy
12. Medical treatment of aortoarteritis
• A course of antitubercular drugs for 9 months.
• A course of steroids.
118 SHORT NOTES AND SHORT CASES IN SURGERY

Tuberculosis is known to cause arteritis of vasa vasorum and


tuberculous arteritis is the principal cause of neurological
deficit in patients of tubercular meningitis.
13. Difference between neurogenic and vascular claudications
Neurogenic Vascular
claudication claudication
Symptoms
Brought on by exertion Yes Yes
Walking distance to Highly variable Fairly constant
produce symptoms
Pain on exertion Thigh and buttock Calf
Paresthesia, weakness Present Absent
on exertion
Relief with rest Variable Prompt
(within 5-15 min) (15-60 sec)
Sign
Peripheral pulse Normal Weak
Motor and sensory Present Absent
weakness after exercise
Tendon reflexes in LL Decreased Normal
after exercise
14. Characteristics of diabetic foot
1. Susceptibility to infection
2. Diabetic neuropathy often unrelated to arterial disease, i.e.
patent pulsative vessels
3. Involvement of popliteal artery and its branches down to
plantar arches
15. Characteristic of trophic ulcer in diabetics
• Almost exclusively over first or third metatarsal head on
sole of foot
• Sharply demarcated punched out margin
• Painless
16. When to suspect superior mesenteric occlusion?
1. Elderly patient presenting with acute central abdominal
pain, persistent vomiting, bloody stools, shock, abdominal
tenderness and rigidity.
2. X-ray shows absence of small bowel gas
3. Presence of predisposing factors, i.e. atrial fibrillation, left
atrial myxoma, recent myocardial infarction, endocarditis,
aortitis, portal hypertension, portal pyemia, sickle cell
disease, nephrotics, TAO and contraceptive pill use.
CASE – 6 119

17. Bowel involvement in main trunk of superior mesentery


occlusion
The resultant gangrene involves entire small bowel, the
caecum and proximal ascending colon. Inferior mesenteric
occlusion is often silent due to better collateral circulation.
18. What is ischemic colitis?
Ischemic colitis refers to ischaemia in region of supply of
inferior mesenteric artery. Though infarction is uncommon,
patient has lower abdominal cramp, bloody stool indistin-
guishable from regional ileitis.
19. What is popliteal artery entrapment syndrome?
It is intermittent claudication caused by an abnormal relation
of popliteal artery to medial head of gastrocnemius, resulting
in ischaemia of leg at an unusual early age. Dorsiflexion of
foot aggravates ischaemia and disappearance or weakening
of distal arterial pulse.
20. What is anterior compartment syndrome?
The anterior compartment syndrome is a progressive
neuromuscular disability related to pressure from tissue fluid,
within the closed anterior tibial compartment, that compro-
mises function of tibial artery, anterior tibial nerve, extensor
hallucis longus and extensor digitorum longus muscles. Pain
in the leg is increased with foot movement, erythema and
tenderness over anterior compartment are present. Early
decompression can prevent ischaemic necrosis of entrapped
muscles.
21. What is thoracic outlet syndrome?
Neurovascular compression involving subclavian artery,
brachial plexus and subclavian vein near thoracic outlet
constitute thoracic outlet syndrome. The causative agents
include cervical rib, scalenus anticus muscle, band arising
from 7th cervical vertebra, pectoralis minor tendon, etc.
22. Sites of potential neurovascular compression in shoulder
• Interscalene triangle between scalenus antecus anteriorly,
scalenus medius posteriorly and first rib inferiorly.
• Between clavicle and first rib
• Where pectoralis minor attaches to coracoid process and
axillary artery courses at this point.
120 SHORT NOTES AND SHORT CASES IN SURGERY

23. What is Adson’s test?


Deep inspiration with extension of neck and its rotation to
one side may decrease the radial pulse by compression of
subclavian artery in interscalene triangle by tense anterior
scalenus muscle.
24. Clinical features of cervical rib
Compression of C8 and T1 nerve roots or the lower trunk of
brachial plexus by cervical rib or band arising from seventh
cervical vertebra joining to first rib causes atrophy of intrinsic
hand muscles, especially those in thenar eminences
accompanied by pain and numbness in medial two fingers
and ulnar border of hand and forearm.
25. How to confirm neurovascular compression in a patient of
suspected cervical rib?
• Positive Adson’s test
• EMG to show denervation of intrinsic muscles of hand
• Nerve conduction studies and somatosensory evoked
potential
• X-ray may show cervical rib or long-transverse process of
the cervical vertebra.
26. What is reflex sympathetic dystrophy?
It is a syndrome of burning pain and swelling in an extremity
accompanied by trophic skin changes and sign-symptoms of
vasomotor instability. It occurs following neck-shoulder
injury or myocardial infarction. Severe osteopenia with
increased bone gallium uptake occur in post-traumatic variety
(Sudek’s atrophy). Prednisolone, stellate ganglion block and
physical therapy are helpful.
Case 7

A 50-year-old male, who had undergone pelvic surgery 2 days


ago, is complaining of acute right-sided chest pain, mild fever and
haemoptysis.
1. What is your presumptive diagnosis?
Pulmonary thromboembolism.
2. In what setting does pulmonary embolism occur in a surgi-
cal patient?
Malignancy—Particularly intra-abdominal cancer.
Orthopedic—Immobilized patients for fracture.
Immobility of any cause—Surgical anesthesia, post-surgical
period.
Trauma—Particularly prone for fat embolism.
3. Relation of deep vein thrombosis in legs to pulmonary
thromboembolism
More than 90 percent of pulmonary emboli originate as clots
in the deep veins of lower extremity. Although 80 percent of
clots in deep veins lyse without embolizing, the rest propagate
to iliofemoral veins—inferior vena cava and ultimately pul-
monary artery. One-half to one-third patients with iliofemoral
thrombosis have clinically significant pulmonary embolism.
4. Predisposing factors for calf vein thrombosis in surgical
patient
• Immobility
• Dehydration
• Pelvic surgery
• Prolonged general anaesthesia
• Obesity
• Burns
5. Which factors by enhancing blood coagulation promote
pulmonary thromboembolism?
• Deficiency of protein C, protein S, antithrombin III
• Estrogen containing medications.
122 SHORT NOTES AND SHORT CASES IN SURGERY

6. Symptoms and signs of pulmonary thromboembolism


Symptoms Signs
Pleuritic chest pain Tachypnoea
Dyspnoea Crepitations
Cough Accentuated P2
Haemoptysis Fever
Syncope ECG changes
Sweating
7. ECG changes in pulmonary embolism
• Tachycardia
• Non-specific ST-T changes
• Right axis deviation, Right bundle branch block
• Acute right heart strain (S1, O3 and T inversion in leads
V1-3)
• P pulmonale
8. Which other diseases can mimic pulmonary embolism?
• Pneumonia
• Myocardial infarction
• Acute pneumothorax
9. Helpful imaging procedures for pulmonary embolism
1. X-ray chest
• Pulmonary infiltration
• Elevated hemidiaphragm
• Prominent pulmonary artery
• Atelectasis
• Wedge-shaped density with base towards pleura
(diagnostic but rarely seen)
2. Ventilation perfusion scan
Multiple segmental perfusion defects persisting for 7-14
days or ventilation perfusion mismatch (hypoperfusion
with normal ventilation) are more diagnostic.
3. Intravenous digital subtraction angiography.
4. Pulmonary angiography is most definitive and can detect
emboli as small as 3 mm diameter. The finding of an
intraluminal defect or an arterial cut of are diagnostic.
5. Spiral CT may show the embolus.
10. Treatment of pulmonary embolism
1. Heparin 10.000 IU IV followed by 1000 IU/hr for 5 days
with concurrent oral anticoagulants (warfarin—continued
CASE – 7 123

for 3 months); low molecular weight heparin (fragmin,


dalteparin) are now a days preferred.
2. Thrombolytic therapy with streptokinase or urokinase or
tPA.
Streptokinase—250000 units IV then 100000 units/hr for
24 hours.
Urokinase—250000 units IV then 250000/hr for 12 hours.
tPA—100 mg over 2 hours.
3. Catheter fracture of embolus.
11. How heparin and warfarin act?
1. Heparin inhibits thrombin and potentiates antithrombin III.
2. Warfarin reduces synthesis of vitamin K dependent factors
(pro-thrombin, factor VII, IX and X). It also inhibits protein
C and S.
3. While heparin acts immediately—warfarin takes 6-7 days
for its effect.
12. How effective is heparin in pulmonary thromboembolism?
Anticoagulation with heparin for established pulmonary
embolism is preventive rather than definitive therapy. It .does
not dissolve pulmonary thrombus or thrombus in the calf,
thigh or pelvic veins. It however, reduces fresh thrombi for-
mation and propagation of formed thrombi.
13. How can pulmonary thromboembolism be prevented?
1. Low dose heparin and antiplatelet agents to patients at risk.
2. Intra-operative and post-operative pneumatic compression
of legs.
3. Early ambulation after surgery.
4. Active and passive leg exercises for those, bed ridden and
toe movements for those in plaster.
14. How to diagnose thrombophlebitis of deep veins in leg?
• Pain in the calf or thigh, often associated with swelling.
• Positive Hofman’s sign i.e. dorsiflexion of big toe causes
pain in the calf.
• Doppler study and plethysmography.
• Radionuclide venography and ascending contrast veno-
graphy.
124 SHORT NOTES AND SHORT CASES IN SURGERY

15. Role of inferior vena caval interruption in prevention of


pulmonary thromboembolism
Surgical interruption of the inferior vena cava is indicated,
when recurrent pulmonary embolism would be life-threate-
ning in a patient with major contraindications to anticoagula-
tion or failure or complications of anticoagulant or thrombo-
lytic therapy. Surgical procedures include ligation, plication,
clipping or percutaneous transjugular placement of filters
below the renal veins.
16. Drugs that potentiate or negate warfarin effect by alteration
of prothrombin time
Increase in Decrease in
Prothrombin time Prothrombin time
Allopurinol
Aspirin Antihistamines
Amiodarone Barbiturates
Antibiotics (erythro Contraceptives (oral)
mycin, cephalosporin, Diuretics
telracycline, sulpha)
Cemetidine Gresofulvin
NSAIDs Rifampicin

17. Which blood diseases predispose to phlebothrombosis?


• Sickle cell anaemia
• Polycythemia
• Leukemias
• Antithrombin III, Protein C and S deficiency.
18. What is difference between thrombophlebitis and phlebo-
thrombosis?
In the former, inflammation of venous endothelium predis-
poses to thrombus formation, while in latter, there is simply
thrombus formation inside the vein due to stasis. The latter
group is more prone for dislodgement of thrombus and
distance embolism as clot is soft and insecurly attached to
venous wall.
19. What is red thrombus vs white thrombus?
White thrombus is platelet dominated thrombus, while red
thrombus is RBC dominated thrombus.
CASE – 7 125

20. Consequence of massive pulmonary embolism


• Sudden death due to ventricular fibrillation or obstruction
to pulmonary inflow.
• Those who survive have severe precordial pain, tightness
in chest, marked air hunger, and hypovolemic shock.
21. Long-term management of patients with deep vein throm-
bosis of legs
• Leg elevation while sleeping
• Compressive stocking
• Leg exercise while long air/motor travel
• Avoidance of dehydration, inactivity.
22. Consequences of deep leg vein thrombophlebitis
• Pulmonary thromboembolism
• Varicosity due to destruction of venous valves.
23. Common post-operative pulmonary complications
• Bronchopneumonia
• Atelectasis, lung abscess
• Pulmonary embolism.
Case 8

A 40-year-old male is complaining of periumbilical pain for 12


hours now localized to right iliac fossa, mild fever and vomiting.
There is mild polymorphonuclear leukocytosis and rebound
tenderness.
1. What do you think of this patient?
A case of acute appendicitis.
2. Why and how appendicitis occurs?
Appendicitis is initiated by obstruction of appendicial lumen
by a faecolith, worm, inflammation, foreign body or
neoplasm. Obstruction is followed by infection, edema, rise
in intraluminal pressure and infarction predisposing to
rupture.
3. Positive local indicators of appendicitis
• Localized tenderness in right iliac fossa.
• Positive rebound tenderness
• Positive psoas and obturator signs.
• Spasm of overlying abdominal muscles.
• Rectal tenderness in pelvic appendicitis.
4. Role of imaging in appendicitis diagnosis
• Plain X-ray of abdomen showing faecolith in lower right
quadrant heightens suspicion of appendicitis
• Barium enema, if visualises entire appendix-appendicitis
is excluded.
• High resolution ultrasound can diagnose appendicitis.
5. Anatomic locations of appendix
• Retrocecal—74%
• Pelvic—21%
• Postileal—5%
• Paracaecal—2%
• Subcaecal—1.5%
• Preileal—l%
CASE – 8 127

6. What is McBurney’s point and its importance?


Mc Burney’s point lies at the junction of lateral third with the
medial two-third of a line joining the anterior superior iliac
spine and the umbilicus. It is the classical site of the greatest
tenderness in appendicitis.
7. What is aberrant appendix?
Here appendix is not present in the right iliac fossa. In trans-
position of abdominal viscera or non-rotation of midgut,
appendix is present in left iliac fossa. It may lie high-up in
right side of abdomen, if caecum has not descended or is
mobile.
8. Blood supply of appendix
Appendix is supplied by appendicular artery, a branch of
lower division of iliocolic artery. It is an end artery and its
thrombosis leads to gangrene of appendix.
9. Microscopic anatomy of appendix
The mucosa of appendix is columnar with few crypts that
contain Kulchitzky cells, from which arise the carcinoid
tumors. The sub-mucosa contains the lymphoid follicles. The
muscular coat contains inner circular and outer longitudinal
layers.
10. Causes of right iliac fossa pain
• Appendicitis
• Amoebic typhlitis
• Mittelschmertz
• Pelvic inflammatory disease
• Crohn’s disease
• Diverticulitis
• Mesenteric adenitis
• Porphyria
• Ectopic gestation
• Twisted ovarian cyst
• Right uretcric colic
11. In which group of patients, false positive diagnosis of
appendicitis is high?
1. Women between 20-40 years who with ruptured ovarian
follicle at midcycle have similar symptoms.
2. Children with enteritis and mesenteric adenitis.
128 SHORT NOTES AND SHORT CASES IN SURGERY

12. What is mesenteric adenitis?


Inflammation of lymphnodes, clustered in the mesentery of
terminal ileum present as pain and tenderness in right lower
quadrant mimicking appendicitis. Yersinia enlerocolitica is the
causative agent or often it is of viral origin.
13. How to differentiate pelvic inflammatory disease (PID)
from acute appendicitis?
In PID, fever is higher, pain is often bilateral, there is vaginal
discharge, tenderness in the parametrium and cervical motion
is painful.
14. Extraluminal causes of appendicitis
• Adhesions
• Kinking
• Compression from carcinoma caecum
15. Complications of acute appendicitis
• Perforation
• Abscess
• Recurrent acute attacks
• Pyelophlebitis
16. In which group of patients perforation is highest?
• Young children (< 5 years).
• Elderly
17. Diagnosis of pyelophlebitis
Pyelophlebitis refers to suppurative thrombosis of portal vein.
There is chill, high septic fever, hepatomegaly and jaundice.
X-ray shows gas in the portal venous system.
18. Common tumor of appendix and its features
Carcinoid is the common tumor of appendix, seen in 0.3% of
appendectomies located mostly at tip of appendix and of
small size (< 1.5 cm). Small tumors (< 2 cm) need only
appendectomy, while larger ones (> 2 cm) need right
hemicolectomy.
19. What is pseudomyxoma peritonei?
It presents with massive gelatin like mucinous ascites associa-
ted with ovarian or appendiceal neoplasm (i.e. mucin secre-
ting adeno carcinoma of ovary or appendix). Small bowel
obstruction is a problem. Appendectomy/oophorectomy
CASE – 8 129

(debulking) followed by 5 fluorouracil and melphalan and


radiotherapy are of help with 5 year survival > 50%.
20. What will be the approach if appendicular pain is of 7 days
duration with formation of a well-defined mass?
Expectant treatment with antibiotics and appendectomy 3
months later
21. Most common organism of the large bowel in man
Bacteroides fragilis
22. Antibiotic regimen in expectant treatment
Injectable ampicillin, plus gentamicin and metronidazole.
23. What is the treatment of appendicular abscess?
Simple drainage followed by appendectomy 6-8 weeks later.
24. Treatment of pelvic abscess after appendectomy
Transrectal drainage.
25. Differential diagnosis of appendix mass
• Carcinoma of cecum
• Crohn’s disease
• Ovarian carcinoma
• Actinomycosis
26. Abdominal layers to be traversed during exposure of appen-
dix
From superficial to deep—Aponeurosis of external oblique
muscle, internal oblique muscle, transversalis fascia, muscle
and peritoneum.
27. When conservative treatment is to be terminated?
• A rising pulse rate
• Vomiting early or copious constant gastric aspirate
• Increasing or spreading abdominal pain
• Increasing size of abscess.
28. Contraindications to delayed treatment
• When diagnosis can not be made between acute appendi-
citis and other serious intra-abdominal catastrophe.
• Inflammation still confined to appendix.
• Child under 10 years of age (poorly developed greater
omentum and early perforation).
• Patient over 60 years of age (rupture is silent with minimal
clinical signs).
130 SHORT NOTES AND SHORT CASES IN SURGERY

29. Indications for opening an appendix abscess


• Swelling not diminishing in size after 5th day of treatment.
• Swinging high temperature
• Pelvic abscess.
30. Complications of appendectomy
Early-ileus, wound sepsis, residual abscess (local, pelvic,
paracolic, subphrenic) pyelophlebitis, fecal fistula. Late
Intestinal obstruction from adhesion, incisonal hernia.
31. Is preoperative antibiotic beneficial?
Yes, a single dose antibiotic against gram negative organism
reduces post-operative complications.
32. Can laparoscopic appendectomy replace traditional appen-
dectomy?
Laparoscopic appendectomy is preferred for those having
minimal inflammation or when done electively.
33. What to do if Crohn’s disease is encountered during appen-
dectomy?
Appendectomy is safe, if base of appendix and surrounding
cecum are not-involved, else fecal fistula may occur.
34. What is mucocele of appendix?
Mucocele results, when the proximal end of lumen slowly
becomes completely occluded, usually by a fibrous stricture
and the pent up secretion remains sterile. Appendix is greatly
distended and the symptoms produced are those of mild
subacute appendicitis. Infected mucocele is converted into
empyema. Rupture of mucocele of appendix causes Pseudo-
myxoma peritonei.
Case 9

A patient with history of incessant diarrhoea for over past two days,
now complains to have absolute constipation, abdominal distention
and vomiting. Examination reveals mild abdominal tenderness,
absence of bowel sounds and plain X-ray shows both small and
large bowel gas with air fluid levels.
1. What do you think of this patient?
A case of adynamic (paralytic) ileus.
2. What might have contributed to this condition?
Electrolyte loss, particularly hypokalemia.
3. Common causes of adynamic ileus
• GI surgery
• Peritoneal irritation (blood, bile, pancreatitis, peritonitis)
• Hypokalemia
• Spinalcord injuries
• Uremia, diabetic coma
• Anticholinergic drugs
• Pneumonia, inferior myocardial infarction.
4. Management of adynamic ileus
• Nothing by mouth
• Ryle’s tube aspiration
• Electrolyte replacement
• Measurements of abdominal girth.
• Colonoscopic/flatus tube decompression
• If bowel sound does not return or there is any suspicion of
organic obstruction, operation is indicated for decompres-
sion of bowel by enterostomy/cecostomy.
5. Clinical diagnosis of hypokalemia
• Extreme fatigue and exhaustion
• Muscle paralysis, weak tendon jerks
• Adynamic ileus
• ECG changes.
132 SHORT NOTES AND SHORT CASES IN SURGERY

6. ECG changes of hypokalemia


• Depressed ST segment
• Prominent U waves
• T wave inversion and flattening.
7. Why hypokalemia occurs in protracted vomiting?
Vomiting causes loss of Cl– and H+ leading to
• Alkalosis (alkalosis favors intracellular movement of K+)
• Volume depletion which leads to aldosterone secretion
(aldosterone causes fluid retention but renal potassium loss)
• Chloride loss causes high tubular concentration of
bicarbonate.
8. Which investigations are a must in a post-operative patient
without return of bowel sound after 48 hours?
• Serum electrolyte
• ECG (flat T waves of hypokalemia).
Case 10

A 20-year-sold male is admitted with complaints of absolute consti-


pation, vomiting and pain in abdomen. Examination reveals
marked dehydration, abdominal distention and increased bowel
sounds.
1. What is your diagnosis?
Intestinal obstruction.
2. Additional tests required to confirm the diagnosis
• Plain X-ray of abdomen in erect position, to show intestinal
distention with gas, gas-fluid levels (step ladder pattern).
• Blood-total and differential count shows no leukocytosis.
3. Causes of acute organic intestinal obstruction
1. External hernia
2. Post-operative adhesions
3. Intus susception
4. Volvulus
5. Congenital bands
6. Round worms
7. Internal hernia
8. Gallstone
9. Neoplasm
10. Granulomatous process.
4. Mechanism of fluid loss in intestinal obstruction
• Vomiting
• Pooling within bowel lumen
• Tissue edema
• Peritoneal oozing
5. Complications of intestinal obstruction
• Strangulation and gangrene of bowel.
• Perforation and peritonitis.
• Dehydration, uraemia, adrenal failure.
134 SHORT NOTES AND SHORT CASES IN SURGERY

6. In which circumstances intestinal obstruction can be


present without absolute constipation?
• Ritchter’s hernia.
• Gallstone obstruction
• Mesenteric vascular occlusion
• Pelvic abscess with intestinal obstruction.
7. How to diagnose strangulating intestinal obstruction
• If there is an external hernia, then it is tense, tender,
irreducible, increasing in size without expansile cough
impulse.
• Presence of rebound tenderness at site of pain
• Sudden increase in intensity and frequency of colicky pain.
• Generalised tenderness.
8. Sources of fluids in intestinal lumen
Saliva—1500 ml
Gastric juice—2500 ml
Bile and pancreatic juice—1000 ml
Succus entericus—3000 ml
Total of 8000 ml/24 hours
9. How best gas and fluid levels are indicative of site of
obstruction?
• In large bowel obstruction large amount of gas is seen in
cecum.
• Number of fluid level is proportionate to the degree of
obstruction.
• Closer of the obstruction to ileocecal valve larger the number
of fluid levels.
• Gas filled small intestine is revealed by relatively straight
segments, that generally lie more or less transversely;
obstructed large intestine is disclosed by its haustral
markings, and distended cecum is shown by rounded gas
shadow.
• In adutls two inconstant fluid levels are regarded as physio-
logical—one in duodenal cap and other at terminal ileum;
in infants few fluid levels in small intestine are normal.
• Multiple gas filled loops above fluid levels in both small
and large intestine are evidence of established paralytic
ileus.
• Gas and fluid levels must be interpreted in an erect film.
CASE – 10 135

10. What is closed loop obstruction?


It is seen in carcinomatous stricture of colon. Distally the
colonic lumen is occluded by the neoplasm, while proximally
the ileocecal valve prevents regurgitation of colonic content
to ileum. The pressure within cecum becomes too high to cause
its gangrene and perforation.
11. What is chronic obstruction?
Chronic obstruction is due to colonic carcinoma or diverti-
culitis. Here constipation appears first and continues for some
days, to end in stoppage of passing of flatus. Abdominal
distension is slow to begin, vomiting is delayed and dehydra-
tion is exceptional. Cecum is greatly distended and stands
out with each bout of colic.
12. Causes of intestinal obstruction in newborn
Atresia or stenosis of duodenum (33%), jejunum (15%), ileum
(25%), ascending colon (10%) and multiple sites (17%).
13. Diagnosis of duodenal atresia
• Persistent vomiting soon after birth.
• Visible peristalsis passing from left to right.
• X-ray of abdomen shows double stomach appearance in
erect film.
14. Differential diagnosis of duodenal atresia
• Hypertrophic pyloric stenosis
• Volvulus of midgut
• Annular pancreas
• Band obstruction.
15. What is meconium ileus of newborn?
It is neonatal manifestation of fibrocystic disease of pancreas,
where the terminal ileum is filled with inspissated meconium
and viscid pancreatic mucus, causing neonatal intestinal
obstruction.
16. What is intussusception?
Intussusception is invagination of one portion of gut into
another; commonly the proximal into distal. Rarely the reverse
or retrograde form may occur.
136 SHORT NOTES AND SHORT CASES IN SURGERY

17. Common types of intussusception


Ileocolic 77%
Ileoileocolic 12%
Ileoileal 5%
Colocolic 2%
18. Causative factors for intussusception
• Polyp
• Submucous lipoma
• Papilliferous carcinoma
• Meckel’s diverticulum
• Idiopathic (in small children due to inflammed and swollen
Peyer’s patches in terminal parts of ileum).
19. Clinical diagnosis of intussusception in children
• Paroxysmal abdominal pain
• Red currant jelly stool
• Blood stained mucus on rectal examination
• Palpable mass hardening inbetween pain.
20. What is volvulus?
A volvulus results from axial rotation of a portion of
alimentary tract. Volvulus commonly involves sigmoid colon.
Predisposing factors include band of adhesion (peridiverti-
culitis), overloaded colon, long-pelvic mesocolon, and narrow
attachment of pelvic mesocolon.
21. Potential sites for internal hernias
• Foramen of Winslow.
• A hole in mesentery, transverse mesocolon.
• Retroperitoneal fossae (left paraduodenal, right duodeno
jejunal, inter sigmoid, superior and inferior ileocecal and
retrocecal fossa).
22. Narrowest point of small intestine, where gall stone
impaction is likely
About 60 cm above ileocecal valve.
23. Differentiation of a viable from non-viable bowel
A non viable bowel is dark; its peritoneal covering is dull and
lusterless; musculature is flabby, thin, friable; there is no
bleeding if mesentery is pricked. The peritoneal cavity
contains blood stained fluid.
CASE – 10 137

The potentially viable intestine regains pulsation and pink


colour, when it is wrapped in warm moist pack and patient
inhales 100% oxygen.
Besides colour, peristalsis and arterial pulsations mentioned
above, other methods used include fluorescein dye, surface
oxymetry, and Doppler flow assessment.
24. Available surgical options during laparotomy in intestinal
obstruction
• Lysis of adhesions
• Reduction and repair of hernia
• Resection of compromised or dead bowel with anastomosis
• Resection of obstructing lesion with anastomosis.
• By pass of obstructing lesion.
25. Mortality of surgical treatment of small bowel obstruction
With timely intervention before strangulation, mortality is
< 1% but with strangulated hernia, mortality is over 25%.
26. Chances of recurrent obstruction after laparotomy
Risk of developing obstruction secondary to laparotomy is
20% and there is no proven way to prevent it, although heparin
instillation in the peritoneal cavity may have some beneficial
effect.
27. What factors must be considered in the timing of surgical
intervention in intestinal obstruction?
1. Duration of obstruction, especially the severity of fluid and
electrolyte abnormalities. Prolonged obstruction should
forgo a trial of conservative therapy (IV hydration, naso-
gastric suction).
2. Development of fever, or signs of peritoneal irritation;
markedly elevated TWBC, hyper amylasemia suggest
strangulated or dead bowel.
3. In old age immediate laparotomy be performed when
complete obstruction is suspected.
28. Small gut vs large gut obstruction
Smallgut Largegut
Onset Sudden Gradual
Duration Short Long
Pain Colicky, mid Crampy, felt in lower
abdominal, recurs abdomen, recurs at 15-20
contd...
138 SHORT NOTES AND SHORT CASES IN SURGERY

contd...
Smallgut Largegut
at 4-5 minutes minutes
Vomiting Persistent Absent or late
and early
Distention Central In flanks
Peristalsis Visible ladder Wavy movement
pattern
Case 11

A 30-year-old male having history of epigastric pain and acid


eructation has landed up in emergency, with diffuse abdominal
pain. Examination reveals board like rigidity of abdominal wall
and absence of bowel sounds.
1. What will you suspect in this case?
Most probably acid peptic disease with rupture and
generalized peritonitis.
2. What are types of peritonitis?
• Bacterial: Primary (pneumococcal and secondary to gangre-
nous bowel, ruptured appendix)
• Tuberculous
• Chemical: Soiling by gastric acid, pancreatic juice
• Spontaneous peritonitis of cirrhotics
• Sclerosing peritonitis
• Benign paroxysmal peritonitis
3. Diagnostic features of generalised peritonitis
• Diffuse pain, rebound tenderness, pelvic tenderness in rectal
and vaginal examination.
• Board like rigidity of abdominal musculature
• Paralytic ileus with absolute constipation and vomiting.
• Leukocytosis
• Plain X-ray of abdomen shows dilatation of small and large
intestine with edema of bowel wall and presence of
peritoneal fluid.
4. Complications of acute peritonitis
• Acute intestinal obstruction due to peritoneal adhesion
• Paralytic ileus
• Abscess formation (pelvic abscess, subphrenic abscess,
between the leaves of mesentery.
• Late intestinal obstruction due to fibrous tissue.
5. When to suspect intra-abdominal abscess?
When fever, toxaemia, leukocytosis and ileus fail to respond
to nasogastric suction and adequate antibiotics and fluids.
140 SHORT NOTES AND SHORT CASES IN SURGERY

Such patients should have ultrasound examination for


localization of the abscess.
6. Anatomical locations of subphrenic abscess
1. Left anterior space (in front of stomach)
2. Left posterior space (behind the stomach, anterior to
pancreas, i.e. lesser sac).
3. In front of liver (right anterior space).
4. Right posterior space.
7. Can X-ray diagnose subphrenic abscess?
When gas is present in the abscess cavity, it shows up beneath
the diaphragm but in that case, it may simply indicate
perforated hollow viscus.
8. When abdominal rigidity is absent in peritonitis?
• Late stages of peritonitis
• Severe toxaemia
• Weak abdominal wall.
9. What is sclerosing peritonitis?
It is associated with neoplastic diseases and certain drugs
like methysergide, beta-blockers.
10. Common bacterial agents of peritonitis
• E. coli
• Aerobic and anaerobic streptococci
• Bacteroids
• Mycobacteria
• Pneumococci.
11. What is Hippocratic facies?
It is the patient’s appearance in terminal stages of generalized
peritonitis, when there is shock with cold clammy skin,
sunken eyes, dry tongue, thready pulse, drawn and anxious
face.
12. Factors influencing prognosis in peritonitis
Bad prognostic factors are:
• Severe toxaemia
• Paralytic ileus
• Bronchopneumonia
• Electrolyte imbalance
• Renal failure
• Undrained collections.
CASE – 11 141

13. Treatment approach in peritonitis


• Nasogastric suction
• IV alimentation
• Antibiotics, i.e. IV ampicillin, gentamicin and metronidazole.
• Ultrasound guided drainage of pus collections.
• Sealing of perforation if any.
14. What is benign paroxysmal peritonitis?
It is a familial peritonitis, characterized by abdominal pain
and tenderness, fever, leukocytosis and joint pains. Attacks
last for 1-3 days with complete remission. In children, it
should not be confused with abdominal pain of epilepsy or
porphyria.
15. Source of infection in tuberculous peritonitis
• Tuberculous mesenteric lymphnodes
• Tuberculosis of ileocaecal region
• Tuberculous pyosalpinx
• Blood borne (from pulmonary disease)
16. Forms of tuberculous peritonitis
• Ascitic form
• Encysted or loculated form
• Plastic form
• Purulent form (rare)
17. Diagnosis of plastic form of tuberculous peritonitis
• Matting and dilatation of coils of intestine with thick
mesentery giving dowy feeling.
• Repeated subacute intestinal obstruction
• Pulled up ileocaecal region in barium follow through.
18. What is the policeman of abdomen?
Greater omentum that tries to seal off any intra-abdominal
infection.
19. What are the types of mesenteric cysts?
• Chylolymphatic
• Enterogenous
• Urogenital remnant
• Dermoid
• Pseudomesenteric cyst (break down of tuberculous
lymphnode in mesentery).
• Hydatid cyst of mesentery.
142 SHORT NOTES AND SHORT CASES IN SURGERY

20. Clinical presentation of mesenteric cyst


1. Painless abdominal swelling, periumbilical, often floctuant,
moving truly at right angles to the attachment of mesentery.
Presence of a belt of resonance across the cyst.
2. Intermittent pain in abdomen due to torsion of mesentary
or compromise of gut lumen.
3. Acute abdominal pain due to bleeding into the cyst, its
rupture, infection or torsion.
21. Posterior attachment of mesentery
A line extending from left hypochondrium to right anterior
superior iliac spine.
22. Common retroperitoneal swellings
• Retroperitoneal cysts (teratomatous or Wolffian remnants).
• Retroperitoneal lymphosarcoma
• Adrenal tumor (Phaeochromocytoma)
• Hypernephroma, Wilm’s tumor, pyonephrosis, hydro-
nephrosis
• Paraganglioma
• Retroperitoneal lipoma.
Case 12

A 40 year old fatty female, having long-history of intermittent right


upper abdominal pain and dyspepsia is now admitted with deep
jaundice, clay coloured stool and fever.
1. What is the probable diagnosis?
Obstructive jaundice probably due to choledocholithiasis,
with cholangitis.
2. What is Charcot’s triad?
Pain, jaundice and fever with chill are the signs of cholangitis
in 70 percent cases.
3. What is Courvoisier’s low?
Patients of jaundice due to bile duct obstruction by stone have
a shrunken, fibrotic non-distensible gallbladder, while those
with obstruction due to malignancy or stricture have a
distended palpable gallbladder. Malignant obstruction is
complete, while some obstruction due to stone is incomplete
or intermittent.
4. Origin of CBD stones
• Expelled from gallbladder
• Secondary to biliary infestation by Ascaris lumbricoides and
C. sinensis
• Formation denovo due to biliary obstruction by stricture/
malignancy.
5. Medical causes of obstructive jaundice
• Viral hepatitis (cholestatic form)
• Drug induced (chlorpromazine, testosterone, INH,
rifampicin).
• Primary biliary cirrhosis.
6. Laboratory diagnosis of obstructive jaundice
• Raised serum direct acting bilirubin
• Raised alkaline phosphatase
• Absent urine urobilinogen
144 SHORT NOTES AND SHORT CASES IN SURGERY

7. Investigations advisable for obstructive jaundice


• Ultrasound to show CBD and intrahepatic duct dilatation.
• Percutaneous transhepatic cholangiography
• Endoscopic retrograde cholangio pancreatography.
8. Why ultrasound is the first modality of investigation in
differentiating medical from surgical jaundice?
Dilated intrahepatic ducts in ultrasound, always indicate
mechanical obstruction (surgical jaundice).
9. How ERCP is advantageous?
ERCP has the following advantages:
1. ERCP can image the bile duct and pancreatic duct.
2. Basketing of CBD stone can be performed.
3. Endoscopic sphincterotomy can be performed.
4. Stent placement in malignant bile duct stricture is possible,
to decompress the biliary tract.
10. What is Calot’s triangle?
It is bounded above by liver, medially by common hepatic
duct and below by cystic duct. The cystic artery lies in this
triangle.
11. What is Sclerosing cholangitis?
This term is used to describe fibrous thickening of the bile
duct walls often associated with multiple strictures. It may
occur secondary to duct stones, congenital lesions, operative
trauma or may be associated with ulcerative colitis. CBD gives
a beaded appearance and stenting may provide relief.
12. Surgical anatomy of gallbladder
The gall bladder is pear shaped, 7.5 to 12.5 cm long with
capacity of 50 ml, divisible into fundus, body and neck. The
mucous membrane contains the crypts of Luschka. The cystic
duct is about 2.5 cm long, and contains spiral valve of Heister.
13. Surgical anatomy of common bile duct
CBD is 7.5 cm long, is formed by junction of cystic and hepatic
ducts. It is divided into 4 parts.
1. Supraduodenal portion, 2.5 cm long, runs in free edge of
lesser omentum
2. Retroduodenal portion
3. Infraduodenal portion
4. Intraduodenal portion opening into second part of duode-
num in ampula of Vater.
CASE – 12 145

14. Constitution of bile


Water 97%
Bile salt 2%
Cholesterol 1%
Hepatic bile secretion is 40 ml. per hour.
15. Value of plain X-ray abdomen in gallbladder disease
1. It can show radio-opaque gallstones in 10% cases.
2. Calcification of gallbladder (porcelain gallbladder).
3. Limey bile
4. Gas in duct system.
16. Role of radio-isotope scanning in diagnosis of gallbladder
and biliary disease
131I Rose Bengal and 99mTc labelled derivatives of iminodia-

cetic acid are excreted in bile and are used to visualise the
biliary tree. In acute cholecystitis, gallbladder is not visible.
Hence, it is superior to ultrasound in diagnosis of acalculous
cholecystitis. It also diagnoses biliary atresia and patency of
biliary enteric anastomosis.
17. Clinical presentation of biliary atresia
• Jaundice at birth or by end of 1st week
• Pale stool and dark urine
• Pruritus
• Rickets, clubbing and xanthoma at late stages.
Ten per cent of biliary atresia are correctable, while 90% are
incorrectable.
18. What is choledochal cyst?
It is a congenital cystic dilatation of a portion of common bile
duct, commonly in females. Patient presents in childhood
with upper abdominal pain, fever and features of obstructive
jaundice. Ultrasound is diagnostic and excision is curative.
Untreated, it may lead to biliary cirrhosis, rupture or
malignant changes.
19. Types of gallstones
Cholesterol stones 6% (usually solitary)
Mixed stones 93% (usually multiple, often faceted)
Pigment stones 1% (black, small, multiple)
146 SHORT NOTES AND SHORT CASES IN SURGERY

20. Components of mixed gallstones


Cholesterol, calcium bilirubinate, calcium phosphate, calcium
carbonate, calcium palmitate and proteins.
21. Aetiology of gallstones
• Metabolic
• Infective
• Bile stasis
22. What is lithogenic bile?
Bile with excess of cholesterol in relation to bile salts and
phospholipids, so that excess cholesterol is likely to precipi-
tate and form stone.
23. Factors predisposing to gallstone formation
• Bile cholesterol increases with age and in obesity, with
contraceptive and clofibrate use.
• Bile salt concentration in bile is reduced by estrogens and
interruption of enterohepatic circulation as in diseases of
terminal ileum, cholestyramine therapy.
• Pigments stones are common in haemolytic states (thalas-
saemia, spherocytosis, sickle cell disease).
• Infection of biliary tree by C. sinensis and Ascaris lumbricoi-
des and E. coli also lead to pigment gallstone formation.
• Vagotomy.
24. What is Saint’s triad?
Gallstones, diverticulosis of colon and hiatus hernia. All the
three frequently coexist and each in capable of producing
dyspepsia.
25. How E. coli infection of biliary tree predisposes to stone
formation?
E. coli produce beta glucuronidase which converts soluble
bilirubin into insoluble form.
26. Common pathogenes of acute calculous cholecystitis
• E. coli
• Klebsiella
• Streptococcus faecalis
• Clostridia (rarely), Salmonella
CASE – 12 147

27. What is Boa’s sign?


An area of hyperesthesia between the 9th and 11th ribs
posteriorly; a sign of cholecystitis.
28. What is Murphy’s sign?
Sudden catching pain below tip of right 9th rib with stoppage
of breathing, when the patient of acute cholecystitis is asked
to take deep breath and examiner palpates the above site.
29. Management of acute cholecystitis
1. Nasogastric suction, IV fluids
2. Antibiotic—IV ampicillin, cephalosporins (oral antibiotic
is of less use as cystic duct is already blocked by stone).
3. Analgesic—Opiates. Although opiates cause spasm of
sphincter of Oddi, they can be combined with atropine
derivative.
4. Cholecystectomy—once acute symptoms subside. The
biliary passage should be explored as in 10% cases, CBD
stones are seen.
30. Complications of acute cholecystitis
• Gangrene; can also result from torsion of a hypermobile
gallbladder.
• Perforation
• Empyema
31. What is mucocele of gallbladder?
This occurs, when neck of gallbladder becomes obstructed
by stone, but the contents remain sterile. The bile is absorbed
and is replaced by secreted mucus. The gallbladder is
enormously dilated and palpable.
32. What is cholecystosis?
This is an uncommon group of conditions affecting gallbladder
in which there are chronic inflammatory changes with hyper-
plasia of all tissue elements. It includes cholesterosis, polyposis,
adenomyomatosis and cholecystitis glandularis proliferans.
Treatment is cholecystectomy.
33. Can polyposis be confused with gallstone?
Yes, both polyp and stone cast negative shadows in oral
cholecystogram and acoustic shadowing in ultrasound. Polyp
is constant in position, while the stone floats and changes
place with change in position.
148 SHORT NOTES AND SHORT CASES IN SURGERY

34. Can lithotripsy replace cholecystectomy?


No, extracorporeal shock wave lithotripsy is suitable only
for patients with 1-3 stones in a well functioning gallbladder.
35. How useful are gallstone dissolution methods?
The bile acids, chenodeoxycholic and ursodeoxycholic acid
taken orally can dissolve radioluscent gallstones in a
functioning gallbladderr. However, this treatment causes
diarrhoea and is to be continued for atleast six months. Stone
reucrrence after stoppage of treatment is great. It can be an
adjunct to lithotripsy.
36. Is laparoscopic cholecystectomy preferable?
Yes, this technique is applicable in 75% of patients. Hospital
stay is short and chances of incisional hernia is remote.
37. Indications for choledochotomy at cholecystectomy
• Multiple small stones in GB with patulous cystic duct
• Stone is felt in CBD
• CBD is dilated (> 10 mm)
• History of jaundice or rigors
• Raised alkaline phosphatase.
CBD exploration is not advisable, if a good ultrasound or
cholangiography excludes CBD dilatation and stone.
38. Difference between acute cholecystitis and biliary colic
Biliary colic is the result of stone impaction against or while
passing through cystic duct. Hence, it is rarely associated with
fever or vomiting and pain subsides with time.
39. What is biliary dyskinesia?
Biliary symptoms secondary to functional disorder of bile
ducts and sphincter of Oddi with abnormal elevation of
intraductal pressure.
40. Should cholecystectomy be performed in asymptomatic
patients with gallstones?
Only 20% of patients with gallstones are symptomatic. Hence,
asymptomatic patients need not be subjected to cholecystec-
tomy unless there is fear of malignancy or there is diabetes.
41. What are the ducts of Luschka?
These are hepatocholecystic ducts that drain bile from the
liver directly into gallbladder.
CASE – 12 149

42. Diagnostic criteria of chronic cholecystitis


• Flatulant dyspepsia of long-duration.
• Non-functioning gallbladder with thick walls in ultra-
sound/OCG
• Non-visualization of gallbladder in OCG as it is unable to
concentrate the dye (cystic duct obstruction can also lead
to non-visualization).
43. Diseases caused by gallstones
• Acute/chronic cholecystitis
• Biliary colic
• Obstructive jaundice
• Cholangitis
• Gallstone ileus (impaction at ileocecal valve).
44. Failure rate of laparoscopic cholecystectomy
In 4% cases, conversion to open laparotomy because of non-
visualization of junction of cystic duct with common bile duct.
45. How cholesterol and lecithin, both of which are insoluble,
are kept in solution in bile?
Bile salts alone are relatively inefficient in solubilizing
cholesterol (approximately 50 molecules of bile salts are
necessary to solubilize 1 molecule of cholesterol), but solubi-
lization of lecithin in bile salt solutions results in mixed
micelle, that is seven times more efficient in solubilization of
cholesterol.
46. What are cystic duct syndromes?
Pain in right hypochondrium and fatty food intolerance occur
in both pre and post-cholecystectomy patients. In the first
group, the gallbladder though normal does not expel the bile
in response to cholecystokinin due to fibrotic stenosis, kinking
or adhesions in cystic duct.
In the second group, pain is due to dilatation of cystic duct
remnant, neuroma formation in ductal wall, traction on CBD
by a long cystic duct.
47. Long-term complications of chronic cholecystitis
• Acute exacerbation
• Cholecystenteric fistula
• Pancreatitis
150 SHORT NOTES AND SHORT CASES IN SURGERY

• Common duct stone


• Carcinoma of gallbladder.
48. Clinical implication of calcified gallbladder
Calcified gallbladder has a high association with gallbladder
carcinoma and is an indication for cholecystectomy.
49. Common causes of obstructive jaundice
• CBD stone
• Carcinoma head of pancreas
• Carcinoma ampulla of Vater
• Common duct malignancy
• Extension of gallbladder malignancy
• Malignant nodes in porta hepatis.
Case 13

A 65-year-old male has been admitted with history of upper


abdominal pain radiating to back, weight loss and anorexia. Exami-
nation reveals a hard mass in epigastrium, palpable gall-bladder
and deep jaundice.
1. What is your presumptive diagnosis?
Carcinoma head of pancreas.
2. Origin of pancreatic cancer
• 90%are exocrine tumors and 75% of them are adenocarci-
noma of ductal origin.
• 70% of tumors arise from head and 25% from body and
tail.
• Among endocrine tumors, insulinoma is the most frequent,
and two-third of them occur in body and tail.
3. Symptoms of tumor of pancreatic head
• Palpable tumor
• Localised dull pain, often radiating to back
• Eating may accentuate pain, sitting in hunched position
relieves pain.
• Progessive jaundice in 75% cases.
• Palpable liver and gallbladder in 50% and 30% respectively.
• Spontaneous migrating venous thrombosis, particularly in
a diabetic, needs exclusion of pancreatic cancer.
4. Radiological diagnosis of pancreatic cancer
• Widening of C-loop of duodenum
• Irregularity of duodenal mucosa due to tumor invasion in
hypotonic duodenography
• Reverse 3 sign, anterior displacement of stomach, distortion
and flattening of gastric antrum.
• Ultrasound and CT are more informative and can easily
diagnose spread
• ERCP shows distortion of pancreatic duct system.
152 SHORT NOTES AND SHORT CASES IN SURGERY

5. Common sites of spread of pancreatic cancer


• Liver (but palpable liver is often due to biliary obstruction
than metastasis)
• Duodenum, gastric antrum
• Portal vein (causes ascites), splenic vein thrombosis (causes
splenomegaly)
• Superior mesenteric vein, para-aortic or mesenteric
lymphnodes.
6. Management of pancreatic cancer
1. Eighty per cent of patients have metastasis at the time of
presentation and because most are in advanced age, treat-
ment is palliative with pain control by opioid analgesic,
relief of jaundice and pruritus by bypass (choledocho duo-
denostomy or cholecysto-jejunostomy) or stenting.
2. Tumors around ampula and body are unresectable. Small
tumors less than 3 cm, localized to head of pancreas can be
resected.
7. What is Whipple procedure?
It is enbloc removal of gallbladder, common bile duct, duode-
num, head of pancreas, and the antrum of stomach with or
without an accompanying vagotomy.
8. Why gallbladder, stomach and duodenum are removed,
when tumor is localized to pancreas?
Once the ampula of Vater is removed, gallbladder does not
function well. The duodenum shares a common blood supply
with head of pancreas. Antrum is removed to achieve a better
margin around the tumor.
9. Chances of survival after resection
50-60% at 1 year
25-40% at 2 years
5-25% at 5 years.
10. Chances of survival with chemoradiotherapy
Median surival in unresectable tumors is only 6-8 months.
11. What is chronic pancreatitis?
The classic syndrome consists of abdominal pain, diabetes,
steatorrhoea and pancreatic calcification. Diabetes only
occurs, when more than 90% of pancreatic function is lost.
CASE – 13 153

12. Is surgery ever required in chronic pancreatitis?


Treatment of chronic pancreatitis is medical with pancreatic
enzyme supplement, low fat diet, pain relief and H2 blockers.
Surgery is done to relieve pain, when ERCP demonstrates
obstruction to duodenal end of pancreatic duct. Such patients
can be helped by pancreatic jejunostomy (anastomosis of
distal end of pancreatic duct by resection of tail of pancreas).
13. Are hyperamylasemia and steatorrhoea present in all cases
of chronic pancreatitis?
Serum amylase is not elevated in chronic pancreatitis.
Steatorrhoea is not present in early cases. In them abnormal
secretin stimulation test (inadequate pancreatic bicarbonate
secretion i.e. < 80 mEq/Lit) speaks of pancreatic deficiency.
14. Local complications of chronic pancreatitis
• Pancreatic pseudocyst
• Fistula (pancreatic ascites)
• Obstructive jaundice
• May mimic pancreatic malignancy.
15. Role of ERCP in diagnosis of chronic pancreatitis
ERCP in chronic pancreatitis shows dilatation and beading
of pancreatic duct system (chain of lakes often with
obstruction at duodenal end). Sphincteroplasty in these
patients, may be of help.
16. Measures for relief of pain of chronic pancreatitis
1. Pancreatic enzymes
2. Sphincteroplasty; pancreatico jejunostomy
3. Visceral ganglionectomy (at the level of celiac axis)
4. Pancreatic excision.
17. Antitumor agents for pancreatic cancer
• Mitomycin-C
• 5 fluorouracil
18. Predisposing factors for chronic pancreatitis
Alcoholism
Hypercalcemia
Hyperlipoproteinemia (type I, IV and V)
CBD stones.
154 SHORT NOTES AND SHORT CASES IN SURGERY

19. Why some surgeons prefer total pancreatectomy rather than


‘Whipel’ in resectable pancreatic cancer?
In 15-20% cases, pancreatic cancer is multifocal with
involvement of head and tail.
20. Can pancreatic cancer attain large size without any
symptoms?
Yes, cancer of body and tail remain asymptomatic for long
time.
21. Mesenteric cyst and its presentation
Mesenteric cysts can be (1) chylolymphatic, (2) enterogenous,
(3) teratomatous or, (4) urogenital remants. Presentation is
in second decade with painless lump, recurrent abdominal
pain, or acute abdomen following rupture of the cyst or
haemorrhage into it, torsion or infection. It is freely mobile at
right angle to mesenteric attachment, is smooth, spherical
with positive flactuation and fluid thrill.
22. Retroperitoneal tumors
Besides tumors of kidney and adrenal, other retroperitoneal
tumors include lymphomas, lymphangioma, lipo/fibro/
neuro/leiomyo sarcomas, dermoid, teratoma, ganglio-
neuroma and neuroblastoma, rhabdomyoma and rhabdo-
myosarcoma.
23. Pancreatic pseudocyst
It is collection of pancreatic secretion and exudates in lesser
omental bursa following acute pancreatitis or pancreatic
trauma. It can be confused with hydronephrosis (left), retro-
peritoneal cyst/tumor, mesenteric cyst. The cyst is pain less,
presenting as a lump in epigastrium with indistinct upper
and lower margins. It is often very tense without flactuation
and fluid thrill. Rupture, haemorrhage, CBD obstruction and
secondary infection are the complications. It has no epithelial
lining (hence pseudocyst). Spontaneous regression in 6 weeks
may occur else drainage to stomach/duodenum or marsupia-
lization be considered. Resection is reserved for small cysts
of body and tail.
Case 14

A 50-year-old male is admitted with history of anorexia, weight


loss and postprandial fullness. Examination reveals moderate
anaemia and a palpable epigastric mass. His stool for occult blood
is positive.
1. What is the probable diagnosis?
Carcinoma of stomach.
2. Risk factor development of gastric cancer
• Blood group A
• Loss of parietal cell mass (Pernicious anaemia)
• Adenomyomatous polyp
• Cigarette smoking
• Acanthosis nigricans
• Gastric mucosal dysplasia
• Dermatomyositis
• Barret’s esophagus
• Postgastrectomy
• Atrophic gastritis
• Gastric ulcer.
• H. pylori
3. Types of gastric cancers
• Polypoid intraluminal mass
• Non-infiltrating ulcer
• Infiltrating ulcer
• Diffuse infiltration (linitis plastica).
4. Spread of gastric malignancy
• Direct spread to pancreas, transverse colon, esophagus and
liver.
• Lymphatic spread of left supraclavicular node (Virchow’s
node)
• Spread by blood stream to liver, skin, lungs and bone
• Transperitoneal implantation (rectal shelf).
156 SHORT NOTES AND SHORT CASES IN SURGERY

5. What is Krukenberg’s tumors?


Malignant implantations in ovaries in postmenopausal
women, with ovarian enlargement due to transperitoneal
spread of cancer stomach or retrograde lymphatic permeation
from carcinoma colon.
6. Is melena specific for cancer stomach in old people?
No, melena can occur due to many benign conditions of
stomach (erosive gastritis, gastric-duodenal ulcer, leomyoma),
bleeding esophageal varices, esophageal malignancy and
ampullary carcinoma.
7. Is epigastric lump a rule in presentation of carcinoma of
stomach?
No, only 30% of patients have palpable lump. Carcinoma in
body of stomach are silent and may manifest first with
metastasis (ascites, jaundice).
8. How accurate is radiodiagnosis for carcinoma of stomach?
Radio diagnosis is accurate in 90% of pyloric growths; 75%
for cardia, 60% for neoplasms in body of stomach.
9. Radiological evidence of malignancy in gastric ulcer
• Ulcer bigger than 2.5 cm in diameter
• Ulcers with converging gastric folds that stop short of ulcer
• Ulcers with elevated margin
• Ulcers with underlying rigid gastric wall.
10. Lymphatic drainage of stomach
1. Lymphatics from proximal half of stomach drain into left
gastric and splenic lymphnodes and thence into pancreatic
lymphnodes.
2. Lymphatics of the antrum drain into right gastric
lymphnodes superiorly and into right gastroepiploic and
subpyloric nodes inferiorly.
3. The lymphatics of pylorus drain into gastric node superiorly
and subpyloric lymphnodes inferiorly.
11. What is colloid carcinoma of stomach?
Colloid carcinoma accounts for 6% of gastric carcinoma. In it,
all the layers of stomach are infiltrated by loose areolar tissue
containing colloid and the tubular glands are extremely
CASE – 14 157

distended with this substance. It is particularly liable for


transperitoneal spread to pelvis.
12. Radiological diagnosis of linitis plastica
• Small capacity of stomach in barium meal
• Enormous thickening and non-pliability of stomach wall.
13. Differential diagnosis of linitis plastica
• Colloid carcinoma of stomach
• Boeck’s sarcoid of stomach
• Crohn’s disease of stomach
14. What is Boeck’s sarcoid?
It is a noncaseating epithelioid granuloma involving lympho-
reticular system, lungs, skin, eyes and intestine in young
persons. Bilateral hilar adenopathy and pulmonary infil-
tration with positive Kveim test and prompt response to
steroid are diagnostic.
15. Does a palpable lump imply inoperable lesion?
No signs, of inoperability are:
1. Fixation to pancreas or posterior abdominal wall or involve-
ment of mesentery in the region of superior mesenteric
vessels.
2. Fixed regional lymphnodes, preaortic lymph node involve-
ment
3. Multiple secondaries in the liver
4. Peritoneal seedling locally or in the pelvis.
Case 15

A 50-year-old man has presented with gradually progressive


dysphagia, weight loss and alkaline regurgitation.
1. What is the presumptive diagnosis?
Esophageal malignancy.
2. Macroscopic types of esophageal cancers
• Annular stenosing lesions usually found at cardia
• An epitheliomatous ulcer with raised everted edges.
• A fungating cauliflower like friable mass.
3. Cell types of esophageal cancer and their relative frequency
Ninety-five per cent of esophageal cancers are squamous cell
carcinoma, with 50% occurring in middle-third, seventeen per
cent in upper-third and 33% in lower-third of esophagus.
Adenocarcinomas account for 3-5% and arise from columnar
cells of lower esophagus (Barret’s epithelium) or are gastric
adenocarcinoma spreading upwards.
4. Spread of esophageal cancer
1. Local spread to mediastinum, left main bronchus, trachea,
left recurrent laryngeal nerve.
2. Lymphatic spread from cervical esophagus to supraclavi-
cular triangle, from thoracic esophagus to paraesophageal
and tracheobronchial nodes.
3. Blood borne metastasis to liver.
5. Significance of cough, sternal pain and rapid weight loss
in esophageal cancer
Cough may occur due to spilling of retained alkaline eso-
phageal content into tracheobronchial tree. Trachea and left
main bronchus may be eroded in growths involving middle
third of esophagus to cause cough and haemoptysis. Rapid
weight loss is an ominous sign. Sternal pain unrelated to
eating indicates local extension of tumor.
CASE – 15 159

6. Do all strictures of esophagus on barium swallow need


esophagoscopy?
Yes, all esophageal strictures need esophagoscopic biopsy to
rule out malignancy.
7. Imagings for esophageal malignancy
1. Barium swallow positive for annular, irregular space
occupying lesions.
2. CT scan for delineating extent of growth and extension to
mediastinum and nodes.
3. Endosonography for staging of lesions.
8. Can achalasia mimic symptoms of esophagial malignancy?
Yes but, achalasia can be differentiated by endoscopy, eso-
phageal manometry and cinefluorography.
9. How the two types of esophageal dysphagia can be
diagnosed from history?
True esophageal dysphagia can result from
1. mechanical narrowing of esophagus or
2. motor disorder. Luminal narrowing can be due to peptic
stricture, esophageal webs, cancer or pressure effect of
mediastinal growth. Motor dysfunction is due to achalasia,
diffuse esophageal spasm or scleroderma.
Mechanical obstruction typically presents with solid food
dysphagia that over time progresses to include liquids.
Dysphagia that worsens on ingesting cold liquids and
improves with warm liquids, suggests motor disorder.
Relief of obstruction by vomiting suggests an organic
narrowing. If the obstruction can be overcome by repeated
swallowing or drinking water, a motor disorder is likely.
Motor disorders often begin with dysphagia for both solid
and liquids.
10. What is transfer dysphagia?
It is the dysphagia due to dysfunction of upper esophageal
sphincter (cricoarytenoid dysphagia).
11. Epidemiology and risk factors of esophageal cancer
• Black male
• Alcohol and tobacco abuse
• Chronic hot beverage ingestion
• Lye ingestion (> 30% develop esophageal cancer)
160 SHORT NOTES AND SHORT CASES IN SURGERY

• Tylosis (> 40% develop esophageal malignancy)


• Achalasia
• Pulmmer Vinson syndrome
• Non-tropical sprue
• Metal workers, construction workers
• Africans, Chinese, Japanese, Russians.
12. Presenting symptoms of esophageal cancer
• Dysphagia
• Weight loss
• Regurgitation
• Aspiration pneumonitis
• Hoarseness
• Cough
• Choking
• Chest pain on swallowing.
13. Premalignant benign disorders of esophagus
• Barrett’s esophagus
• Pulmmer Vinson syndrome
• Leukoplakia
• Caustic injuries
• Achalasia
• Probably scleroderma.
14. What is Barrett’s esophagus?
A glandular metaplasia of the squamous esophageal
epithelium, usually secodnary to chronic reflux esophagitis.
15. Treatment modalities for esophageal cancer
1. Postcricoid carcinoma—Radiotherapy
2. Carcinoma upper third—Radiotherapy as metastasis has
occurred by time of diagnosis. However, esophagoctomy
with jejunal or colonic transposition can be done in early
cases.
3. Carcinoma middle third—Principally radiotherapy as
tumor is frequently adherent to aorta, left main bronchus
or esophagogastrectomy with anastomosis above level of
aortic arch.
4. Carcinoma lower third—Esophagogastrectomy (3/5 of
stomach is sacrificed) with colonic interposition or jejunal
Roux-en-Y loop.
5. Squamous cell lesions are treated by radiotherapy.
CASE – 15 161

16. Role of radiotherapy in esophageal caner


• Pre-operative radiotherapy enhances resectability.
• With radical radiotherapy, 1 year survival is in 45% and 5
year survival in 15%.
17. Surgical outcome in esophageal cancer
Median survival in stage I disease is 29 months, stage II
disease 20 months and stage III disease 7 months.
Radiotherapy is most effective for bulky tumors, rather than
ulcerating lesions while surgery in best treatmetn for tumors
at the cardia.
18. Reason for right hemithoracotomy approach for upper and
middle esophageal lesions
Though thoracic esophagus is left sided, it passes behind the
aortic arch and is difficult to approach. On the right side, the
only intervening structure is the azygous vein which can be
sacrificed.
Case 16

A 40-year-old chronic alcoholic has been admitted with history of


haematemesis, bleeding per rectum and gradually increasing
swelling of abdomen. Examination reveals mild ascites and marked
splenomegaly.
1. What is the presumptive diagnosis?
Portal hypertension probably consequent to cirrhosis.
2. What is portal hypertension?
Portal hypertension is an elevation in portal venous pressure
from normal of 7-10 mm of Hg to 15-60 mm of Hg. Portal
vein pressure 4 mm of Hg above inferior venacaval pressure
also implies portal hypertension.
3. Causes of portal hypertension
Prehepatic —Twenty percent cases, commonly children;
congenital anomaly of portal vein, cavernoma-
tous malformation of splenic or portal vein,
thrombosis consequence to omphalitis, acute
pancreatitis.
Hepatic —Eighty per cent cases, commonly adults;
cirrhosis, schistosomiasis, non-cirrhotic portal
fibrosis.
Posthepatic —Budd-Chiari syndrome, constrictive peri-
carditis.
4. Sites of portosystemic anastomosis
1. Around lower end of esophagus, where esophageal veins
join with left gastric and short gastric veins.
2. Around umbilicus, where paraumbilical veins accompa-
nying round ligament of liver join superficial veins of
anterior abdominal wall.
3. Around lower third of rectum and anal canal, where
tributaries of superior mesenteric vein join middle and
inferior haemorrhoidal veins.
4. Extraperitoneal surface of abdominal organs, where
tributaries of superior and inferior mesenteric veins
CASE – 16 163

anastomose with subdiaphragmatic and retroperitoneal


veins (veins of Retzius).
5. Anatomy of portal vein
Portal vein is 8-9 cm in length and is formed by the confluence
of splenic and superior mesenteric veins. The portal vein lies
dorsal and medial to common bile duct and hepatic artery in
the hepato-duodenal ligament. There are no valves in portal
circulation.
6. Characteristics of blood-flow to liver
Total hepatic blood-flow is 1500 ml/minute. Three-fourth of
this flow and one-fourth of oxygen supply to liver are derived
from portal vein, while one-fourth of bloodflow and three
fourth of oxygen requirement are provided by hepatic artery.
7. Are esophageal varices truly esophageal?
No, these varices extend well into the stomach.
8. Is upper GI bleed in portal hypertension always from
esophageal varies?
In 20% cases, bleeding from another source (gastritis, peptic
ulcer, Mallory Weiss tear) can be demonstrated by endoscopy.
9. Incidence of variceal bleed in cirrhotics
Thirty percent of cirrhotics will experience a bleeding episode.
Variceal haemorrhage can be anticipated within 2 years of
the diagnosis of esophageal varices.
10. Incidence of second variceal bleed
Forty to sixty per cent of cirrhotics, who have bleed once from
esophageal varices will rebleed within 1 year of the first
episode. Risk of fatal haemorrhage is 50% for the initial
episode and is not increased by subsequent episodes of
variceal bleeding.
11. Major side effects of vasopressin
Peripheral vasoconstriction, intestinal cramps, bradycardia,
hypotension, decreased cardiac output and angina pectoris.
Skin necrosis can occur at site of peripheral infusion. Con-
comitant nitroglycerin infusion can minimize above side
effects and further reduces portal pressure.
12. How effective is endoscopic sclerotherapy?
Acute variceal bleed may be controlled with a single treatment
in 70-80% of patients. Repeat sclerotherapy can obliterate
164 SHORT NOTES AND SHORT CASES IN SURGERY

varices. Half of patients treated by sclerotherapy, however,


rebleed at sometime, but of less severe nature. Endoscopic
therapy probably does not increase overall early survival in
bleeding cirrhotics.
13. What next, if bleeding uncontrolled by sclerotherapy?
Five to ten per cent of patients do not benefit from endoscopic
sclerotherapy in whom, percutaneous transhepatic coronary
vein embolisation or emergency surgery may be required.
14. Operations available for bleeding from esophageal varices
1. Shunting operations
• Portocaval shunt
• Mesocaval shunt
• Splenorenal shunt
• Transjugular intrahepatic portosystemic stenting (TIPS)
2. Devascularization operations
• Esophageal transection
• Direct ligation of esophageal varices
• Splenectomy
• Esophageal devascularization + splenectomy.
15. Difference between selective and non-selective shunts
Selective shunts are designed to decompress esophageal
varices, while allowing prograde portal blood flow to liver.
The selective splenorenal shunt involves anastomosis of distal
splenic vein to the left renal vein. The coronary (left gastric)
and other periesophageal collateral veins are also ligated.
Non-selective shunts decompress the entire portal system,
resulting in stagnation or even reversal of portal hepatic flow.
This reduction in prograde hepatic blood flow may result in
hepatic encephalopathy.
16. What is hepatic encephalopathy and its treatment?
Hepatic encephalopathy is a constellation of symptoms
pertaining to CNS, occurring in patients with chronic liver
disease and in patients undergoing porto-systemic shunting.
Symptoms range from behavioral changes to confusion,
asterixis and coma. Treatment is with exclusion of protein
from diet, lactulose, neomycin and dopamine/dopamine
agonists. Those, who do not respond to this regimen and have
CASE – 16 165

chronic encephalopathy may require ileostomy, colectomy,


or colonic exclusion. Liver transplant is life saving.
17. Criteria used to select patients for elective shunt surgery
Prime candidates are patients under 60 years with good
hepatic and nutritional status according to Child’s classifi-
cation.
Child’s classification
A B C
Serum bilirubin mg/dl ≤ 2 2-3 ≥3
Serum albumin gm/dl ≥ 3.5 3-5 ≤3
Ascites Nil Minimal Severe
poorly controlled
Neurological disorder Nil Minimal Advanced
Nutrition Excellent Good Poor

Patients in class A are good risk cases, while those in C have


highest operative mortality.
The operative mortality in elective porto-systemic shunts is
5% for Child’s A, 10% for Child’s B and 20-40% for Child’s C
patients.
Rebleeding occurs in approximately 5%, usually due to
thrombosis of the shunt.
18. Risk and benefit of emergent surgery in acute variceal bleed
Emergency surgery in variceal bleed are associated with a
mortality of 30-50%. Hepatic failure accounts for two-third
of these deaths. It is worth a trial in face of exsanguination.
19. Pathogenetic mechanism of ascites formation in cirrhosis
1. Increased hydrostatic pressure in portal circulation
2. Increased sodium and water retention, secondary to
hyperaldosteronism and increased ADH secretion
3. Impaired hepatic and splanchnic lymph removal due to
raised hepatic sinusoidal pressure.
4. Hypoalbuminemia with decreased intravascular pressure
5. Raised plasma vasopressin and epinephrin.
20. Surgical option in resistant ascites of cirrhosis
• Peritoneovenous shunt (Le Veen).
• TIPS
166 SHORT NOTES AND SHORT CASES IN SURGERY

21. What is esophageal stapling?


It is a simple way of interrupting dilated varices by passing a
stapling device through a small hole in anterior wall of
stomach. When operated, the instrument simultaneously
divides esophagus and joins it together again with staples,
which occlude the vessels.
22. Endoscopic treatment of choice in varices
Banding is the endoscopic treatment of choice since it achieves
lower rates of rebleeding complications and death than
sclerotherapy.
23. Balloon tube tamponade in variceal bleed
Sengstaken-Blackmore or Minnesota tubes provide initial
control of variceal bleed in 60-90% cases but rebleeding occurs
in 50%. Esophageal ulceration, and perforation are dangers.
24. Why vasopressin has fallen out of favour?
Octreotide is superior to vasopressin in control of variceal
bleed and controls bleed is 80% of cases. It does not compro-
mise cardiac circulation and mesenteric ischaemia.
Case 17

A 40-year-old man who had recurrent urinary stones has been


admitted with bone pain, polyuria, polydypsia and pain in the
epigastrium. Serum and urine calcium are elevated and blood
alkaline phosphatase is normal.
1. Which disease can have such protean manifestations?
Hyperparathyroidism.
2. Anatomy of parathyroid glands
The parathyroid glands, four in number are oval 0.5 cm in
size, lie closely applied to thyroid gland within its capsule
on either side of neck. The upper pair, more constant in
position, lie close to cricothyroid articulation above termina-
tion of inferior thyroid artery. The lower pair may lie along-
side or within thymus, behind trachea and great vessels.
3. Hormones of parathyroids and their functions
The chief cells fo parathyroid produce parathermone, a 84
amino acid polypeptide, which in conjunction with vitamin
D3 is responsible for maintenance of calcium, phosphorus,
magnesium balance and for integrity and normal minerali-
zation of bone. Parathermone stimulates osteoclastic activity,
increases reabsorption of calcium by renal tubules, augments
absorption of calcium from gut and promotes phosphate
excretion by kidneys.
4. Types of hyperparathyroidism
1. Primary hyperparathyroidism due to hyperplasia, ade-
noma or rarely carcinoma.
2. Secondary hyperparathyroidism associated with chronic
renal failure and malabsorption syndrome, that cause
hypocalcemia, which is a potential stimulus for parathyroid
hyperplasia.
3. Tertiary hyperparathyroidism, where parathyroids are
autonomous, not responding to physiologic stimuli.
168 SHORT NOTES AND SHORT CASES IN SURGERY

5. Characteristics of parathyroid adenoma


A single adenoma is the most common finding. It is multiple
only in 6% cases. In secondary hyperparathyroidism, all the
four glands are enlarged. Adenomatous gland is enlarged,
firm, dark and vascular with histological appearance as
hyperplasia. ABO(H) cell surface antigens are lost from
parathyroid cell surface in adenoma and carcinoma, but not
in hyperplasia.
6. Types of clinical presentation in hyperparathyroidism
1. Asymptomatic hypercalcemia
2. Classical tetrad of Bones, stones, abdominal groans and
psychic moans in 50% cases
3. Non-specific symptoms like weakness, thirst, anorexia,
polyuria (hypercalcemia induced).
7. Laboratory findings in hyperparathyroidism
1. Raised serum calcium
2. Low serum phosphorus
3. Increased 24 hours urinary excretion of calcium
4. Elevated serum alkaline phosphatase in bone involvement
5. Elevated serum parathormone (by radioimmunoassay).
8. Pre-operative localisation of parathyroids
• Ultrasound
• CT scan (the best for mediastinal glands)
• Thallium—Technetium isotope subtraction scan
• Selective angiography and selective venous sampling when
above methods fail.
9. Bone changes in hyperparathyroidism
• Generalised decalcification
• Single or multiple cysts, pseudotumor (brown-tumor of
jaw)
• Pathological fracture
• Loss of lamina dura, chondrocalcinosis.
Early X-ray changes appear in skull bones and phallanges
with loss of density and subperiosteal erosions.
10. Renal changes in hyperparathyroidism
• Nephrocalcinosis (stone formation in renal parenchyma)
• Repeated urolithiasis
• Polyuria and polydypsia secondary to hypercalcemia.
CASE – 17 169

11. Manifestations of hypercalcemia


• Thirst, anorexia, vomiting
• Recalcitrant peptic ulceration
• Constipation, weakness, paresthesia
• Band keratopathy
• Intense pruritus
• Pancreatitis
• Nephrocalcinosis with polyuria, polydypsia
• Chondrocalcinosis.
12. Differential diagnosis of hyperparathyroidism (hypercalce-
mia)
• Multiple myeloma
• Sarcoidosis
• Vitamin D intoxication
• Osteolytic bone metastasis
• Carcinoma with endocrine secretion (Ca of bronchus, ovary,
kidney).
13. Treatment options for hyperparathyroidism
• Surgical exploration and excision
• Alcohol injection into hypertrophied/adenomatous gland
• Autotransplantation of parathyroid tissue into arm in
tertiary hyperparathyroidism of chronic renal failure on
dialysis for easy follow up.
14. Treatment modalities for hypercalcemia
Normal saline 300 ml/hr, with KCl and Mg and furesemide
Mithramycin 15 g/kg. IV then every 3-7 days
Prednisolone—60 mg daily in hypercalcemia due to neoplasm
or sarcoidosis.
Calcitonin—IV infusion
Diphosphonates—Orally to inhibit osteoclasts
Oral phosphates only in resistant cases.
15. What are ‘APUD’ cells and their relation to MEA?
APUD is the acronym for amine precursor uptake and
decarboxylation. APUD cells irrespective of their location
have this common chemical characteristic. The cells under
this category are in parathyroid, thyroid, adrenal and
pancreas. A case of hyperparathyroidism may have multiple
endocrine adenomas in the above sites and the disease then
has a familial tendency.
170 SHORT NOTES AND SHORT CASES IN SURGERY

Type I: There is hyperplasia of parathyroid, pancreatic islets,


pituitary (chromophob cells), thyroid and adrenal cortex.
Type IIa: Parathyroid hyperplasia, pheochromocytoma,
medullary carcinoma of thyroid.
Type IIb: It is Type IIa plus mucosal neuroma (lumpy and
bumpy) lip, megacolon, ganglioneuromatosis.
16. Pathology of primary hyperparathyroidism
Single adenoma 85%
Double adenoma 3%
Diffuse hyperplasia 11%
Cancer < 1%
17. Cause of hypocalcemia after parathyroid surgery
• Hungry bone syndrome
• Edema of viable parathyroid tissue.
18. Physical symptoms of hypocalcemia
i. Chvostek’s sign—Tapping of facial nerve, infront of ear
causes contraction of orbicularis oris.
ii. Trousseau’s sign—Occluding the brachial artery for 3
minutes by BP cuff causes carpal spasm.
iii. Erb’s sign—hyper excitability of muscles to electric stimu-
lation
19. Consequence of hypoparathyroidism
Cataract (80%)
Mental deterioration
Paresthesia
Irritability
20. Who was Captain Martell?
Captain Martell was an officer in US Navy, who lost 6” of
height due to progressive hyperparathyroidism.
Case 18

A 65-year-old male has been admitted with complaints of nocturia,


urgency and increased frequency of micturition. Urinary stream
is poor and on straining is reduced further. Urine examination
shows few RBCs.
1. What is the probable diagnosis?
Benign hyperplasia of prostate.
2. What causes benign prostatic enlargement?
1. Hormonal theory—prostate enlarges because of predomi-
nance of estrogenic hormones in old age, consequence to
withdrawal of male hormones. It is akin to fibroadenosis
of breast.
2. Neoplastic theory—it prostulates that prostatic enlarge-
ment of old age is a neoplasm i.e., fibromyoadenoma.
3. Surgical anatomy of prostate
Prostate is chestnut shaped, weighing 15 gram and surrounds
the proximal male urethra. It is firmly attached to bladder
neck and symphysis pubis. The blood supply is from inferior
vesical and middle haemorrhoidal arteries. The periprostatic
venous plexus drains to hypogastric veins and also
communicates with Baston’s veins. Just, lying below the
peripheral capsule are the long bronched prostatic glands
proper. Deeper to these glands are the submucosal glands.
Surrounding urethra are the unbranched urethral glands.
Prostate has two lateral lobes and one median lobe (above
the ejaculatory duct).
4. Which areas of prostate are involved in benign enlargement
and cancer?
The submucosal glands and adjoining tissue enlarge in benign
hypertrophy. The median lobe contains more secretory
glandular elements and is more prone for benign hyper-
trophy. The subcapsular prostatic glands in the periphery is
the carcinomatous zone.
172 SHORT NOTES AND SHORT CASES IN SURGERY

5. Secondary effects of prostatic enlargement


1. Elongation of prostatic urethra, converted to an antero-
posterior slit.
2. Hypertrophy of bladder wall, trabeculation, diverticula
formation, increased residual urine, tendency for infection
and calculus formation.
3. Ureteral dilatation, hydronephrosis, renal failure.
4. Increased libido initially, but impotency in advanced
disease
5. Urinary retention.
6. Findings in rectal examination in benign hypertrophy
1. Ninety per cnet of cases show smooth symmetrical convex,
elastic enlargement of prostate, with consistency that of
thenar eminence; the rectal mucosa easily movable over
the gland.
2. Ten per cent have only enlargement of median lobe intra-
vesically, with a normal gland on rectal examination.
3. Increased residual urine can be palpated as a cystic floctua-
ting swelling above prostate.
4. Often the hypertrophied full bladder may depress the
prostate, which feels relatively larger.
7. Mechanism of urinary retention
1. Reduction in cross sectional area of prostatic urethra.
2. Compromised internal sphincter of bladder neck by up-
ward enlarged median lobe and often the lateral lobes.
3. Detrusor decompensation.
8. Cause of haematuria in BPH
• Ruptured prostatic venous plexus (vesical piles)
• Erosion of enlarged prostate
• Urinary tract infection.
9. What is micturograph?
A graphic record of patient’s stream, rate and volume of urine,
to know the degree of outflow obstruction is called
micturograph. The normal flow rate is 20 ml/sec. when more
than 200 ml of urine is voided. In prostatic hypertrophy, this
is reduced to 5 ml/sec. or less.
CASE – 18 173

10. Role of ultrasound in BPH


US is very helpful in the following ways:
1. Determination of size and weight of prostate. Normal
prostate weighs less than 19 gm.
2. Estimation of residual urine.
3. Documentation of bladder wall hypertrophy, pulsion
diverticula, hydronephrosis and bladder stone.
4. Condition of prostatic urethra, prostatic calcification.
11. Indications for surgical intervention in BPH
1. Significant residual urine (≥ 50 ml).
2. Annoying frequency, nocturia, poor stream
3. Secondary effects like bladder diverticula, recurrent urinary
infection, hydronephrosis.
12. Surgical options for BPH
• Transurethral resection
• Open enucleation
• Thermal destruction
• Balloon dilatation
13. Advantages of transurethral resection
• Low mortality and morbidity (4 times less than that of open
surgery).
• Inflammed and small glands do not enucleate easily in open
route.
• Young persons in whom sexual potency is valued. Open
procedure may damage parasympathetic supply and hence
poor erection.
14. Types of open prostatectomy
• Transvesical
• Retropubic
• Perineal.
15. Complications of prostatic surgery
• Haemorrhage
• Incontinence when external sphincter is also damaged
(bladder neck is made incompetent by prostatic surgery)
• Retrograde ejaculation
• Bladder neck contracture
• Stricture urethra.
174 SHORT NOTES AND SHORT CASES IN SURGERY

16. Why open prostatectomy is termed enucleation?


The adenomatous zone compresses the surrounding prostatic
tissue, containing the prostatic glands proper into a false
capsule. The surgeon during open surgery enucleates the
adenomatous tissue, leaving behind the false capsule (surgical
capsule) intact.
17. How urethral patency is maintained after prostatic surgery?
Since prostatic urethra including the adenomatous zone are
removed, the cavity, thus left is epithelialized from bladderr
neck, from urethral epithelium and from ducts of acinar
glands remaining in surgical capsule.
18. Types of presentation in carcinoma prostate
1. Asymptomatic, discovered only after examination of
excised prostate. Twenty per cent of enucleated prostates
for BPH show malignancy.
2. Hard nodule with adherent rectal mucosa in PR exami-
nation (50% of these are malignant).
3. Symptoms like backache, sciatica due to osseous metastasis.
4. High acid phosphatase with non-specific symptoms.
19. Causes of raised serum acidphosphatase
• Carcinoma prostate with metastasis
• Acute prostatitis
• Paget’s disease of bone
• Cirrhosis of liver.
20. Spread of prostatic cancer
1. Local spread to seminal vesicles, base of bladder, prostatic
urethra, rectum (stricture).
2. Pelvic bones and lower lumbar vertebra by blood borne
metastasis.
3. Lymphatic metastasis to lymphnodes along internal iliac
vein and in hollow of sacrum; external iliac lymphnodes.
21. Tumors with skeletal metastasis
Carcinoma of prostate, breast, kidneys, bronchial tree and
thyroid gland in order of frequency.
22. Differential diagnosis of a firm-hard prostate
• Chronic prostatitis
• Prostatic calcification
• Prostatic malignancy
• Tuberculosis of prostate.
CASE – 18 175

23. TNM classification of prostatic cancer


Nodes
N0—no evidence of involvement of regional nodes.
N1—involvement of one regional node.
N2—involvement of several regional nodes.
N3—fixed regional nodes.
N4—involvement of common iliac and para-aortic nodes.
Metastasis
M0—no evidence of distant metastasis
M1—distant metastasis.
24. Characteristics of bone metastasis in cancer prostate
Metastasis is usually sclerotic (osteoblastic). The bone is dense
and coarse resembling Paget’s disease.
25. Role of prostate specific antigen (PSA) in management of
prostatic cancer
PSA has greater specificity for prostate cancer. It is useful for
following the pattern and extent of disease and its relapse.
26. Treatment of prostatic cancer
• Radical prostatectomy (removal of prostate, seminal vesicle,
orchiectomy).
• Pelvic lymphnode dissection and 131I implantation into
prostate.
• Radiation to prostatic bed and pelvic lymphnodes.
27. Drug treatment of disseminated prostate cancer
• Stilbestrol
• Nitrogen mustard
• Progesterone
• LHRH analog
28. Disadvantage of stilboestrol therapy
• Salt and water retention
• Gynaecomastia, loss of libido, impotence, nausea
• Increased tendency to thrombosis (venous thrombosis,
myocardial infarction)
29. Causative agents of chronic prostatitis
Predominant organisms are E. coli, staphylococci, streptococci,
diphtheroids, trichomonas, chalmydia, and gonococci.
176 SHORT NOTES AND SHORT CASES IN SURGERY

30. Symptoms of chronic prostatitis


• Dull pain in the peineum or rectum referred to low back
• Acute or subacute non-tuberculous epididymitis
• Recurrent attacks of mild pyrexia
• Sexual dysfunction—premature ejaculation, impotency
• Chronic posterior urethritis
• Recurrent haematuria.
31. Role of prostatic massage in diagnosis of chronic prostatitis
Normal prostatic fluid in slightly opalescent and viscid. A
purulent discharge, showing organisms and pus cells and
finally culture for isolation of organisms are definitive.
32. What is Cowperitis?
Acute inflammation of Cowper’s gland can be diagnosed
from rectal examination, when there is excruciating pain on
sides of perianal raphe. Spontaneous rupture can cause fistula
formation.
33. Which is the best modality for diagnosis of prostatic
cancer—ultrasound, needle biopsy or NMR?
NMR cancer tissue characterisation is dependable and can
provide early clue to metastasis.
Case 19

A 55-year-old lady has presented with a history of a slowly growing


non-tender nodule in the left breast. Examination reveals a hard
nodule in the outer upper quadrant fixed to the skin. There is
shrinking and elevation of the breast with nipple retraction.
1. What is the diagnosis?
Schirrous carcinoma of left breast.
2. Pathological classification of breast cancer
1. Schirrous carcinoma (70-80%), i.e. anaplastic cells
producing stromal fibroblastic reaction.
2. Colloid carcinoma (5%) where cancer cells produce
abundant mucin.
3. Medullary carcinoma (3-5%) where solid sheets of large
cells are associated with marked lymphocytic reaction.
3. Relationship of breast carcinoma to different quadrants of
breast
Upper inner quadrant 15%
Upper outer quadrant 60%
Lower inner quadrant 05%
Lower outer quadrant 15%
Around nipple 05%
4. Modalities for early detection of breast carcinoma
• Self examination of breast with palms in all women after
35 years of age
• Mammography, MR mammography
• Zero radiography and thermography
• Ultrasound
• Needle aspiration cytology of all breast nodules.
5. How effective is mammography in early diagnosis?
Mammography can detect breast cancer 2-4 years earlier than
clinical detection. The most common mammographic
abnormalities associated with carcinoma of breast are
clustered polymorphic microcalcification. Such calcifications
178 SHORT NOTES AND SHORT CASES IN SURGERY

are 5-8 in number, aggregated in one part of breast, differing


from each other in size and shape (branched, V or Y shaped).
There may be an associated mass density usually with
irregular border.
6. Groups of lymphnodes involved in breast tumor metastasis
• Axillary lymphnodes
• Internal mammary nodes
• Supraclavicular nodes
• Mediastinal nodes (very late).
7. Mechanism of lymphatic spread
1. Tumor emboli
2. Lymphatic permeation, i.e. growth of cancer cells along the
lymphatic channels.
8. Lymphatic drainage of breast
1. From subareolar plexus of Sapey and outer quadrants to
pectoral, then central and finally apical group of axillary
nodes.
2. From inner upper quadrants to internal mammary group
and other breast.
3. From lower inner quadrant to subperitoneal lymph plexus
and liver.
9. Pre-operative staging of breast cancer
Stage I—Tumor ≤ 1 cm confined to breast
Stage II—Tumor 2-5 cm with palpable axillary lymphnodes
Stage III—Tumor ≥ 5 cm palpable fixed axillary lymphnodes
Stage IV—Distant metastasis, extension to skin, fixity to
pectoralis, inflammatory carcinoma.
10. Which characteristics of breast lump indicate malignancy
and benignity?
Discrete, smooth tender nodules, and that change in size
during menses indicate benign lesion. Malignant lesions are
firm to hard, non-tender with irregular or ill defined margins.
11. Risk factors for breast cancer
• Early menarche
• Late menopause
• Nulliparity
• First childbirth after age of 35 years
CASE – 19 179

• Mother or sister having breast cancer


• Mammary dysplasia with atypical epithelial hyperplasia
• Positive endometrial cancer
12. Indications for mammography
1. To evaluate questinable or suspicious breast mass.
2. Evaluation of each breast in detail when diagnosis of poten-
tially curable breast cancer has been made.
3. To search for occult primary in metastatic disease in axillary
lymphnodes, bone or liver.
4. To screen women prior to cosmetic breast surgery.
5. To screen women, who are high risk of developing breast
cancer.
6. To follow those women with breast cancer who have been
treated with breast conservative surgery and radiation.
13. Limitations of mammography
1. A small mass or architectural distortion particularly in a
young breast may be missed.
2. Young women with mammary dysplasia be evaluated by
biopsy than mammography.
3. Mammography may not reveal medullary cancer.
14. Time of mammography as a screening procedure
1. Baseline mammography in all women between 35-40 years.
2. Those between 40-49 years should have mammogram every
1-2 years.
3. Women above 50 year should have mammogram every
year.
About one-third of abnormalities detected on screening mam-
mograms will found to be malignant on biopsy.
15. Characteristics of advanced breast carcinoma
1. Edema, nodularity, redness or ulceration of skin
2. Large primary tumor
3. Tumor fixed to chestwall
4. Marked axillary lymphadenopathy
5. Edema of ipsilateral arm (axillary vein invasion, or obs-
tructed lymph flow)
6. Supraclavicular lymphadenopathy
7. Involvement of brachial plexus
8. Distant metastasis
180 SHORT NOTES AND SHORT CASES IN SURGERY

16. Characteristics of axillary nodes in carcinoma breast


a. One or two movable axillary nodes of 5 mm or less in size
are of no clinical significance.
b. Biopsy of these clinically negative nodes may show
micrometastasis in 30% cases.
c. Firm or hard nodes ≥ 5 mm. certainly contain metastasis.
d. Nodes matted or fixed to skin/deeper structures indicate
advanced disease.
17. Investigations advisable in breast cancer
• ESR high in disseminated disease
• Raised alkaline phosphatase in bone/liver metastasis
• Hypercalcemia in advanced disease
• Positive carcinoembryogenic antigen; may be used for
follow up for recurrence
• Chest X-ray for pulmonary metastasis
• Tc 99 m bone scan for skeletal metastasis.
18. Efficacy of clinical suspicion vs needle biopsy in diagnosis
of breast cancer
1. About 30% of lesions thought to be cancer prove to be
benign on biopsy.
2. Fifteen per cent of lesions thought clinically benign prove
to be malignant on biopsy.
3. Needle biopsy has a false negative rate of 5-10%.
All biopsy specimens from beginning be preserved for deter-
mination of estrogen and progesterone receptors.
19. What is Paget’s carcinoma of breast?
The basic lesion is usually an infiltrating ductal carcinoma
with infiltration of nipple epithelium. Itching and burning of
nipple; superficial erosion or ulceration first draw attention,
often misdiagnosed as dermatitis or bacterial infection. When
palpable tumor is absent with only nipple changes, incidence
of axillary node metastasis is 5%.
20. What is inflammatory carcinoma?
This is the most malignant form of breast cancer and
constitutes less than 3% of all cases. The involved breast is
painful, with rapidly enlarging mass, the overlying skin being
edematous, red and warm due to early invasion of dermal
lymphatics. Metastasis occur early and widely; hence surgery
CASE – 19 181

is rarely helpful. It is a dictum that any inflammatory lesion


of breast not responding to 1-2 weeks of appropriate antibiotic
therapy be biopsied to exclude inflammatory carcinoma.
Radiation, hormone therapy and anticancer chemotherapy
are of limited use and prognosis is grave.
21. Bilaterality of breast cancer
Clinically evident simultaneous bilateral breast cancer occurs
in less than 1% patients, but subsequently other breast may
be involved in 5-8% cases. Bilaterality occurs more often in
women under 50 years of age, and more often when the tumor
in the primary breast is lobular. The incidence of second breast
cancer increases directly with the length of time patient is
alive, after her first cancer—about 0.5% per year.
22. Hormone receptor sites in breast cancer
Presence of estrogen and progesterone receptors in cytoplasm
of breast tumor cells is of immense value in management of
recurrent or advanced disease by hormonal manipulation.
Upto 60% of patients, with metastatic breast cancer will
respond to hormonal manipulation, if tumor cells are positive
for estrogen receptors. Estrogen receptor positive tumors have
a better prognosis. Upto 80% of metastatic progesterone
receptor positive tumors respond to hormonal manipulation.
Receptors however, have no relationship to chemotherapeutic
response.
Receptor assay also helps in selection of adjuvant therapy.
Receptor status change after radiotherapy, chemotherapy and
hormonal therapy.
23. What constitute clinically, early operable breast cancer
(stage I and II)?
Tumor of < 4 cm in a patient with no evidence of metastatic
disease, inflammatory skin changes, fixity to chestwall,
fixation of axillary nodes, arm edema or multiple tumors.
24. What is modified radical mastectomy?
Remvoal of all breast tissue and resection of axillary
lymphnodes.
25. What is radical mastectomy?
Modified radical mastectomy plus removal of sternal portion
of pectoralis major with its sheath, pectoralis minor with its
sheath and costocoracoid membrane.
182 SHORT NOTES AND SHORT CASES IN SURGERY

26. Limitations of radical mastectomy


1. Internal mammary nodes are not dissected since they are
involved, when the primary tumor lies in central area or
medial quadrants.
2. It overtreats axilla, when these nodes are uninvolved
leading to complications like edema of upper limb and stiff
shoulder.
3. Removal of pectoralis major adds to functional disability.
27. What is extended radical mastectomy?
It is radial mastectomy, that includes resection of internal
mammary lymphnodes, but it does not improve survival. It
rather adds to morbidity.
28. Indications for breast conserving therapy
Postmenopausal women with tumor size < 4 cm, provided it
can be safely excised and clinically negative axillary nodes.
Breast conservative therapy includes lumpectomy/qyad-
rantectomy, radiation to remaining breast and axillary node
dissection.
29. Role of adjuvant therapy in breast cancer
The aim of adjuvant therapy is to eliminate occult metastasis
responsible for late recurrence. Since two-third of patients
eventually develop distal disease regardless of the form of
primary therapy, breast carcinoma is thought to be multifocal.
Adjuvant therapy includes cyclophosphamide, methotrexate
and fluorouracil given IV on day 1 and 8 every month for six
months. CMF regime is best reserved for premenopausal
women with axillary metastasis. Postmenopausal women
who are—estrogen receptor positive, should be given tamoxi-
fen alone or in combination with adriamycin, and cyclophos-
phamide or prednisolone, adriamycin and fluorouracil.
Adjuvant chemotherapy guidelines.
Premenopausal women
Nodal Estrogen Adjuvant therapy
involvement receptor
Yes Positive Combination chemotherapy
Yes Negative -do-
No Positive Tamoxifen
No Negative Combination chemotherapy
contd..
CASE – 19 183

contd...
Postmenopausal women
Nodal Estrogen Adjuvant therapy
involvement receptor
Yes Positive Tamoxifen
Yes Negative Combination chemotherapy
No Positive Tamoxifen
No Negative Combination chemotherapy
Adjuvant systemic chemotherapy is not indicated for patients
with negative lymphnodes with favorable DNA studies.
30. Prognostic factors in node negative breast cancer
Increased recurrence of breast tumor occurs in the following
categories
1. Tumor size > 5 cm
2. Negative hormone receptors
3. Aneuploid pattern in DNA flow cytometry
4. Highly undifferentiated cells
5. Tumor labeling index < 3%
6. S-phase fraction >5%
7. Lymphatic and vascular invasion
8. High cathepsin D
9. High HER—neu oncogene
10. High epidermal growth factor receptors
31. Treatment of advanced breast cancer
1. Radiotherapy—Palliation for locally advanced disease with
distant metastasis; for isolated bony metastasis.
2. Hormone therapy.
a. Tamoxifen (antiestrogen) 10 mg twice daily.
b. Diethylstilboestrol (estrogen) 5 mg three times daily
c. Megestrol (progestin) 40 mg 4 tiems daily.
d. Aminoglutethimide (aromatase inhibitor) 250 mg 4 times
daily.
Hormonl manipulation is more effective in postmenopausal
women, but response is equal in both pre and postmeno-
pausal groups provided hormone receptor status is positive.
Nearly one-third of patients with metastasis respond
favorably to hormonal manipulation.
184 SHORT NOTES AND SHORT CASES IN SURGERY

32. Approximate survival (%) of patients with breast cancer by


TNM stage
TNM stage Five years Ten years
0 95 90
I 85 70
IIA 75 60
IIB 70 40
IIIA 55 30
IIIB 30 30
IV 5 2
Overall 65 30

33. “QUART”
QUART denoteo combined approach consisting of (1)
quadrantectomy, (2) axillary dissection and, (3) radiotherapy
34. Eczema vs Paget’s disease of breast
Eczema Paget’s disease
1. Seen in lactating women At menopause
2. Bilateral Unilateral
3. Vesicles present Absent
4. Itchy Nonitchy
5. Intact nipple Destroyed
6. No lump Palpable lump deep
to areola
7. Axillary nodes normal Axillary nodes involed
35. Huge enlargement of breast
• Elephantiasis,
• Cystosarcoma phylloides
• Mastitis carcinomatosa
• Colloid carcinoma
• Sarcoma of breast
Case 20

A 35-year-lady has been admitted with weight loss, hypomenor-


rhoea, palpitations and preference for cold. Examination reveals
fine tremor and nodular enlargement of thyroid.
1. What do you think of this case?
Toxic nodular goiter.
2. Principal causes of hyperthyroidism
1. Diffuse toxic goiter (Grave’s disease)
2. Toxic nodular goiter (Plummer’s disease)
3. Thyroiditis with transient thyrotoxicosis
3. Characteristics of Grave’s disease
• Eight times more common in female
• Diffuse hyperplasia of thyroid with thrill and bruit
• Exophthalmos
• Cardiac rhythm disturbances (PSVT, AF)
• Weight loss, inspite good appetite
• Tremor, sweating, preference for cold
4. Which manifestations are not the part of hyperthyroidism
per se?
• Orbital proptosis
• Ophthalmoplegia
• Pretibial myxedema
5. Characteristics of secondary thyrotoxicosis
• Insidious onset
• Symptoms less severe
• Eye signs rare
• Cardiac symptoms predominate often with CHF.
6. Natural history of toxic nodule
A toxic nodule may be a true toxic adenoma or a part of
generalized nodularity. In multinodular goiter, which is end
result of simple goiter, either sporadic or endemic, one or more
nodules become autonomous, while others are hypofunc-
tioning due to excess thyroxin produced from the autonomous
nodule that suppresses pituitary TSH.
186 SHORT NOTES AND SHORT CASES IN SURGERY

7. What are thyroid nodules?


Nodules may be true adenoma or may be colloid, or cellular
and cystic degeneration/haemorrhage with subsequent calci-
fication. These nodules are smooth and firm, painless, moving
freely on deglutition. Hardness and irregularity due to
calcification may simulate carcinoma. Thyroid carcinoma may
present as a nodule, which is hard, painful, irregular with
rapid growth. Haemorrhage into a simple nodule can have
similar presentation. A toxic adenoma is rarely malignant.
8. Role of ultrasound in assessment of thyroid swelling
US remains a preliminary investigation of nodular thyroid
swellings. It can tell whether the nodule is cystic or solid.
Cystic nodules are always benign. US guided biopsy of solid
nodules can detect malignancy and follicular adenoma.
9. Role of isotope scan in thyroid nodule
Isotope scan helps to differentiate nodules to be hot, warm or
cold; only 20% of cold swellings are malignant.
10. Role of FNAC in tissue diagnosis of thyroid nodule
FNAC can differentiate follicular adenoma and malignant
nodule from benign lesons. Ten to twenty per cent of clinically
isolated swellings turnout to be malignant; and additional
30% are follicular adenomas. The remainder are benign, the
large majority being areas of colloid degeneration.
11. Importance of thyroid cysts
Over 30% of clinically isolated thyroid swellings are cystic or
partly cystic in FNAC. Tense cystic swellings are hard on
palpation often mistaken for carcinoma. About 50% of cystic
swelling are either due to colloid degeneration or are of
uncertain origin. Most of the remainder are due to degenera-
tion in a follicular adenoma. About 30% cystic swellings in
male and 10% in female are malignant.
12. Limitations and advantages of FNAC
FNAC can diagnose colloid nodules, thyroiditis, papillary
carcinoma, medullary carcinoma, anaplastic carcinoma and
lymphoma. FNAC can not distinguish between a benign
follicular adenoma and follicular carcinoma as this distinction
is dependent not on cytology but on histological criteria,
which includes capsular and vascular invasion.
CASE – 20 187

FNAC has a false negative diagnosis of 1-10% and false


positive diagnosis of 0-2%.
13. Main types of solitary thyroid nodules
1. Nodular goiter with one dominant nodule 50%
2. Adenoma 20%
3. Cancer 20%
4. Thyroiditis 5%
5. Cysts 5%
14. Histologic types of thyroid cancer
1. Papillary (ground glass nuclei, psammoma 70%
bodies)
2. Follicular 15-20%
3. Medullary 5%
4. Anaplastic and lymphoma 5%
15. Common kind of thyroiditis found within nodule
Lymphocytic thyroditis. Although Hashimoto’s thyroiditis
appears as a diffuse rubbery goiter, it can arise asymmetrically
mimicking solitary thyroid nodule.
16. Treatment of larger papillary cancer
Total or near total thyroidectomy with local excision of
involved lymphnodes and post-operative radioiodine, if any
remaining tissue takes up radioiodine. A patients with distant
metastasis requires same treatment. Excision of normal
thyroid enhances radioiodine uptake by metastasis.
17. Difference in nodes of metastasis between follicular and
papillary carcinoma
Follicular carcinoma metastasizes by haematogenous (distant
metastasis), while papillary carcinoma has local metastasis
first by lymphatic route.
18. Ten year survival rates for thyroid cancer
1. Papillary 85%
2. Follicular 85%
3. Localised with metastasis 20%
3. Medullary
• Negative nodes 85%
• Positive nodes 45%
4. Anaplastic 0%
188 SHORT NOTES AND SHORT CASES IN SURGERY

19. Difference between papillary and follicular carcinoma


Papillary Follicular
• Young female Elderly male
• Nodal metastasis common Uncommon
• Non encapsulated Commonly encapsulated
• Hormone dependent Harmone independent
• Multiple intraglandular Uncommon
foci common
• Local recurrence more Local recurrence less
• Haematogenons spread Haematogenous
less common spread more common
20. Complications of thyroid surgery
• Haemorrhage
• Respiratory obstruction due to laryngeal edema, or tense
haematoma
• Recurrent laryngeal nerve palsy.
21. Nodular goitre vs true adenoma
Nodular goitre True adenoma
• Multiple nodules in Single nodule in normal
enlarged thyroid sized thyroid
• Poorly encapsulated Well encapsulated
• Variable structures Uniform cell pattern
• FNAC reveals FNAC reveals plenty
scanty colloid cellular material
Case 21

A 40-year-old male complains of a slowly enlarging tumor at angle


of jaw. Examination reveals a hard irregular tumor fixed to deeper
structures. Movement of jaw is limited and there is facial palsy.
1. What do you think of this growth?
Malignant parotid tumor.
2. Classification of salivary gland tumors
1. Epithelial tumors
a. Adenomas (pleomorphic and monomorphic)
b. Mucoepidermoid tumors
c. Acinic cell tumors
d. Carcinomas (adenocarcinoma, epidermoid carcinoma,
adenocystic carcinoma)
2. Non-epithelial tumors
3. Statistical data of salivary neoplasms
• Seventy-five per cent salivary neoplasms arise in parotids
• Eighty per cent of these are benign and 80% of benign
tumors are pelomorphic adenoma
• Fifteen per cent salivary neoplasms arise in submandibular
gland; of these 60% are benign and 9% of these benign
tumors are pleomorphic adenoma.
• Ten per cent of salivary neoplasms arise in minor salivary
glands of palate, lips and cheeks and sublingual glands.
Forty per cent of them are benign and all the benign ones
are pleomorphic adenoma.
4. Histological characteristics of pelomorphic adenoma
As the name implies, the histological appearance is a picture
of epithelial cell proliferation in strands or a duct like
arrangement, myoepithelial cell proliferation in sheets,
mucoid material interspersed between cells, and invasion of
the capsule by tumor cells.
5. Most common location of parotid tumor
In the superficial lobe just beneath the lobe of ear.
190 SHORT NOTES AND SHORT CASES IN SURGERY

6. Most common cause of a mass in front of tragus of ear


An enlarged parotid lymphnode. In older patients, such a
mass must be considered site of metastatic cancer, unless
proved otherwise.
7. What is Warthin’s tumor?
Otherwise known a adenolymphoma, it is a benign parotid
tumor composed of double layered epithelium lining cystic
spaces, often folded to give papillary appearance. The stroma
contains lymphoid tissue. The tumor usually arises from lower
pole of parotid and unlike other parotid neoplasms, which
form “cold spot” in 99MTc scan, it produces `hot spot’ thus
obviating the need of a biopsy for preoperative diagnosis.
8. Types of malignant parotid tumor in order frequency
Mucoepidermoid carcinoma, malignant mixed tumor, acinic
cell carcinoma, adenocarcinoma, adenoid cystic carcinoma
and epidermoid carcinoma.
9. Is nerve palsy a contraindication for surgery?
No, tumor can be resected and nerve biopsy be performed. If
nerve is involved, a nerve graft using greater auricular nerve
from opposite side is sutured to the portion of excised nerve.
10. Is radical neck dissection and excision of mandible required
in all cases?
No, only in high grade cancer, the upper cervical lymphnodes
are removed. Mandible is rarely touched, unless there is true
invasion of bone. Simple osteopenia of mandible in a parotid
tumor does not imply true invasion.
11. Role of radiotherapy in parotid cancer
Excepting low grade cancers, all other cases of malignant
neoplasms of protid after removal, radiation should be
employed. Unfortunately most salivary neoplasms are
relatively radioresistant.
12. Cure rate of parotid cancers
In low grade cancer cure rate is 80-90% . Overall 5, 10 and 15
years cure rate is 60%, 50% and 45% respectively.
13. Most common type of nerve injury following parotidec-
tomy
Injury to the ramus marginalis mandibularis, the lowest
branch of the facial nerve in the face innervating depressor
muscles in the lower lip.
CASE – 21 191

14. What is Frey’s syndrome or gustatory syndrome?


In this condition, there is flushing and sweating of the skin
innervated by auriculo temporal nerve upon eating. It occurs
in 30% or more patients undergoing parotidectomy. It can
occurs following accidental injury of parotid glands or
tempero mandibular joint. It is thought to arise from regene-
ration of parasympathetic fibres within auriculo temporal
nerve, resulting in stimulation of the sweat glands or para-
sympathetic fibres from ottic ganglion become united to
sympathetic nerves from superior cervical ganglion, destined
to supply vessels and sweat glands. The topical application
of antiperspirant aluminium chloride hexahydrate may be
tried.
15. What is sialosis?
Sialosis is enlargement of salivary glands due to metabolic
reasons, like obesity, diabetes, and acromegaly.
16. Should all salivary neoplasms be biopsied?
No, pre-operative biopsy is largely confined to fixed tumors
of sublingual, palatal and minor glands, which are mostly
malignant. Biopsy carries the risk of facial palsy (in parotid
biopsy), salivary fistula and increased risk of recurrence. Malig-
nancy can mostly be diagnosed on clinical grounds. Peroral
biopsy for parotids may be done, but not external one.
17. What are dumb-bell parotid tumors?
Some parotid tumors arising from deepr parts of gland
enlarge medially, passing between the styloid process and
mandible to present as a swelling of soft palate, lateral wall
of pharynx, but no visible or palpable swelling in preauricular
location. CT scan is best for diagnosis of these lesions.
18. Can parotitis and sialoadenitis enhance risk of parotid
malignancy?
No, only radiation to head and neck is an established risk
factor, that enhance occurrence of parotid neoplasms.
Case 22

A 40-year-old male has been admitted with complaints of heaviness


in scrotum. Examination reveals enlarged left testis, which is
smooth, firm and bossy. Epididymis is not palpable and vas is
normal. There is loss of testicular sensation.
1. What is the probable diagnosis?
Malignant testicular tumor.
2. Tyeps of testicular tumors
Seminous 40%
Teratoma 32%
Combined seminous and teratoma 14%
Interstitial tumor 1.5%
Lymphoma 7%
Other tumors 5.5%
3. Statistical data about testicular tumors
• 99% of testicular neoplasms are malignant
• Maldescent of testis predisposes to malignancy.
• Peak incidence of teratoma is between 20-35 years and that
of seminoma is between 35-45 years.
• 30% testicular tumors are painful.
4. Hormonal status of testicular tumors
• Leydig cell tumors produce estrogen (75%)
• Sertoli cell tumors produce estrogen and androgens in 30%
• 5-10% of seminomas and non-seminomas produce hCG or
estrogen in 90%.
5. Tumor markers of testicular malignancy
• LDH
• hCG
• Alfa-fetoprotein (by non-seminomas).
6. Sites of metastasis in testicular tumors
• Para-aortic lymphnodes at the origin of spermatic artery
• Lymphnode at bifurcation of common iliac artery
CASE – 22 193

• Mediastinal lymphnode and left supraclavicular lymp-


hnode
• Pulmonary metastasis from teratoma.
7. Clinical presentation of testicular tumors
I. Smooth firm testicular enlargement
II. Symptoms of metastasis i.e. abdominal and lumbar pain,
epigastric retroperitoneal mass in seminoma or chest pain,
dyspnoea and haemoptysis in teratoma.
III. Gynaecomastia due to increased estrogen and hCG;
epididymo-orchitis not clearing up with antibiotics.
8. Histological types of tetratoma
• Well differentiated teratoma
• Teratocarcinoma
• Trophoblastic malignant teratoma
• Anaplastic malignant teratoma.
9. Characteristics of interstitial tumors of testis
1. Leydig cell tumors arising in young children cause
masculinisation and precocious puberty.
2. Sertoli cell tumors are feminising producing gynaeco-
mastia, loss of libido and aspermia in adults.
10. Staging of testicular tumors
Stage I—Lesion confined to testis.
Stage II—Nodes involved below diaphragm.
a. < 2 cm diameter
b. 2-5 cm diameter
c. > 5 cm diameter.
Stage III—Nodes above diaphragm involved.
Stage IV—Pulmonary or hepatic metastasis.
11. Radiological investigations in testicular tumors
1. X-ray chest for pulmonary metastasis (canon ball shadow)
2. Lymphangiography
3. Gallium scan for seminoma metastasis
4. CT scan and US for para-aortic lymph nodes and liver meta-
stasis.
12. Plan of action in suspected testicular malignancy
1. Estimation of tumor markers to monitor subsequent
response to treatment.
2. X-ray chest for pulmonary metastasis.
194 SHORT NOTES AND SHORT CASES IN SURGERY

3. Orchiectomy by inguinal route with high ligation of cord


4. Histological examination for confirmation of diagnosis and
grade of tumor
5. CT scan and ultrasound to exclude metastasis.
13. Treatment strategy after orchiectomy
• Cisplatin for metastatic seminoma; alternatively radio-
therapy for Stage II disease.
• Chemotherapy (cisplatin, vincrystine, bleomycin, metho-
trexate) for Stage II-IV teratoma. Teratoma is radioesistant.
• Leydig and sertoli cell tumors are usually benign.
14. Prognosis in testicular cancer
Seminoma
Stage I—95%
II-IV—75%
Teratoma
Stage I, II—85%
III, IV—60%
15. Role of cytoreductive surgery in management of testicular
tumors
Since, seminoma in Stage II (c) recur following radiotherapy—
cytoreductive surgery for bulky abdominal disease is
recommended, provided blood vessels and ureter are not to
be damaged.
16. What will be your advise to a elderly patient suddenly
developing hydrocele?
Immediate ultrasound to detect on underlying seminoma,
since 10% of seminoma patients have secondary hydrocele.
17. Are all swollen epididymis inflammatory?
No, mesothelioma of epididymis may simulate chronic
epididymitis.
18. Common causes of chronic and acute epididymitis
Tuberculosis accounts for 90% cases of chronic epididymo-
orchitis; commonly secondary to disease of seminal vesicles.
The epididymis is firm, painless and craggy, often with a thick
beaded vas. The epididymis in non-specific epididymitis is
larger and smoother. Acute epididymitis can be gonococcal,
viral (mumps), filarial or chlamydial.
Case 23

A 40-year-old male is admitted with history of intermittent profuse


haematuria and colicy pain passing from loin to groin. X-ray chest
shows cannon ball opacity and blood examination shows PCV of
55% with Hb of 20 gm%.
1. What is the diagnosis?
Hypernephroma with pulmonary metastasis and polycythe-
mia.
2. What is hypernephroma?
Hypernephroma is an adenocarcinoma arising from urinife-
rous tubules of renal cortex. It is the most common (75%)
neoplasm of kidney in adults.
3. Pathological features of hypernephroma
The tumor is usually spherical, commonly occupying upper
pole of kidney. On section, it is characteristically yellow with
haemorrhagic areas and divided into numerous lobules by
fibrous septa. Microscopically, the cells are polyhedral or
cubical with abundant cytoplasm containing lipid and choles-
terol. The stroma is scanty, but rich in large blood vessels.
4. Spread of hypernephroma
• Growth into renal veins with systemic metastasis into lungs
and bones.
• Lymphatic metastasis to nodes at hilum of kidney.
5. Clinical presentation of hypernephroma
• Predominantly in males (M:F 2:1)
• Palpable renal swelling
• Painless haematuria with or without clot colic
• Symptoms of bone metastasis like pathological fracture,
bone pain, haemoptysis
• Persistent pyrexia presumably due to absorption of blood
and necrotic tissue from the tumor
• Unexplained anaemia out of proportion to haematuria
• Polycythemia due to increased erythropoietin secretion
196 SHORT NOTES AND SHORT CASES IN SURGERY

• Sudden left sided varicocele


• Nephrotic syndrome
• Hyper reninemia (hypertension) and hypercalcemia.
6. Differential diagnosis of a palpable kidney with haema-
turia in adult
1. Hypernephroma
2. Haemorrhage into hydronepohrosis
3. Congenital cystic kidney with haematuria
4. Solitary cyst kidney
5. Aneurysm of renal artery
7. Diagnosis of hypernephroma
• Intravenous pyelography (elongation and splaying of
calices)
• Ultrasound differentiates solid from cystic lesion
• CT scan shows the tumor and hilar adenopathy if any, and
vascularity.
8. Treatment of hypernephroma
1. Nephrectomy with removal of perinephric fat achieves a
cure rate of 60-70% if there is no metastasis.
2. Metastatic lesions may improve with alpha interferon or
interleukin II.
Hypernephroma is radioresistant. Vinblastine and progeste-
rone may be tried.
9. What is Wilm’s tumor?
Wilm’s tumor or nephroblastoma is a mixed tumor of children
below 5 years of age, containing epithelial and connective
tissue elements arising from embryonic nephrogenic tissue.
10. Clinical presentation of Wilm’s tumor
1. Abdominal mass with deteriorating general health
2. Unexplained fever
3. Haematuria only when tumor bursts into renal pelvis
(ominous sign).
11. Clinical differentiation of Wilm’s tumor from adrenal
neuroblastoma
• Wilm’s tumor does not cross the midline unlike adrenal
neuroblastoma.
• Calcification is common in neuroblastoma
CASE – 23 197

• Neuroblastoma metastasizes early to lungs, bones, liver,


eyes and brain, but bone; metastasis are rare in Wilm’s
tumor
• Elevated blood dopamine and urinary HVA in neuro-
blastoma.
12. Sites and origin of neuroblastoma
1. Arises from
• Adrenal medulla
• Sympathetic chain
2. Sites may be abdominal, in the mediastinum, neck and
pelvis.
13. Treatment of Wilm’s tumor
1. Immediate nephrectomy followed by post-operative radio-
therapy.
2. Inbilateral and inoperable cases actinomycin D and vincrys-
tine are of value.
Prognosis is best, if child is below 1 year (80% survival); over
this age survival is 30%. Recurrences usually occur within a
year.
14. Prognosis of hypernephroma
Sixty per cent 5 year survival rate, if tumor is only confined
to kidney.
15. Surgical causes of haematuria
• Hypernephroma
• Papilloma of bladder
• Enlarged median lobe of prostate
• Tumors of renal pelvis
• Renal trauma
Case 24

A young lady has been brought to casualty, after sustaining exten-


sive burns from kerosene flames gutting her thatched apartment.
1. What problems do you anticipate in this case?
1. Burn shock
2. Upper airway obstruction from edema
3. Carbon monoxide poisoning
4. Smoke inhalation injury syndrome.
2. Causes of burn injury
Burns are caused by exposure to excessive heat (e.g. flame,
hot liquids, hot surfaces, friction and electricity). Heat causes
tissue necrosis by coagulation of proteins. Other causes
include actinic rays, irradiation and chemicals. Skin cancer
can follow exposure to ultraviolet rays and irradiation.
Chemicals cause injury not by burning, but by chemical
reactions like oxidation, reduction, corrosion, salt formation,
vesicant activity and desiccation.
3. What causes burn shock?
1. Loss of circulating blood volume by water, electrolyte and
plasma protein loss in burn blisters, tissue edema and
evaporation.
2. Sudden intense pain of burn wounds.
4. Factors that determine severity of burn injury
1. Extent of body surface burnt; patients’ own palm area is
equivalent to 1%
2. Depth of burn
3. Age of patient (children and aged tolerate burn poorly)
4. Associated injuries and illness.
5. Clinical differentiation between partial thickness and full
thickness burn
Partial thickness burn usually appears moist red, and
blistered, while full thickness burn is white or brown, dry
and firm. Full thickness burns are anaesthetic.
CASE – 24 199

6. What is ‘rule of nine’s’ and its limitations?


Rule of nines is employed for estimating the body surface
area in adults. In it the hands are 9% each, legs 18% each,
front and back of chest and abdomen 18% each, head and
neck 9% and perineum 1%. In a child of 1 year head and neck
constitute 18% and at age of 5 they constitute 13%.
7. Guidelines for fluid replacement in a burn patient
Children with more than 10% burn and adults with more
than 15% burn should be given fluids. In adults, during first
24 hours, fluid requirement is 1-1.5 litres per each 10% burn.
Half of this is given in first 8 hours and rest in next 16 hours.
Second day requirement is half of first day. In children fluid
requirement during first 24 hours, is 40 ml/kg.
Simpler formulas include 4 ml/kg wt./% of burnt BSA with
half of administered during first 8 hours and rest during next
16 hours. The fluid replacement should be adequate to cause
urine output of 50 ml/hour in adult and 15 ml/hour in baby.
8. Early complications of burn
1. Acute renal failure
2. Gastroduodenal erosion and ulceration.
9. Treatment of burn wounds
After patient’s condition is stabilised, loose devitalized debris
is gently trimmed away and silver sulfadiazine is applied to
the wound. The wounds are dressed daily or alternate day to
absorb drainage and conserve heat. Once wound is dry, it is
left open. When a full thickness burn encircles the trunk or
limbs, escharotomies are required. All wounds, that will heal
spontaneously within 1 month are allowed to do so. When
the decision is that the wound will not heal within this time
frame (full thickness burns), burn eschar is removed and skin
grafting undertaken.
10. Mechanism of healing burn wounds
Partial thickness burns heal by regeneration from residual
epithelial tissue of sebaceous glands and hair follicles. Deep
or full thickness burns only heal from margin and are liable
for contracture. Burns that heal within 3 weeks have an
excellent result with good pliability of the skin, usually
normal pigmentation, and no hypertrophic scarring (scars
that are red, raised, and indurated). The longer the burn
200 SHORT NOTES AND SHORT CASES IN SURGERY

wound takes to heal beyond 3 weeks, the greater the inci-


dence of hypertrophic scarring.
11. Choice of fluid replacement in burn
Ringer lactate for first 24 hours, then 5% normal serum
albumin. The dose of serum albumin or plasma is 0.3-0.5 ml/
kg/% burn BSA given between 24-32 hours postburn followed
by 5% DW as needed to maintain adequate urine output.
Blood is required in major burns to raise haematocrit and
maintain tissue oxygenation.
12. Coverings used for burn wounds
• Patient’s own skin
• Cultured skin
• Amnion
• Cadaver or donor skin provided HIV negative
• Potato skin
• Synthetic membranes.
12. What is meshing?
When there is large burn wound with limited available donor
skin, a method of skin expansion is used, called meshing. The
split skin, taken from donor area is fed through a device that
cuts a series of parallel slits. The skin split can now be pulled
like lace to cover large area. Meshing increases the area of
skin grafts and allows blood and exudate to escape, thus
minimising haematoma. The grafts should be dressed with
tulle, covered by gauze, cotton wool and elasticated bandages.
13. Nutritional support to burn patients
Patients with burns exceeding 30% require a high protein and
high calori diet.
For adults—Daily protein requirement is 1 g/per kg body
weight + 3 gm% of BSA burnt. Daily calori need is 20 cal. per
kg wt. + 70 cal. × % of BSA burnt.
14. Why burn patients are more susceptible for infection?
1. Depressed immunity after burn
2. Dead organic tissue is an excellent medium for growth of
bacteria
3. Once implanted or colonized, the burnt skin is incapable of
preventing systemic dissemination.
CASE – 24 201

15. Late complications of burn


• Chronic scarring, contracture, poor temperature control
• Chronic renal failure
• Protein loosing enteropathy
• Immune deficiency.
16. Characteristics of major burns requiring treatment in a burn
center
1. Partial thickness burn > 25% BSA in adult and > 20% BSA
in children
2. Full thickness burn in excess of 10% in any age group
3. Deep burns involving face, hands, perineum
4. Burns complicated by inhalation injury
5. Electrical and chemical burns
6. Burns in highr risk groups (aged, infant, with intercurrent
disease).
17. How to treat inhalation injury in a burn patient?
1. Upper airway obstruction due to edema needs intubation
2. Carbon monoxide poisoning needs 100% O2 to prevent
hypoxic encephalopathy and renal failure. O2 inhalation
to continue till carboxy-haemoglobin concentration falls to
< 5%.
3. Smoke inhalation injury causes pulmonary insufficiency
and superadded infection, often ending in bronchiolitis
obliterans. Steroids and antibiotics have no role. Gentle
suctioning by bronchoscope, humidified O2 and good
physical measures are helpful.
18. Difference between high voltage and low voltage electrical
burns
High voltage electricity causes extensive tissue necrosis,
whereas, low voltage carries the risk of death from ventricular
fibrilation.
19. What is a burn team?
It consists of plastic surgeons, nurses, dieticians, physical and
occupational therapists and counsellers.
20. Clinical signs of burn wound infection
1. Conversion of second degree burn to full thickness necrosis
2. Dark brown or black discolouration of wound
3. Neoeschar formation
202 SHORT NOTES AND SHORT CASES IN SURGERY

4. Unexpectedly rapid eschar separation


5. Haemorrhagic discolouration of subeschar fat
6. Erythematous and edematous wound margin
7. Crusted serrations of wound margin
8. Vesicular lesions in healing or healed second degree burns
9. Metastatic lesions in unburnt tissue.
21. Histologic criteria for burn wound infection
• Dense microbial growth in eschar or subeschar-space
• Haemorrhage in unburnt tissue
• Micro-organisms in unburnt tissue
• Inflammatory reaction in nearby healthy tissue
• Perineural or intralymphatic presence of organisms
• Vasculitis with perivascular cuffing of organisms
• Small vessel thrombosis
• Intracellular viral inclusions.
Case 25

A 25-year-old has been admitted to trauma center after vehicle


accident. He is deeply comatose with retention of urine. His right
pupil is dilated and sluggishly reacting to light.
1. What do you think of this case?
Severe head injury with right oculomotor palsy.
2. Possible cause of right third nerve palsy
Right epidural, subdural bleed/contusion, intracerebral
haematoma or brain edema causing uncal herniation and
stretching/paralysis of third nerve.
3. Mechanism of brain injury
1. Direct impact causing displacement and distortion of
cerebral tissue with loss of neuronal function (unconscious-
ness) associated with cerebral contusion and laceration.
2. Secondary changes due to intracranial bleed or edema
gradually supervene leading to persistence of unconscious-
ness, and focal neurological deficit.
4. Factors determining the severity of head injury
1. Degree of impact which creates shearing forces to cause
diffuse damage to neuronal tissue and blood vessels.
2. Degree of mobility of brain in relation to skull and membranes,
e.g. mobility of brain is more in the aged due to its shrinkage.
3. Configuration of interior of skull, i.e. skull, falx and tentorium.
Where the skull is smooth (convexity), brain damage is less
severe than where it is rough, sharp or the cavity relatively
confined. Thus the temporal lobe will be damaged by sharp
splenoid ridge and confined middle cranial fossa; frontal
pole by rough anterior cranial fossa; the occipital pole by
meeting of falx, tentorium and occipital convexity; the
corpus callosum by sharp edge of falx and cerebral
peduncles by sharp edge of the tentorium.
4. Deceleration and acceleration as in traffic accidents, where a
moving head strikes road or stationary head is struck by
moving vehicle.
204 SHORT NOTES AND SHORT CASES IN SURGERY

5. What do you understand by concussion, contusion and


laceration?
In cerebral concussion there is brief physiological disruption
of function without organic structural damage. Hence the loss
of consciousness is only transient.
In cerebral contusion there is bruising of cerebral parenchyma
with bleeding, anatomical disruption of neurons and blood
vessels along with brain edema.
In cerebral laceration in addition to anatomical changes of
contusion, there is effusion of blood into CSF as the brain
surface is turned.
6. Types of cerebral haematomas and their presentation
Cerebral haematoma following injury can be extradural,
subdural and intracerebral.
Extradural haematoma is due to torn branches of middle
meningeal artery, commonly associated with fracture of
temporal or parietal bone. It is uncommon in aged because
the dura is more adherent to skull and brain is elastic in them.
After injury, patient regains consciousness and remains alert
for sometime. This is the lucid interval, during which,
haematoma enlarges. Once, it reaches sufficient size, there is
rise in intracranial pressure with uncal herniation along
tentorial orifice to cause midbrain compression, coma and
ipsilateral third nerve palsy. If pressure is not relieved,
displacement of brain stem at tentorial opening forces the
opposite crus against the sin of tentorium producing
ipsilateral hemiparesis.
Extradural haematoma can also be due to tear of middle
meningeal vein or superior longitudinal sinus with massive
bleed causing rapid deterioration of consciousness with
unilateral or bilateral leg weakness. Subdural haematoma is
formed by tear of subdural vein or by laceration of cortex
and subsequent arterial or venous haemorrhage. The
presentation is similar to extradural haematoma with the
following exceptions (1) lucid intervals is unusual (2) haema-
toma is extensive (3) deterioration is rapid (4) haematoma
may be ‘coup’ or contrecoup.
Intracerebral haematoma: Traumatic intracerebral haematomas
are not due to rupture of major intracranial vessel, but due to
CASE – 25 205

cerebral laceration, edema and necrosis which combinely form


a compressive lesion. Patient is in deep coma from beginning.
When there is a distinct lucid interval, then bleed has occurred
from a pre-existing aneurysm or a-v malformation.
7. Your approach to a patient of head injury in casualty
1. Protection of airway and if need be endotracheal intubation
2. Neurological assessment, i.e. level of consciousness, pupils,
limb paralysis, cranial injury
3. Associated injuries, i.e. injury to cervical spine, thorax and
abdomen
4. CT scan and skull radiograph depending upon indications.
8. Informations made available by skull-ray
• Presence of skull fracture, which may foretell presence of
extradural haematoma
• Depressed fracture needs surgery
• Presence of otorrhoea, bleeding from pharynx and nose can
be correlated to fracture of base of skull
• Shifting of calcified pineal gland may indicate supraten-
torial mass lesion
• Presence of intracranial air.
9. Which patients qualify for admission?
• Depression in level of consciousness
• Skull fracture
• Focal neurological signs
• Persistent headache/vomiting
• Alcohol intoxication
• Circumstances of injury unknown
• Crime related injury
• Absence of responsible relative/friend.
10. Measures to control brain edema
• Osmotic diuretics, i.e. mannitol 1 gm/kg
• Oral glycerol
• Dexamethasone
• Frusemide
• Controlled hyper ventilation to achieve pCO2 of 25 mm Hg.
206 SHORT NOTES AND SHORT CASES IN SURGERY

11. Features of basal skull fracture


• CSF rhinorrhea, otorrhea
• Air in subarachnoid space and ventricular system; bruising
around orbit (Racoon sign), bruising over mastoid process
(Battle’s sign)
• Signs of meningitis 48-72 hours after injury (neck stiffness
immediately after injury may be due to subarachnoid bleed)
• Pituitary failure when basal fracture crosses across pituitary
fossa
• Cranial nerve palsy (I, II, III, IV, V, VI, VII, VIII)
12. Symptoms and signs of raised intracranial pressure
Symptoms Signs
Headache Slow pulse
Vomiting Rising BP
Papilloedema
Constricted visual field Impaired consciousness
13. What is the meaning of a large reactive pupil?
This means that third nerve is intact. The opposite side pupil
is rather small from Horner’s syndrome (miosis, ptosis and
anhydrosis), probably traumatic carotid dissection.
14. Meaning of bilateral pinpoint pupils
High brainstem compression (pontine haemorrhage), but
toxicology screen is also required (narcotic overdose).
15. Which group of patients need intracranial pressure moni-
toring?
In unconscious patients, with raised ICP when neurologic
examination becomes insensitive to the changes in intracranial
pressure.
16. Role of barbiturate coma
When mannitol, hyperventilation and ventriculostomy fail
to control rise in ICP, barbiturate coma may be helpful by
reducing brain metabolism and blood flow. Barbiturate
induced hypotension can be managed with dopamine and
colloids.
17. Late complications of head injury
• Chronic subdural haematoma
• Normal pressure hydrocephalus
CASE – 25 207

• Post-traumatic epilepsy
• Post-traumatic headache.
18. Features of chronic subdural haematoma
• Mental changes like slowness, drowsiness, confusion,
memory disturbances, personality changes
• Sudden progressive dementia
• Focal neurological deficit, i.e. hemiparesis or hemisensory
disturbances.
19. What is ‘Contre coup’ injury?
‘Contre coup’ implies cranial injury diagonally opposite to
the point of trauma. These injuries may cause subdural bleed
and cerebral contusion and laceration.
20. What is Glasgow coma scale?
It is a scale to monitor patients of head injury. The three
aspects of coma, which are specifically assessed are eye
opening, best verbal response and best motor response.
Eye opening spontaneous—(4), to voice (3), to pain (2), none
(1).
Verbal response—oriented (5), confused (4), inappropriate
words (3), incomprehensible sounds (2), none (1).
Motor response—Obeys command (6), localises pain (5),
withdraws to pain (4), flexion to pain (3), extension to pain
(2), none (1).
A fully conscious person has E4 M5 V6, while a patient of deep
coma has E1, M1, V1.
21. What is revised trauma score?
It measures level of consciousness with trauma scores relating
to cardiovascular and respiratory system. High score like
Glasgow coma score relates to good prognosis.
22. What is trimodal pattern of trauma deaths?
Deaths due to trauma fall into three groups, i.e. fifty per cent
immediate death (brain laceration, major vessel rupture);
thirty per cent early deaths (epidural-subdural bleed, chest
trauma, abdominal/pelvic trauma with blood loss); twenty
per cent late deaths due to sepsis and multiple organ failure.
208 SHORT NOTES AND SHORT CASES IN SURGERY

23. What is Triage?


Triage means sorting, i.e. sorting out casualties for priority
in attention. In hospital triage may be used in 3 situations (1)
in casuality/trauma center (2) in X-ray department (3) in
determining priorities for intervention.
24. What other injury may be associated with craniofacial
trauma?
Cervical spine fracture. Forces associated with significance
head injury may be transmitted through cervical spine. All
victims with evidence of craniofacial trauma should be
initially treated as to have cervical spine injury. Prior to any
movement cervical immobilization with a rigid device be
undertaken.
25. Value of rectal examination in injuries of spine
A patulous anus is a good indication of spinal cord or cauda
equina injury.
26. Value of odontoid view (open mouth view)
Fracture of C1 and C2 vertebra are best seen in this view. In
normal person lateral masses of C1 overhang by 1-2 mm.
Values greater than this indicates fracture of C1. Fracture of
odontoid is apparent. In Type I fracture there is avulsion of
tip of odontoid; in Type II fracture is at base of odontoid and
in Type III odontoid fracture extends into vertebral body.
27. Difference between Jefferson’s fracture and hangman’s
fracture
Jefferson’s fracture occurs in posterior arch of C1 bilaterally,
usually without neurologic deficit. A hangman’s fracture
results from the avulsion of posterior arch of C 2 with
dislocation of C2 and C3 as seen in vehicle accidents and
judicial hanging. Neurological deficit is severe.
28. Role of steroids in spinal trauma
Methylprednisolone 30 mg/kg/24 hours may improve
neurological outcome in patients of spinal cord trauma.
29. Chances of recovery from complete traumatic transverse
myelopathy
Chances are remote, but MRI should be done because epidural
haematoma and soft disc herniation suggest a better
CASE – 25 209

prognosis. This is especially below T12, where lumbar roots


may reinnervate with time.
30. What takes priority in head injury—hypotension, hypoxia
or head injury itself?
Air way and breathing are to be attended first followed by
hypotension. Injury itself be attended last.
31. Types of deficits seen in spinal trauma
• Complete transection of cord
• Anterior cord syndrome
• Central cord syndrome
• Brown Sequard syndrome.
Complete transection Complete sensory motor palsy below the
lesion, with spinal shock resulting from disruption of
sympathetic tone and loss of vasomotor reactivity.
Anterior cord syndrome Loss of motor function, pain and
temperature with preservation of proprioception.
Central cord syndrome Severe motor dysfunction of upper
limbs than lower extremities, minimal sensory dysfunction;
patients with spinal canal stenosis (< 11 mm) and those with
osteoarthritis of spine are more liable.
Brown-Sequard syndrome (hemisection) Motor and propio-
ceptory dysfunction ipsilateral to lesion with abolition of pain
and temperature contralateral to lesion.
Case 26

A 60-year-old male is admitted with a slowly growing lesion of


penis, which on examination reveals to be a fungating ulcer with
rolled out margin and indurated base. The inguinal lymphnodes
are enlarged, hard and fixed.
1. What is your diagnosis?
Squamous cell carcinoma of penis with lymphnode meta-
stasis.
2. Common sites of squamous cell cancer
Lip, tongue, anus and penis, backs of hand and forearm,
pinna, esophagus.
3. Evolution of squamous cell cancer of skin
The first clinical evidence of malignancy is induration. The
area may be plaque like, verrucous or ulcerated and firm.
Later on the margin becomes everted and rolled out.
Squamous cell carcinoma does not usually arise from healthy
looking skin. Elastotic degeneration of dermis, keratosis,
irregular pigmentation, telangiectasia, leukoplakia and
fissuring predispose to squamous cell carcinoma.
4. What is basal cell carcinoma?
It is a malignant tumor arising from basal layer of epidermis.
Ninety per cent of lesions are found on face, usually around
inner canthus of eye and commonly in the male. On the lower
leg, the incidence in women is three times greater than men.
The lesions may be nodular, cystic and ulcerated. The early
tumors are small, translucent or pearly and raised with
rounded edges. Ulcerated lesion has raised rolled out edge.
The morphoeic type presents as a firm, raised red plaque.
The superficial type found on the trunk presents as a red scaly
patch and raised edge. It infiltrates deeper tissues like muscles
and bones (rodent ulcer). Hence, its diversity of appearance
compares it with syphilis as a great imitator.
CASE – 26 211

5. What is cylindroma?
Also known as turban tumor, it is considered as a variant of
basal cell carcinoma but some consider it as endothelioma.
The tumor gradually forms an extensive turban like swelling
over the scalp. It is a relatively benign tumor and ulceration
is uncommon.
6. How will you proceed to examine a swelling?
1. Inspection Number, extent, shape and size, colour, surface,
edge, pulsation, impulse on coughing, condition of skin
over the swelling, any visible pressure effect.
2. Palpation Rise in local temperature, tenderness, consistency,
(cystic, firm, hard), crepitation, fluctuation, translucency,
reducibility, fixity to skin and deeper structures.
3. Percussion (to differentiate air containing swelling from
cystic and solid swellings)
4. Auscultation (machinery murmur of aneurysmal varix).
7. What other organs, one is supposed to examine while
dealing with a malignant swelling?
• Draining lymphnodes for enlargement, consistency and
fixity.
• Respiratory system for consolidation, effusion.
• Liver for enlargement, consistency, nodule.
• Bony tenderness and pathological fracture.
8. Types of swellings
• Congenital (dermoid, teratoma, cysts)
• Traumatic (haematoma, aneurysm)
• Inflammatory (erysepalous, cellulitis, abscess, carbuncle,
elephantiasis)
• Neoplastic (fibroma, lipoma, haemangioma, lymphan-
gioma, melanoma)
• Miscellaneous.
9. Common premalignant lesions of skin
• Bowen’s disease
• Solar keratosis
• Radio dermatitis
• Chronic scars (Marjolin’s ulcer)
• Leukoplakia
• Paget’s disease of nipple.
212 SHORT NOTES AND SHORT CASES IN SURGERY

10. What is Bowen’s disease?


Bowen’s disease is an intraepidermal squamous cell
carcinoma and appears as a persistent, progressive, usually
non-elevated red, scaly or crusted plaque. Fifty per cent of
these patients may develop squamous carcinoma 6-7 years
later. Twenty-five per cent of them have coincident systemic
cancer involving GI or respiratory tract. Any psoriatic like
lesion not improving with steroid should be biopsied to
exclude Bowen’s disease. Podophyllin or 5 fluorouracil local
application; cryosurgery, cauterisation, and radiotherapy are
all effective.
11. What is Marjolin’s ulcer?
It is carcinoma developing on scars. The ulcer is painless,
grows slowly without lymphatic spread in early stages (scar
is avascular, without nerves and lymphatics).
12. What are haemangiomas?
Haemangioma is a developmental malformation of blood
vessel rather than a true tumor. It commonly occurs in skin
and subcutaneous tissues. A haemangioma can be capillary,
venous (cavernous) or arterial. Capillary haemangioma can
appear as a salmon patch, portwine stain (both commonly on
face) or strawberry angioma. Salmon patch and strawberry
angioma have normal involution, while portwine stain needs
laser treatment or excision and skin grafting.
Cavernous haemangioma gives rise to soft spongy masses, which
can be partially emptied by pressure, refilling on release of
pressure. They commonly occur in the lips, cheeks, face and
brain. Plexiform haemangioma consists of dilated arterio-
venous anastomosis that feel like a bag of pulsating earth
worms. Treatment is by injection of sclerosing agents, laser
application or embolisation.
13. Characteristics of lymphangioma
Lymphangioma also involves lips and tongue. The lymphatic
vessels often are enormously dilated to form multilocular cysts
called cystic hygroma, commonly seen in neck, pectoral region
and axilla.
14. What are the characteristics of malignant melanoma?
Malignant melanoma is the malignant neoplasm arising from
epidermal melanocytes. They can be (1) superficial spreading
CASE – 26 213

melanoma with an irregular edge and variegated pattern


(65%) (2) melanotic freckle (3) nodular melanoma (4) acral
lentiginous melanoma, (5) amelanotic melanoma. A mole that
increases in size, itches or bleeds should be biopsied to exclude
malignancy. All moles excepting congenital hairy, papillary,
large and darkly pigmented ones are virtually always benign
prior to puberty. Nodular melanoma are most malignant.
Treatment of small localised lesion is by excision, laser or
cryoprobe. When lymphnodes are involved, their dissection
is warranted. When metastasis is beyond the lymphnodes,
vindesine, interferon, interleukin II may be tried. Isolated limb
perfusion with melphalan can be employed for recurrent lesion
in a limb or prophylactically for poor prognosis extremity
tumors.
15. What are noncutaneous malignant melanoma?
Melanoma arising in conjuctiva, mouth, tongue and anus fall
into this group so also that of choroid. As choroid has no
lymphatics, it spreads by blood stream especially to liver.
Enucleation of eye is the answer. At other sites complete cure
is rare.
16. What is Kaposi sarcoma of skin?
It is a neoplasm of multifocal origin, derived from prolife-
rating capillary vessels and perivascular connective tissue
cells. The tumor has been associated with lymphoma,
haemolytic anaemia, diabetes mellitus and AIDS. The lesions
are multiple dark blue or purple papules associated with
systemic symptoms. Visceral lesions and mucosal lesions also
occur. Treatment is with intralesional vinblastine or systemic
alfa interferon. Cryotherapy can also be useful. Visceral
lesions need radiotherapy or chemotherapy. AIDS associated
Kaposi sarcoma occurs in young people and is virulent but is
fortunately declining.
17. What are the primary cutaneous malignant lymphomas?
These rare tumors fall into three groups:
1. Mycosis fungoides and Sezary syndrome
2. Primary cutaneous non-Hodgkin’s lymphoma
3. Primary cutaneous Hodgkin’s lymphoma.
Mycosis fungoides is characterised by initial infiltration of
the skin by malignant lymphocytes, with subsequent
214 SHORT NOTES AND SHORT CASES IN SURGERY

involvement of bone marrow, liver, spleen and lymph nodes.


The skin lesion is a papular rash, which progresses to plaque
and then tumor formation. Treatment is by chemotherapy or
radiotherapy. In Sezary syndrome, a form of fatal reticulosis,
giant T-lymphocytes infiltrate, the skin and blood. There is
pruritus, lymphadenopathy, keratoderma and leonine facies.
18. What are ulcers and their types?
An ulcer is a discontinuity of an epithelial surface. Ulcers
can be nonspecific, specific (e.g. syphilitic, tuberculous) or
malignant. Non-specific ulcers are due to infection of wounds,
physical or chemical agents. Trophic ulcers are due to impai-
rment of nutrition, which depends upon adequate blood
supply and intact innervation.
19. Points to be noted during examination of an ulcer
Site, size, shape, edge, floor, base, discharge, pain, fixity and
condition of draining lymphnodes.
20. Difference between syphilitic and tuberculous ulcer
The ulcer edge is undermined and often bluish in tuberculous
ulcer, but vertically punched out in syphilitic ulcer. The floor
of tuberculous ulcer is like apple jelly granulation, while it is
like wash-leather slough in syphilitic ulcer.
21. Types of squamous carcinoma in penis
There are two types of squamous carcinoma of penis—the
flat or infiltrating and the papilliferous. The former may be
associated with leukoplakia; the latter commences in a long-
standing papilloma. In either case, the spread is to inguinal
and then iliac lymphnodes. As the lesion is usually infected,
palpable inguinal lymphnodes do not imply metastasis in
every patient.
22. Treatment of carcinoma penis
Small, well differentiated growth can be managed by
radiotherapy (high voltage X-ray, radium mould applicator/
implantation). Surgery is required for large anaplastic
growths, when there is infiltration of shaft of penis, when
therapy has failed, or in elderly men, who do not mind
amputation of penis. Partial amputation is made for distal
growths leaving at least 2 cm of uninvolved shaft. With an
advanced, infiltrating or anaplastic lesion total amputation
CASE – 26 215

has to be performed. If the enlarged nodes do not regress with


antibiotics, a block dissection has to be made 3 weeks after
amputation. If the enlarged lymphnodes are massive and
fixed, X-ray therapy may cause temporary regression.
Radiotherapy and surgery carry a 5 year survival of 60-70%,
but if inguinal nodes are involved, it drops to 35%.
23. What is Fournier’s gangrene?
Otherwise known as idiopathic gangrene of scrotum, it is a
vascular disaster of infective origin. The three cardinal
features are (1) sudden appearance of scrotal inflammation
(2) rapid onset of gangrene (3) no accountable cause in 50%.
Haemolytic streptococci, probably cause a fulminating
inflammation, that results in obliterative arteritis and
gangrene of superficial tissue. Wide excision of the sloughing
areas and suitable antibiotics can save the patient.
24. Predisposing factors for carcinoma penis
• Phimosis in over 50% cases
• Balanoposthitis
• Papilloma and Buschka-Lowenstein tumor
• Leukoplakia
• Paget’s disease of penis
• Erythroplasia of Queyrat
• Bowen’s disease
• Balanitis xerotica.
25. What is condyloma accuminata?
This is sexually transmitted papilloma virus disease, mostly
appearing near coronal sulcus as sharp pointed, red, papillary
excrescences, sessile or pedunculated with offensive odor.
The lesions are premalignant and be treated with local 25%
podophyllin/0.1% bleomycin or fulgration.
Buschka-Lowenstein tumor is a giant variety of condyloma
accuminata which is often confused with penile cancer.
26. Non-melanomatous pigmented skin lesions
• Seborrheic keratosis
• Pigmented basal cell carcinoma
• Glomus tumor
• Kaposi sarcoma
• Histiocytoma
216 SHORT NOTES AND SHORT CASES IN SURGERY

• Keratoacanthoma
• Acanthosis nigricans
• Cafe-au-lait spots.
27. Histological classification of melanoma
Stage I—thickness < 0.75 mm
Stage II—thickness 0.76-1.5 mm
Stage III—thickness 1.51-3 mm
Stage IV—thickness > 3 mm.
While there is no risk of metastasis in stage I, it is 25% in
stage II and around 60% in stages III and IV.
28. What is keloid and its treatment?
Keloid is a clinical condition characterized by proliferation
of immature fibroblasts, collagen fibrils and immature blood
vessels (mature fibroblastic proliferation without blood
vessels is hypertrophied scar). There may be itching, oozing
and blanching. It has a familial diathesis. Treatment includes
intralesional hyaluronidase, vitamin A and hydrocortisone,
deep X-ray therapy, ultrasonic therapy and parenteral
vitamin A. Excision of keloid followed by skin grafting and
post- operative radiotherapy to both donor and recipient sites
are also rewarding, when above conservative methods fail.
29. Complications of neurofibroma
1. By position, i.e.
a. Acoustic neurofibroma with 7th and 8th cranial nerve
palsy.
b. Root pain and paralysis by dumb-bell neurofibroma
arising from dorsal nerve roots
c. Bizarre intestinal symptoms by abdominal neurofibroma
2. Sarcomatous changes
3. Cystic degeneration
4. Cosmetic problems by plexiform lesions.
30. Clinical evidence of malignancy in neurofibroma
• Appearance of pain in a painless lesion
• Sudden increase in size
• Paralysis or anaesthesia in the distribution of nerve of origin
• Evidence of increased vascularity
• Fixity to surrounding structures.
31. What is hamartoma?
Hamartoma is a tumor like developmental malformation,
consisting of overgrowth of tissue. Common hamartoma
CASE – 26 217

lesions include benign pigmented moles, majority of angiomas


and neurofibromas. Malignant transformation in hamartoma
is rare.
32. What is glomangioma?
Otherwise known as glomus tumor, it arises from glomus
body. Glomus body is a special arteriovenous anastomosis
surrounded by smooth muscle cells and large pale cuboidal
cells known as glomus cells, encompassed by a net work of
myelinated and unmyelinated nerve fibres. Glomus body is
present in nail bed, tips of fingers and toes and the palmar
surface of phallanges.
Glomus tumor is a benign, slowly growing tumor, blue or
reddish in colour, causing severe pain due to pressure on
nerve endings by dilated vessels. Surgical excision provides
complete relief.
Case 27

A 20-year-old male has presented with a slowly growing painless


cystic swelling below the angle of jaw. The swelling is ovoid in
shape with distinct edges and is freely mobile.
1. What is the likely diagnosis?
A branchial cyst.
2. Origin of a branchial cyst
Persistence of cervical sinus during development is the likely
explanation. The secretion usually comes from the appen-
dages of ectodermal lining, sweat and sebaceous glands. The
cyst hence, lies superficial to the structures derived from 2nd
and 3rd branchial arches, i.e. lesser cornu of hyoid, posterior
belly of digastric, facial nerve and external carotid artery.
Some consider it to develop from inclusion of parotid epithe-
lium.
The cyst contains cheesy material secreted by sebaceous
glands with high cholesterol content.
3. Differential diagnosis of swellings below angle of jaw
• Cold abscess
• Cervical dermoid
• Plunging ranula
• Cystic hygroma
• Carotid body tumor
• Submandibular gland tumor
• Solitary enlarged cervical lymphnode.
4. Characteristics of thyroglossal cyst
Thyroglossal cyst can occupy a position along the path of
thyroglossal duct, that extends from foramen caecum of the
tongue upto isthmus of thyroid gland. Hence, the cyst can
occur anywhere along the course of the duct, i.e. in the sub-
stance of tongue, in the floor of the mouth, suprahyoid posi-
tion, subhyoid region (most common), in front of thyroid or
cricoid cartilage.
Hence, the cyst is in midline of neck, usually elongated, moves
with deglutition and moves up on protrusion of tongue.
CASE – 27 219

Surface is smooth and side to side movement is free. Unless


inflected, it is painless and transillumination is negative.
Recurrent infection can lead to fistula formation.
5. Neck swellings that move on deglutition
• Thyroid swellings
• Thyroglossal cyst
• Enlarged subhyoid bursa
• Enlarged pretracheal glands
• Laryngocele.
6. Differential diagnosis of midline swellings in neck
Cystic Solid
Ranula Lymphnode
Cervical dermoid thyromegaly
Enlarged subhyoid bursa
Thyroglossal cyst
Cold abscess in space of Burns
7. Swellings in submandibular triangle
Cystic Solid
• Plunging ranula • Lymphnode
• Sublingual dermoid • Enlarged sub mandibular
• Salivary gland retention gland (tumor, sialitis)
cyst Vincent’s angina.
8. Swellings in carotid triangle
Cystic Solid
• Branchial cyst • Carotid body tumor
• Cold abscess • Lymph node
• Carotid aneurysm • Sternomastoid tumor
• Tumor of lateral lobe • Branchiogenic carcinoma of
thyroid
• Laryngocele • Solid swelling of lateral lobe of
thyroid
9. Swellings in posterior triangle
Cystic Solid
• Cystic hygroma • Lymphnode
• Solitary lymphatic cyst • Cervical rib
• Cold abscess
• Subclavian aneurysm
• Pharyngeal pouch.
220 SHORT NOTES AND SHORT CASES IN SURGERY

10. What is ranula?


Ranula is a mucous retention cyst, arising from mucous glands
in the floor of the mouth and undersurface of tongue. The
swelling is spherical with characteristic faint blue colour.
Transillumination and floctuation are positive. It should be
distinguished from sublingual dermoid, which is midline
swelling and transillumination is negative. Excision is curative.
11. What is dermoid?
Inclusion of surface ectoderm at the fusion line of body
produces dermoids. Sebaceous cysts are lined by squamous
cells and are better called ‘epidermoid’. Dermoids can be (1)
teratomatous dermoids (2) sequestration dermoids (3)
implantation dermoids. Dermoids though contain all the
three elements of embryonic tissue, epithelial differentiation
is predominant with often hair and teeth.
12. What is branchial fistula?
Branchial fistula can be congenital or acquired. Congenital
variety is most common occuring due to failure of fusion of
2nd branchial arch with 6th branchial arch. The fistulous tract
extends from lateral pharyngeal wall passing between
external and internal carotid to open up in the neck. In most
cases, only the external opening is visible, which lies at the
anterior border of sternomastoid at the junction of lower one-
third and upper two-third. Sometimes, only a dimple moving
up with deglutition marks the external opening. The entire
tract needs excision taking care not to damage carotid vessels
and hypoglossal nerve.
13. What is cystic hygroma?
Cystic hygroma is a variety of lymphangioma or hamartoma
containing dilated embryonic lymph channels filled with clear
colourless lymph. It is multilocular and multilobular. The
usual sites are root of neck in posterior triangle, axilla,
inguinal region, mediastinum, and buccal mucosa. The victim
is a young infant or child with a soft cystic swelling, which is
transillumination positive. Cystic hygroma can cause obs-
tructed labor or when present in mediastinum—mediastinal
compression. Treatment consists of injection of sclerosing
agents or complete removal.
Note Sebaceous cyst, lipoma, fibroma, neurofibroma can be
found anywhere in the neck.
CASE – 27 221

14. Cholesterol containing cysts


• Branchial cyst
• Cystic hygroma
• Dentigerous cyst
• Thyroglossal cyst.
15. What is pharyngeal pouch?
Pharyngeal pouch is a pulson diverticulum of pharynx
through the two parts of inferior constrictor, i.e. thyropharyn-
geus and cricopharyngeus. While thyropharyngeus is
supplied by accessory nerve, cricopharyngeus is supplied by
recurrent laryngeal; the former being propulsive in function
and the latter constrictive. Muscular incoordination between
two parts of inferior constrictor raises intrapharyngeal
pressure to produce the pulson diverticulum. When big
enough, it may appear as a swelling in neck causing dys-
phagia. Barium shallow is diagnostic and when symptomatic,
excision is the answer.
16. What is laryngocele?
Laryngocele is due to herniation of laryngeal mucosa, through
thyrohyoid membrane, commonly found in professional
trumpet players and blowers. The swelling moves up during
swallowing and is made prominent by valsalva.
17. What is sternomastoid tumor?
It is otherwise known as congenital torticollis produced by
trauma or ischaemia damaging sternomastoid muscle in
infants. There is a fusiform tender swelling in the belly of
sternomastoid at beginning, which gradually disappears to
produce the peculiar deformity. Sternomastoids then feels
firm and cord like. If not corrected, early scoliosis and
contracture of deep cervical fascia and scalene muscles may
occur. However, in every case squint and cervical hemi-
vertebra be excluded. Division of upper head or both lower
heads of sternomastoid are curative.
18. What do you know about cold abscesses, and their sites?
A cold abscess is collection of pus in tuberculous infection
without pain and signs of inflammation. Cold abscess in the
neck can form due to caseation of tuberculous lymph node or
due to caries of cervical spine.
222 SHORT NOTES AND SHORT CASES IN SURGERY

Tubercular infection of cervical lymphnode passes through


three phases; the First Stage of lymphadenitis with large tender
discrete lymphnodes; the Second Stage of perilymphadenitis,
when the glands are matted and third stage of liquefaction
(caseating necrosis) and cold abscess formation. The jugular
nodes in anterior triangle are affected most. In the beginning
the abscess remains deep to deep cervical fascia, but gradually
tracks superficially to form the collar stud abscess.
Cold abscess originating from cervical spine may track
anteriorly or posteriorly. Anteriorly it first lies deep to
prevertebral layer of deep cervical fascia where it may form
the midline retropharyngeal abscess or may track laterally
behind the carotid sheath to form a cold abscess in posterior
triangle of neck. When rupturing posteriorly, the pus may
enter spinal canal, where from, it may follow the cervical
spinal nerves, thus appearing in posterior triangle of neck.
Tenderness over spinous processes, limitation of neck
movement and cervical muscle spasm are clue to diagnosis.
Aspiration only yields caseous material, but no cholesterol
crystals.
Nodular leprosy with degeneration, gumma and actinomyco-
sis can form cold abscess but are rare.
A psoas abscess from caries of thoraco-lumbar vertebrae may
present in iliac fossa as a floctuating cold abscess. Tuberculous
abscess of mesentery presents as cystic retroperitoneal
swelling. Majority of cold abscesses of chest are secondary to
tuberculous intercostal lymphadenitis. The intercostal
lymphnodes are situated posteriorly, near the neck of the rib
or anteriorly in association with internal mammary vessels.
Cold abscess of chest is to be distinguished from lipoma and
empyema necessitatis, which is not only tender, but exhibits
impulse on coughing.
19. Common primary malignant tumors of neck
• Chemodectoma
• Lymphomas
20. What is chemodectoma?
Otherwise, known as carotid body tumor or potato tumor, it
is histologically a non-chromaffin paraganglionoma, arising
from carotid body situated at the carotid bifurcation. The
CASE – 27 223

tumor moves from side to side, but not vertically and trans-
mits carotid pulsation. Biopsy is dangerous because of
torrential bleed, but complete excision with carotid graft is
curative.
21. Distribution of cervical lymphnode involvement in
tuberculosis
There are 800 lymphnodes in body and nearly 300 are in the
neck. Infection is commonly by human type of tuberculous
bacillus. Distribution is—upper jugulare 54%, posterior
triangle 22%, submaxillary 12%, supraclavicular 4%. More
than one group may be involved.
22. What is Ludwig’s angina?
It is a brawny swelling of submandibular region, combined
with inflammatory edema of mouth, commonly due to
virulent streptococcal infection of cellular tissue around sub-
mandibular gland. It may also be a complication of advanced
carcinoma of mouth.
Vincent’s infection refers to necrotising ulcerative gingivitis,
caused by spirochetes and fusiform bacilli during periods of
stress.
Case 28

A 50-year-old male has been admitted with complaints of mild


constant pain in left iliac fossa, increasing constipation and
occasional bleeding per rectum.
1. Which disease is to be excluded in this patient?
Carcinoma colon
2. Macroscopic forms of cancer colon
• Annular
• Tubular
• Ulcerative
• Cauliflower (fungating).
3. Spread of colonic malignancy
• Local spread
• Lymphatic spread to epicolic, paracolic and finally to
lymphnodes around superior and inferior mesenteric
vessels
• Blood borne metastasis (late) to liver via portal circulation.
4. Clinical presentation of colonic cancer
Leftsided colon Right sided colon
• Increasing intestinal • Mass in right iliac fossa
obstruction
• Pain due to pericolitis or • Suspected appendicular
invasion lump without regression on
treatment
• Alternating constipation
and diarrhoea
• Subacute intestinal obstruction • Unexplained anaemia
• Colicky pain in carcinoma • Abdominal pain
sigmoid colon
• Tenesmus in papilliferous • Unexplained weight loss
growths
• Palpable lump
CASE – 28 225

5. Common characteristics of colonic cancer


• Males above 50 are common victims.
• Twenty-five per cent cases present as emergencies with
intestinal obstruction or peritonitis.
• Carcinoma of ascending colon is more common in women
even though overall prevalence is M:F = 3:2
• In about 75%, neoplasm is situated on left side of colon.
• Often it may present as intussusception.
6. Investigations required to confirm the diagnosis
1. Sigmoidoscopy in all aged patients complaining of blood
and mucus in stool
2. Colonoscopy often discovers carcinoma at multiple sites
3. Barium enema (double contrast) to show constriction or
constant irregular filling defect
4. Barium meal better delineates a tumor of cecum
5. Ultrasound and CT to exclude hepatic metastasis.
7. Treatment approach in colonic cancer
1. Right hemicolectomy for carcinoma cecum and ascending
colon.
2. Excision of transverse colon and two flexures along with
transverse mesocolon and greater omentum with end to
end anastomosis, when transverse colon is involved.
3. Excision from splenic flexure upto upper one-third of
rectum, when carcinoma is in pelvic colon.
4. When hepatic flexure is the site, then upper half of ascen-
ding colon, transverse colon including splenic flexure be
resected.
5. In involvement of splenic flexure—the transverse colon
along with both the flexures be resected.
8. Is one stage resection and anastomosis recommended?
Yes, if colon preparation is adequate and lesion is non-
obstructive.
9. How colon preparation is to be made prior to surgery?
• High calori low residue diet.
• Metronidazole plus gentamicin to sterilize the gut.
• Gut irrigation with isotonic saline 2-3 litres/hour for 3-4
hours.
226 SHORT NOTES AND SHORT CASES IN SURGERY

10. Criteria for non-operability and follow-up action


1. Fixity of tumor or fixed loco regional lymphnodes, involve-
ment of pelvic peritoneum imply inoperability.
2. Inoperable tumors are tackled by transverse colostomy for
left colon tumor; ileocolostomy for growths of ascending
colon; left iliac colostomy for pelvic colon tumors.
Ninety per cent of colonic cancers are resectable.
11. Management of hepatic metastasis
• Resection if single focus
• Cytotoxic drugs or hepatic artery ligation for multiple
painful metastasis.
12. Types of colostomy
1. Temporary colostomy to relieve a distal obstruction of
sigmoid colon either by carcinoma or diverticulitis,
operative treatment of high fistula is ano, protection of low
colorectal anastomosis after anterior resection, and in vesico
colic fistula.
2. Permanent colostomy after excision of rectum for carcinoma,
usually done in left iliac fossa, 6 cm above and medial to
anterior superior iliac spine. The lateral space between the
intraperitoneal segment of sigmoid colon and the
peritoneum of pelvic wall be carefully closed to prevent
internal herniation of loops of small bowel through the
deficiency. Of course, a retroperitoneal tunnel for the
colostomy avoids creating a lateral space.
13. Complications of colostomy
• Prolapse
• Retraction
• Necrosis of distal end
• Stenosis of the orifice
• Colostomy hernia
• Bleeding from granuloma around colostomy margin.
14. High risk factors for carcinoma colon
• Rectocolonic polyps (familiar polyposis, villous polyps,
adenomatous polyps, history of juvenile polyps)
• Family history of colon cancer
• Ulcerative colitis
• Crohn’s disease
• Immunodeficiency disease.
CASE – 28 227

15. Distribution of carcinoma large bowel


Cecum and ascending colon 16%
Transverse colon 5%
Descending colon 9%
Sigmoid colon 20%
Rectum 50%
16. Carcinoma potential of polyps
1. Adenomatous polyps greater than 1 cm carry a cancer risk
of 2-10%; bigger the size, higher is the risk of cancer.
2. Villous adenoma are usually sessile and become malignant
in 10% cases; if < 2 cm and in upto 50% cases, if larger.
3. In familial polyp syndromes with high chances of
malignancy, colectomy with ileostomy or ileoproctos-
tomy—after stripping of rectal mucosa is indicated.
17. What are hamartomatous polyps?
Polyps of Peutz-Jegher’s syndrome, juvenile polyps and
mucous polyps are benign with negligible malignancy poten-
tial. They are removed, when they produce pain, bleeding or
hypoproteinemia.
18. What is intestinal carcinoid tumor?
Malignant carcinoid tumor occurs throughout the GI tract,
but commonly in the appendix (65%) and ileum (25%). They
arise in the cells of kulchitsky at the base of crypts of
Lieberhühn with multiple metastasis to liver. The tumor
produces 5 HT, kinins, indoles and prostaglandins. Symp-
toms include flushing, diarrhoea, asthmatic attacks, and often
pulmonary/tricuspid stenosis. Treatment is surgical removal
of primary lesion and if possible the metastasis. Medical
treatment includes serotonin antagonists, alpha methyl dopa
and antihistaminics.
19. Effect of diet on colon cancer
• Low fiber diet predisposes to colon cancer
• High intake of polyunsaturated fat is also carcinogenic.
20. Clinical manifestation of diverticular disease of colon
In 90% cases, sigmoid colon is involved and 5% cases have
associated gallstones and hiatus hernia (Saint’s triad).
1. Diverticulosis is usually asymptomatic, but may cause
heaviness in lower abdomen and flatulence. Excessive
228 SHORT NOTES AND SHORT CASES IN SURGERY

colonic segmentation may cause severe colic pain in left


iliac fossa.
2. Diverticulitis presents with recurrent periodic inflamma-
tion and pain, perforation leading to peritonitis and
paracolic abscess; haemorrhage, intestinal obstruction due
to progressive fibrosis, or fistula formation.
21. Diagnosis and management of diverticulosis
1. Diagnosis is by barium enema and sigmoidoscopy, done
only when acute inflammation has subsided.
2. Treatment of acute diverticulitis with rest, antispasmodics,
broad spectrum antibiotic and metronidazole. Morphine
and enema are contraindicated for fear of perforation.
3. Diverticulosis be treated with high fiber diet or fiber
supplement.
4. Surgical treatment is required for recurrent attacks or
complications in the form of resection of involved segment
and end to end anastomosis.
22. Differentiation of diverticulitis from carcinomas
Diverticulitis Carcinoma
Tenderness Very often Rare
Mass Common May be
Bleeding Intermittent, profuse Persistent, small amount
X-ray Diffuse change Localized change
Sigmoidoscopy Surrounding No inflammation
inflammation
Case 29

A 20-year-old young male has been admitted with history of


flushing, headache and palpitation. Chest X-ray has revealed a
homogeneous round opacity close to upper thoracic vertebra and
blood pressure recording has been varying from 160/100 mm to
220/140 mm Hg.
1. What do you think of this patient?
Most likely a case of extra-adrenal pheochromocytoma.
2. What are pheochromocytoma and their incidence?
Pheochromocytomas arise from chromaffin cells, within adre-
nal medulla, ten per cent of these tumors are malignant, ten
per cent are bilateral, ten per cent are extra-adrenal and 10%
are multiple `rule of ten’. They can be found anywhere along
the paraganglionic system. The most common extra-adrenal
site is organ of Zuckerkandl at carotid bifurcation. Other sites
are urinary bladder, renal hilum, chest and neck. Extra-
adrenal tumors have higher malignancy potential (25-40%).
3. Laboratory diagnosis of pheochromocytoma
1. Urinary assay (24 hours)
• VMA ≥ 5.5 μ gm/mg of creatinine.
• Metanephrine ≥ 2.2 μ gm/mg of creatinine.
2. Raised serum epinephrine and norepinephrine; plasma
catecholamine > 1000 μ gm/ml
3. CT scan/MRI
4. Radionucleide imaging using 131I MIBG.
4. Differential diagnosis
• Carcinoid
• Diabetes with hypertension.
• Thyrotoxicosis
• Anxiety state.
5. Which type of patients need urine screening tests for
pheochromocytoma?
• Young hypertensives
• Labile hypertension
230 SHORT NOTES AND SHORT CASES IN SURGERY

• Unexplained angina
• Paroxysmal atrial tachycardia.
6. Complications of pheochromocytoma
• Severe hypertension leading to sudden blindness or
cerebrovacular accident, occurring after manipulation,
trauma or emotion.
• Cardiomyopathy
• Severe hypotension and shock after removal or due to
haemorrhage and necrosis of tumor.
7. Patient care before, during and after surgery?
• Control of hypertension prior to surgery by alpha-blocker
like, phenoxybenzamine and labetalol given over few
weeks.
• During surgery, sodium nitroprusside to control severe
hypertension unresponsive to alpha and beta blockers.
• The main adrenal vein is to be ligated at first step and
minimum tumor manipulation be made.
• After surgery, resistant hypotension unresponsive to
noradrenaline may occur and be managed with plasma
expanders.
8. Medical treatment of inoperable malignant pheochromo-
cytoma
• 131I MIBG
• Combination chemotherapy.
9. Does hypertension vanish after surgery?
In most cases, hypertension disappears or becomes easily
controllable with drugs. If it does not, multiple tumors are
likely.
10. Other causes of surgical hypertension
• Coarctation of aorta
• Cushing’s syndrome
• Conn’s syndrome (primary hyperaldosteronism)
• Renovascular hypertension.
11. When to suspect coarctation
• Diminished delayed femoral pulse
• Systolic BP lower in the legs than arms with equal diastolic
BP.
CASE – 29 231

• Rib notching in chest X-ray


• Systolic murmur in the back
• Heart failure in infancy
• Intracranial bleed.
12. When to suspect Conn’s syndrome?
Hypertension with hypokalemia and extreme weakness with
polyuria, and hypernatremia.
13. Which findings suggest Cushing’s syndrome?
Trunkal obesity, stria, muscle weakness and atrophy,
osteoporosis, moonface, hirsutism, amenorrhoea and
impotency (male), hyperglycemia, leukocytosis, lympho-
penia, hypokalemia, staunted growth in children.
14. Causes of renovascular hypertension
• Renal artery stenosis
• Fibromuscular dysplasia
• Renal artery atherosclerosis.
15. Investigations for renovascular hypertension
• Rapid sequence IVP
• Radiohippurate scan
• Renal vein renin estimation
• Aortogram.
16. Findings in rapid sequence IVP suggestive of renal artery
stenosis
• Delayed appearance of dye or prolonged persistence
• Irregular kidney contour due to segmental atrophy
• Kidney smaller by ≥ 1.5 cm.
17. How specific are IVP and renal scan for diagnosis of
renovascular hypertension?
No, test for screening of renovascular hypertension is ideal.
Renal vein renin value 1.5 times more than opposite side or
values significantly higher from both renal veins in compari-
son to inferior vena cava are suggestive. Post-drug test (after
captopril 50 mg or enalapril 2.5 mg) if shows further reduction
in renal function (GFR) in radionucleide scan then stenosis is
very likely.
18. What are chromogranin A and MIBG?
Chromogranin A is found in catecholamine storage vesicles
and its value is elevated in pheochromocytoma. MIBG stands
232 SHORT NOTES AND SHORT CASES IN SURGERY

for metaiodobenzyl guanadine. It is used for localisation of


pheochromocytoma as well as for therapeutic of inoperable
tumor.
19. Role of balloon angioplasty in renovascular hypertension
Effective in 60% patients particularly fibro muscular
dysplasia; most likely to fail in osteal lesions and atherosclero-
tic lesion. Dissection and thrombosis with loss of kidney can
occur.
20. Incidence of surgical hypertension
Five to six per cent.
21. Surgical outcome in pheochromocytoma and renovascular
hypertension
1. Pheochromocytoma—Operative mortality 3-5%; Five years
survival in benign disease 85%, 5 years survival in malig-
nant disease is O, even though hypertension is controlled.
2. Renovascular disease—Renal revascularisation in fibro-
muscular dysplasia—operative mortality 1%, cure rate 90%;
for atherosclerotic disease-operative mortality 2-4%; cure
80%.
22. When to suspect renovascular hypertension?
Young hypertensive with bruit in back or on the flanks.
Case 30

An young man, a victim of serious road accident has just been


received in casualty. He has been bleeding profusely and is gasping
for breath. His pulse is feeble and he is unconscious, cyanosed
without recordable BP.
1. What will be your approach to this patient?
• Maintenance of airway and oxygenation
• Combating haemorrhagic shock
• Attention to injuries and bleeders.
2. How to restore airway and breathing?
Most common cause of airway obstruction in unconscious
patient is the base of relaxed tongue impinging against
posterior pharyngeal wall. The head hence be turned to
oneside, hyper extended and the chin is lifted (head tilt, chin
lift and jaw thrust). Suction be made to clear the throat of
secretions. Thereafter an oral or nasopharyngeal airway is
put and Ambu bag ventilation with O2 started. If there is no
improvement, endotracheal intubation is to be done with
ventilation by Ambu bag or respirator (PEEP).
3. Disadvantage of ventilatory support without endotracheal
intubation
Positive pressure breathing (mouth to mouth, bag and mask)
can be associated with significant air delivery to stomach
leading to gastric dilatation that impairs diaphragmatic
excursion and hence limits ventilation. Aspiration of gastric
contents is also a problem. Normal intragastric pressure is 7
cm H2O. Regurgitation is likely with pressure exceeding 20
cm H2O and can be quickly relieved by nasogastric intubation
and aspiration.
4. How to know proper positioning of endotracheal tube?
1. Auscultation of lungs for equal air entry
2. Symmetric chest expansion with each breath
3. Absence of auscultated air sounds over epigastrium
4. Improvement in cyanosis.
234 SHORT NOTES AND SHORT CASES IN SURGERY

5. How will you attend to circulation?


1. External cardiac compression
2. MAST suit placement
3. Fluid/plasma, haemaccel through large bore IV line
4. Vasopressures
5. Raising of foot end of bed.
6. Complications of external chest compression
• Rib and sternal fracture (40-80%)
• Bone marrow and fat emboli (upto 80%)
• Damage to intra-abdominal organs (Liver, spleen, kidneys,
stomach).
7. What is MAST and its indications?
MAST is military antishock trouser, an inflatable three
compartment garment that surrounds abdomen and both
lower limbs. When inflated, it raises peripheral resistance,
increases venous return and hence cardiac output. Indications
for use include systolic BP < 80 mm Hg. secondary to hypo-
volemic shock, pelvic/femoral fracture that cause exsangui-
nation and need immobilisation. It is contraindicated in
pneumothorax.
8. Which is better—crystalloid or colloid?
Colloids like plasma and albumin though, are theoretically
better, as they remain in intravascular compartment, they may
leak through anoxic dilated capillary bed. Hence, for
emergency, Ringer lactate is best. If hypotension persists after
infusion of 2 litres of ringer lactate in a patient of haemor-
rhagic shock, blood transfusion is the answer. Stored blood
is poor in platelets, factor V and VIII. For every 5 units of
blood 1 ample (500 mg) of calcium chloride and one unit of
fresh plasma be given.
9. Ways of extending mechanical assistance to failing circu-
lation
• Intra-aortic balloon counterpulsation
• Left ventricular assist device.
10. Which vasopressures are helpful in emergency?
Dopamine 2-10 μ gm/kg/min
Dobutamine 2-4 μ gm/kg/min
Epinephrine 0.5-1 ml of 1:1000 sol slow IV
Mephentine 30 mg IV
CASE – 30 235

10. Role of Sodium bicarbonate


Hypoxia induced anaerobic metabolism and CO2 retention
lead to acidosis, which should be neutralized by NaHCO3. 1
mEq/kg given IV. Adequate ventilation is required for bic-
arbonate therapy to be effective since bicarbonate combines
with H+ to produce CO2 and H2O.
11. Components of Ringer’s lactate
Na+ 130 mEq/L
K+ 4 mEq/L
++
Ca 3 mEq/L
Cl– 110 mEq/L
Lactate 28 mEq/L

12. Role of Swan-Ganz catheter


Swan-Ganz catheter measure pulmonary artery wedge
pressure, which parallels left atrial filling pressure. Latter is
much more useful than central venous pressure in monitoring
volume status. Cardiac output and various other cardiovas-
cular parameters can also be measured.
13. What is “Third spacing”?
Pathophysiologic processes frequently seen in surgical
patients (shock, ischaemic injury, trauma, peritonitis, and
bowel obstruction) result in rapid accumulation of fluid in
tissues and hollow viscus spaces secondary to anoxic dilata-
tion of capillary bed. This fluid is drawn from vascular spaces,
resulting in functional volume depletion, requiring additional
fluid replacement.
14. Volume of blood loss necessary to produce shock
20% blood loss—Mild shock
20-40% blood loss—Moderate shock
40% blood loss—Severe shock
15. Types of shock
4 principal types—hypovolemic, cardiogenic, septic and
neurogenic.
236 SHORT NOTES AND SHORT CASES IN SURGERY

16. Clinical picture of shock


Mild Moderate Severe
Cold clamny skin Pale, thirsty Apathetic or
Raised heart rate Low BP comatose
Normal BP Low urine output Rapid thready pulse
Postural hypotension Anxious Shallow rapid breath
Flat neck veins Mottled skin
Unrecordable BP
17. What is “Warm Shock”?
Patients of septic or neurogenic shock appear warm and
flushed. They have “third space” volume loss and hence
features of circulatory failure. Analphylactic shock and
adrenal failure also produce shock by similar mechanism and
are grouped as “distributive shock”.
18. Pathophysiology of shock
Ischaemia mediates the tissue injury leading to anaerobic
metabolism and lipid peroxidation with production of highly
toxic O2 radicals, H2O2 and hydroxyl radicals. These agents,
further damage the normal tissues, maintain or aggravate
shock after reperfusion and are responsible for post-shock
states like acute ‘respiratory distress syndrome and multiple
organ failure.

Anda mungkin juga menyukai